Você está na página 1de 98

Socio - Economic Development Test - 2

1. Which of the following criteria are calculated under Social Inclusion to Ensure Equal Access to Economic Opportunity?
1.Pupil-teacher ratio (at primary and secondary level).
2.Percentage of population using improved drinking water sources.
3.Gender parity in labor force participation.
4.Growth rate of average per capita income/consumption.
Codes:

(A) 1, 2 and 3
(B) 3 and 4
(C) 2, 3 and 4
(D) All
Selected answer: (Not selected..)
The correct answer is:(A)
Solution:Social Inclusion to Ensure Equal Access to Economic Opportunity-
1. Access and Inputs to Education and Health-:
School life expectancy (primary to tertiary)
Pupil-teacher ratio (primary)
Diphtheria, tetanous toxoid, and pertussis (DTP3) immunization coverage among 1-year-olds
Physicians, nurses, and midwives per 10,000 population
Government expenditure on education as percentage of total government expenditure
Government expenditure on health as a percentage of total government expenditure
2. Access to Basic Infrastructure Utilities and Services-:
Percentage of population with access to electricity
Share of population using solid fuels for cooking
Percentage of population using improved drinking water sources
Percentage of population using improved sanitation facilities
3. Gender Equality and Opportunity-:
Gender parity in primary, secondary, and tertiary education
Antenatal care coverage (at least one visit)
Gender parity in labor force participation
Percentage of seats held by women in national parliament
4. Social Safety Nets-:
Social protection and labor rating
Social security expenditure on health as a percentage of government expenditure on health
Government expenditure on social security and welfare as percentage of total government expenditure

2. RSBY scheme has had little or no impact on medical impoverishment in India because:
1.RSBY provides the participating BPL household with no freedom of choice between public and private hospitals.
2.Hospital gain zero incentive for providing treatment to large number of beneciaries.
3.Lack of cashless benets to the patients.
4.Pre-existing health conditions are not covered under RSBY.
Which of the above statements are correct?

(A) 1, 2 and 4
(B) None
(C) 3 and 4
(D) 2, 3 and 4
Selected answer: (Not selected..)
The correct answer is:(B)
Solution:RSBY: RSBY provides the participating BPL household with freedom of choice between public and private hospitals and makes him a potential client worth attracting on
account of the signicant revenues that hospitals stand to earn through the scheme. A hospital has the incentive to provide treatment to large number of beneciaries as it
is paid per beneciary treated. Cash less and Paperless transactions - A beneciary of RSBY gets cashless benet in any of the empanelled hospitals. He/ she only needs to
carry his/ her smart card and provide. Pre-existing conditions are covered from day one and there is no age limit.

3. The rapid growth of cities in India, coupled with increasing rural to urban migration has led to rapid & unsustainable urbanization. In the light if above statements
consider the following statements regarding the 'Smart Cities Mission':
1.Only infrastructure projects, and with larger public benet, will get central assistance in the 'smart city' scheme.
2.Each project will be operated through 50-50 Special Purpose Vehicles between the Union and the State.
3.Under this 35 per cent of housing in Greeneld projects must be for economically weaker sections.
4.Smart cities Mission is in consonance with the 17 Global Goals that make up the 2030 Agenda for Sustainable Development.
Which of the above statements is/are incorrect?

(A) 1 and 3
(B) Only 2
(C) Only 3
(D) 1 and 4
Selected answer: (Not selected..)
The correct answer is:(B)
Solution:Under this each city can get Rs 100 crore of central funds annually for ve years; the rest has to be nanced by states, local bodies and private parties. The project will be
operated through 50-50 Special Purpose Vehicles between states and urban local bodies. It is also true that only infrastructure projects, and with larger public benet, will
get central assistance in the 'smart city' scheme.

4. Presently, the Metropolitan Planning Committees/District Planning Committees plan on which of the following areas?
1.Planning for the metropolitan region including the areas falling outside municipal limits for sustainable development.
2.Planning for the rural urban fringe.
Codes:

(A) Only 1
(B) Only 2
(C) Both
(D) None
Selected answer: (Not selected..)
The correct answer is:(D)
Solution:Presently the areas/villages/rural urban fringe falling outside the jurisdiction of municipality are not included in the plan

5. Undernutrition and poor dietary intake of iron is a serious public health problem among pregnant women and infants in India. Which of the following initiatives have
been taken by the GOI to reduce the anemic deciency?
1.Iron and folic acid supplementation to children till age of 12 years.
2.Mother and Child Protection Card to monitor service delivery for mothers and children.
3.De-worming initiative for the children.
4.Provide Vitamin A supplementation for all children in age group of 6-59 months.
Codes:

(A) 1, 2 and 3
(B) 2, 3 and 4
(C) 1, 3 and 4
(D) All
Selected answer: (Not selected..)
The correct answer is:(B)
Solution:The steps taken are:
Universal screening of pregnant women for anaemia is an integral part of ante-natal care, through the existing network of sub-centers and primary health centres and other
health facilities as well as through outreach activities at Village Health & Nutrition Days (VHNDs).
Iron and folic acid supplementation to pregnant and lactating women in the recommended daily dose of 100 mg of elemental iron and 0.5 mg of folic acid for at least 100
days for prevention and double the dose for treatment; to children 6-59 months in the daily dose of 20 mg of elemental iron and 100 mcg of folic acid for 100 days and to
children 6-10 years in the daily dose of 30 mg elemental iron and 250 mcg folic acid for 100 days.
To reduce the prevalence of anemia in adolescent girls, both in and out of school, Government of India has launched a scheme of Weekly Iron and Folic Acid
Supplementation (WIFS) all over the country.
Mother and Child Protection Card in collaboration with the Ministry of Women and Child Development to monitor service delivery for mothers and children.
Non -Pregnant and Non Lactating women are given Weekly Iron and Folic Acid Supplementation
Engagement of 8.8 lakhs Accredited Social Health Activists (ASHAs) to generate demand and facilitate accessing of health care services by the community.
Janani Shishu Suraksha Karyakaram (JSSK) has been launched.

6. Consider the following statements related to HRIDAY:


1.HRIDAY is a government of India's initiative to develop green industrial corridors.
2.HRIDAY aims at strengthening cultural ties between India and Indian Ocean Region Nations.
3.HRIDAY aims to preserve and rejuvenate the rich cultural heritage of the country.
4.HRIDAY is a Central Scheme and the entire expenditure will be shared between Centre and States in the ratio of 75:25.
Which of the above statements is/are correct?

(A) 1 and 2
(B) 3 and 4
(C) Only 3
(D) Only 4
Selected answer: (Not selected..)
The correct answer is:(C)
Solution:The National Heritage Development and Augmentation Yojana (HRIDAY) that seeks to preserve and rejuvenate the rich cultural heritage of the country. HRIDAY is a step
towards reviving the soul of cities hosting rich heritage'. HRIDAY seeks to promote an integrated, inclusive and sustainable development of heritage sites, focusing not just
on maintenance of monuments but on advancement of the entire ecosystem including its citizens, tourists and local businesses.' With 32 UNESCO recognized natural and
cultural heritage sites, ranking second in Asia and fth in the world, the tourism potential of India is still to be fully harnessed and the new scheme 'HRIDAY' will help in this
regard. Rs.500 cr would be provided to the 12 cities selected in the rst phase under the 'Central Scheme' of HRIDY, central government will meet the entire expenditure
under the scheme. However, states and local urban bodies are urged to supplement their resources for rapid development of heritage cities.

7. Which of the following can be the various methods to increase non-budgetary revenues of ULBs?
1.Monetization of Land.
2.Increase user charge collection.
3.Facilitating PPPP.
4.Empowering ULBs to leverage Municipality Bonds.
5.Bolstering State Finance Commission.
Codes:

(A) 1, 4 and 5
(B) 1 and 5
(C) 1, 2, 3 and 5
(D) All
Selected answer: (Not selected..)
The correct answer is:(D)
Solution:.

8. Development alone cannot bring peace and prosperity unless social justice and equality of older people are ensured. In the light of above statement considers the
following statements regarding Geriatric Issues in India:
1.Fundamental rights and DPSPs under the Indian constitutional document mandate well-being of older persons.
2.Social security has been made the state subject to ensure well being of old age people.
3.The National Policy on Senior Citizens covers all concerns pertaining to the welfare of older persons including employment.
Which of the above statements are correct?

(A) 1 and 2
(B) 1 and 3
(C) 2 and 3
(D) All
Selected answer: (Not selected..)
The correct answer is:(B)
Solution:International convention, legal enactments, constitutional provisions etc emphasises the need for equality and empowerment of older people. Well-being of older persons
has been mandated in the Constitution of India. Article 41, Directive Principle of State Policy stipulates that the State shall, within the limits of its economic capacity and
development, make eective provision for securities, right to public assistance in case of old age. There are other provisions too, which mandates the state to improve the
quality of life of its citizens. Right to equality as a Fundamental right further strengthen older persons. Social security has been made the concurrent responsibility of the
central and state Governments.
Ministry of Social Justice & Empowerment is the nodal Ministry responsible for welfare of the Senior Citizens.
The Government has formulated the National Policy on Older Persons, which was announced in the year 1999 covering all aspects concerning older persons. It is to be
noted that the Government of India has constituted the National Commission for older persons (NCOP) on 10th May 1999 under the Chairmanship of Minister for Social
Justice and Empowerment. The NCOP is the highest body to advice and coordinate with the Government in the formulation and implementation of policy and programmes
for the welfare of the aged. The NCOP has been reconstituted in 2005. The present strength of NCOP is 47.

9. Increasing the female labour force participation rate is a global concern. Consider the following statements regarding the Gender Gap in Labour Force:
1.Labour Force Participation Rate (LFPR) for women is signicantly higher than that for males in rural areas.
2.G-20 countries have targeted to reduce the gender gap rate by 25% by 2025.
3.To promote participation of women in the workforce through skilling and gender mainstreaming is one of the missions of National Policy for Skill development and
Entrepreneurship.
Which of the above statements are correct?

(A) 1 and 2
(B) 2 and 3
(C) Only 3
(D) All
Selected answer: (Not selected..)
The correct answer is:(B)
Solution:Labour Force Participation Rate (LFPR) for women is signicantly lower than that for males in both rural and urban areas (Source-PIB). During G20 Summit Brisbane 2014 the
leaders agreed to reduce the gap in labour force participation rates between men and women by 25% by 2025. This would bring more than 100 million women into the
workforce, increase global growth and reduce poverty and inequality. The OECD, with the ILO, has been charged with implementing this commitment which will help achieve
the growth targets. Promote increased participation of women in the workforce through skilling and gender mainstreaming is one of the missions of National Policy for Skill
development and Entrepreneurship.
10. Consider the following statements regarding the forest rights of Scheduled Tribes and Other Traditional Forest Dwellers:
1.Under Forest Rights Act minor forest produce including timber are allowed for Scheduled Tribes and Other Traditional Forest Dwellers.
2.Under Forest Rights Act Gram Panchayat makes a recommendation about who has been cultivating land for how long and which minor forest produce is collected, etc.
Which of the above statements is/are correct?

(A) Only 1
(B) Only 2
(C) Both
(D) None
Selected answer: (Not selected..)
The correct answer is:(D)
Solution:Under the Scheduled Tribes and Other Traditional Forest Dwellers (Recognition of Forest Rights) Act, 2006, minor forest produce things like tendu patta, herbs, medicinal
plants etc "that has been traditionally collected is allowed. This does not include timber. Section 6 of the Act provides a transparent three step procedure for deciding on
who gets rights. First, the Gram Sabha (full village assembly, NOT the gram panchayat) makes a recommendation - i.e who has been cultivating land for how long, which
minor forest produce is collected, etc. The gram sabha plays this role because it is a public body where all people participate, and hence is fully democratic and transparent.
The gram sabha's recommendation goes through two stages of screening committees at the taluka and district levels. The district level committee makes the nal decision.

11. Which of the following is/are the objectives of Unnat Bharat Abhiyaan?
1.Unnat Bharat Abhiyaan focuses on improving nutrition level of adolescent girls.
2.Unnat Bharat Abhiyaan focuses on industrialization in rural areas.
3.Unnat Bharat Abhiyaan relates to skill development of women workers in the unorganized sector.
4.Unnat Bharat Abhiyaan enable higher educational institutions to work with the people of rural India.
Codes:

(A) 2 and 3
(B) Only 4
(C) Only 3
(D) Only 2
Selected answer: (Not selected..)
The correct answer is:(B)
Solution:The Ministry of Human Resource Development (MHRD) has launched the Unnat Bharat Abhiyan (UBA) on 11th November, 2014 to enable higher educational institutions to
work with the people of rural India in identifying development challenges and evolving appropriate solutions for accelerating sustainable growth. The programme is being
launched in collaboration with the IITs, NITs. Unnat Bharat Abhiyan is being coordinated and steered by IIT Delhi. The programme involves engaging with neighbouring
communities and using technologies for their upliftment. The Ministry of Human Resource Development has advised all the higher educational institutions which are funded
by the Central/State Governments and all Institutions which are under approval by the regulatory bodies to adopt clusters of backward Gram Panchayats/ villages in their
vicinity and apply their knowledge and expertise to improve the infrastructure in the Gram Panchayats (GPs).

12. Consider the following statements regarding the Merit-cum Means based Scholarship Scheme:
1.Merit-cum Means based Scholarship Scheme is a Centrally Sponsored Scheme.
2.It is directly implemented by the Union Government under the Ministry of HRD.

3.The Merit-cum Means based Scholarship Scheme provides nancial assistance to the poor and meritorious minority students pursuing professional studies.
4.The Merit-cum Means based Scholarship Scheme also provides nancial assistance to the poor and meritorious Bhutanese nationals.
Which of the above statements are incorrect?

(A) 1, 2 and 3
(B) 1, 2 and 4
(C) 2 and 4
(D) 3 and 4
Selected answer: (Not selected..)
The correct answer is:(C)
Solution:Merit-cum Means based Scholarship Scheme is a centrally Sponsored Scheme. The objective of the Scheme is to provide nancial assistance to the poor and meritorious
students belonging to minority communities to enable them to pursue professional and technical courses. These scholarships are available for studies in India only and will
be awarded through State Government/UT Administration or an Agency designated by the State Government/UT Administration for this purpose. It is administered by
Ministry of Minority Aairs.

13. Consider the following statements:


1.Census town refers to an urban area having a functioning Urban local government unit like municipality or corporation.
2.Statutory towns are urban areas where population is more than 5000 and more that 75% of male working population engages in non agricultural profession.
Which of the above statements is/are correct?

(A) Only 1
(B) Only 2
(C) Both
(D) None
Selected answer: (Not selected..)
The correct answer is:(D)
Solution:Census town: Places that satisfy the following criteria (population>5000, 75% of the male main working population engage in non agricultural pursuit, density>400/sqkm).
Census'towns are agglomerations that grow in rural and peri-urban areas, with densication of population that do not have an eective urban governance structure or
requisite urban infrastructure, for example, sanitation, roads and so on in place.
Statutory town:'Statutory'towns are towns with municipalities or corporations.
Census 2011 notes that the number of towns in India increased from 5,161 in 2001 to as many as 7935 in 2011. It points out that almost all of this increase was in the growth
of 'census'towns (which increased by 2,532) rather than 'statutory'towns (which increased by only 242).

14. Which of the following initiative may ensure decentralized urban governance?
1.Setting up of Area Sabhas and Ward Committees.
2.Clear demarcation of areas of operation of ULBs and other parastatals.
3.Empowering Mayor.
Codes:

(A) Only 1
(B) 1, 2 and 3
(C) 2 and 3
(D) 1 and 3
Selected answer: (Not selected..)
The correct answer is:(D)
Solution:Demarcation of work between ULB and other parastatals aids easy and ecient service delivery and accountability and not decentralization.

15. Village Grain Bank Scheme is concerned with:

(A) Providing deposit facility to small and marginal farmers in case of excess crop.
(B) Providing facility to borrow food grains in food scarce and calamity hit areas.
(C) Giving loans for food supply chain creation in rural areas.
(D) Providing a single window mechanism for farmers to procure agricultural inputs.
Selected answer: (Not selected..)
The correct answer is:(B)
Solution:The main objective of the scheme was to provide safeguard against starvation during the period of natural calamity or during lean season when the marginalized food
insecure households did not have sucient resources to purchase rations.
Such people in need of food grains will be able to borrow food grains from the Village Grain Bank. The grain banks are to be set up in food scarce areas like the drought
prone areas, the hot and cold desert areas, tribal areas and the inaccessible hilly areas which remain cut o because of natural calamities like oods, etc. These villages are
to be notied by the concerned State Government/Union Territory. The scheme envisages inclusion of all willing BPL/AAY families in the villages to be identied by the State
Government in food decit areas. The quantity to be lent and the period of repayment is to be decided by the Group themselves. Village Panchayat/Gram Sabha, Self Help
Group for NGOs etcare identied by the state governments and eligible for running the Grain banks.)

16. Which of the following functions have been assigned to municipalities under 74th Amendment Act?
1.Urban poverty alleviation.
2.Promotion of Culture, educational and aesthetic aspects.
3.Maintenance of Public Distribution System.
4.Technical training & vocational education.
5.Prevention of cruelty to animals.
Codes:

(A) 1, 2 and 3
(B) 1, 2, 4 and 5
(C) 1, 2 and 5

(D) All
Selected answer: (Not selected..)
The correct answer is:(C)
Solution:Basic Urban functions are:
Urban planning including town planning.
Regulation of land use and construction of buildings.
Planning for economic and social development.
Roads and bridges.
Water supply for domestic, industrial and commercial purpose.
Public health, sanitation, conservancy and solid waste management.
Fire services
Urban forestry protection of the environment and promotion of ecological aspects.
Safeguarding the interests of weaker sections of society including the handicapped and mentally retarded.
Slum improvement and up gradation
Urban poverty alleviation
Provision of urban amenities and facilities such as parks, gardens, playgrounds.
Promotion of cultural, educational and aesthetic aspects.
Burials and burial grounds, cremations grounds and electric crematoriums.
Cattle ponds, prevention of cruelty to animals.
Vital statistics including registration of births and deaths.
Public amenities including street lighting, parking lots, bus stops and public conveniences.
Regulation of slaughter houses and tanneries.

17. The nances of ULB can be strengthened by which of the following strategies?
1.Create robust tax and non-tax based revenue streams for ULBs.
2.Attract private capital to the urban sector.
3.Systematically monetize land with sustainable development.
Codes:

(A) 1 and 2
(B) Only 2
(C) All
(D) 1 and 3
Selected answer: (Not selected..)
The correct answer is:(C)
Solution:According to the report, the nances of ULBs could be strengthened with a three- pronged approach:
Create robust tax and non-tax based revenue streams for ULBs;
Attract private capital to the urban sector; and
Systematically monetize land.

18. With reference to the provisions related to panchayats in the Indian Constitution, which of the following statements is/are incorrect?
1. To reserve seats (both members and chairpersons) for SCs, STs, OBCs and women in Panchayats at all three levels is a compulsory provision.
2. The PESA Act, 1996 is applicable only to Fifth Schedule Areas.
Select the correct answer using the code given below:
Codes:

(A) Only 1
(B) Only 2
(C) Both
(D) None
Selected answer: (Not selected..)
The correct answer is:(A)
Solution:Providing reservation of seats for OBCs in Panchayats at any level is a voluntary provision.

19. What is Greening Rural Development initiative of Ministry of Rural Development?

(A) An attempts to factor environmental costs into the nancial results of schemes of the ministry.
(B) Integrating environmental con-siderations in implementation of the programs of the ministry to improve ecological balance.
(C) Promote Agroforestry under UN sponsored Reducing Emissions from Deforestation and Forest Degradation plus scheme.
(D) None of the Above.
Selected answer: (Not selected..)
The correct answer is:(B)
Solution:Greening Rural Development initiative of Ministry of Rural Development integrate environmental considerations in implementation of the programmes, to help improve
ecological balance and provide a cleaner, healthier environment for the rural population. It denes 'green' outcomes for major rural development schemes, reviews the
design and evidence from the eld to highlight potential green results and recommends steps to improve green results.

20. Which of the following is the objective of Rashtriya Gram Swaraj Abhiyan?

(A) Help Panchayat Raj Institutions to deliver Sustainable Development Goals.


(B) Devolve more functions and nances under 73rd Amendment Act so as to make them living institutions of self-governance.
(C) Promote Digital literacy in rural India in convergence with National Optical Fibre Network Project.
(D) Implement revamped Bharat Nirman Mission.
Selected answer: (Not selected..)
The correct answer is:(A)

Solution:Rashtriya Gram Swaraj Abhiyan' has been proposed to help Panchayat Raj Institutions deliver Sustainable Development Goals.

21. Which of the following statements is/are correct about 'Neeranchal'?


1. Neeranchal program is the Watershed component of Rashtriya Krishi Vikas Yojana.
2. Neeranchal National Watershed Project will get assistance from Asian Development Bank.
Select the correct answer using the code given below:

(A) Only 1
(B) Only 2
(C) Both
(D) None
Selected answer: (Not selected..)
The correct answer is:(D)
Solution:For achieving the major objectives of the Watershed Component of the Pradhan Mantri Krishi Sinchayi Yojana (PMKSY) and for ensuring access to irrigation to every farm
(Har Khet Ko Pani) and ecient use of water (Per Drop More Crop), Neeranchal has been launched.
Neeranchal will translate into better implementation outcomes of PMKSY. The programme will lead to reducing surface runo of rainwater, increasing recharge of ground
water and better availability of water in rainfed areas resulting in incremental rainfed agriculture productivity, enhanced milk yield and increased cropping intensity through
better convergence related programmes in project areas.
Neeranchal is designed to further strengthen and provide technical assistance to the Watershed Component of PMKSY, in particular and all components of PMKSY, in
general, to enhance its delivery capacity. Neeranchal will support the Watershed component of PMKSY (erstwhile IWMP) which was implemented by the Department of Land
Resources (DoLR) in 28 States.
It will be done with assistance of World Bank.

22. The Atal Mission for Rejuvenation and Urban Transformation will focus on which of the following Thrust Areas?
1. Water Supply.
2. Sewerage facilities.
3. Storm Water drains formation.
4. Public transport facilities.
5. Green spaces and recreation centers, especially for children.
Codes:

(A) 1, 2 and 3
(B) 1, 2, 3 and 4
(C) 1, 2, 3 and 5
(D) All
Selected answer: (Not selected..)
The correct answer is:(D)
Solution:The Mission will focus on the following Thrust Areas:
Water supply,
Sewerage facilities and septage management,
Storm water drains to reduce ooding,
Pedestrian, non-motorized and public transport facilities, parking spaces, and
Enhancing amenity value of cities by creating and upgrading green spaces, parks and recreation centers, especially for children.

23. The Stand up India component under "Start up India, Stand up India" is anchored by Department of Financial Services (DFS) to encourage greeneld enterprises by
which of the following category of entrepreneurs?
1.SC/ST entrepreneurs
2. Women entrepreneurs
3. Minority entrepreneurs
Codes:

(A) 1, 2 and 3
(B) Only 1
(C) 1 and 2
(D) 1 and 3
Selected answer: (Not selected..)
The correct answer is:(C)
Solution:Stand Up India Scheme" to promote entrepreneurship among SC/ST and Women entrepreneurs. The Scheme is intended to facilitate at least two such projects per bank
branch, on an average one for eachcategory of entrepreneur. It is expected to benet atleast 2.5 lakh borrowers.
The expected date of reaching the target of at least 2.5 lakh approvals is 36 months from the launch of the Scheme.
The Stand Up India Scheme provides for:
Renance window through Small Industries Development Bank of India (SIDBI) with an initial amount of Rs. 10,000 crore.
Creation of a credit guarantee mechanism through the National Credit Guarantee Trustee Company (NCGTC).
Handholding support for borrowers both at the pre loan stage and during operations. This would include increasing their familiarity with factoring services, registration with
online platforms and e-market places as well as sessions on best practices and problem solving.

24. Which of the following statements is/are true regarding the Pradhan Mantri Khanij Kshetra Kalyan Yojana (PMKKKY)?
1. The scheme covers only those people who are directly aected in mining areas.
2. The scheme earmarks 60% of expenditure of this fund for high priority areas, and 40% towards other priority areas.
3. The scheme will utilize the fund accruing to District Mineral Foundation established under the MMDR Amendment Act, 2015.
Select the correct answer using the codes given below:
Codes:

(A) 1 and 3
(B) 2 and 3
(C) Only 1
(D) 1, 2 and 3
Selected answer: (Not selected..)
The correct answer is:(B)
Solution:This is a new programme meant to provide for the welfare of areas and people aected by mining related operations, using the funds generated by District Mineral
Foundations (DMFs). The objective of PMKKKY scheme will be (a) to implement various developmental and welfare projects/programs in mining aected areas that
complement the existing ongoing schemes/projects of State and Central Government; (b) to minimize/mitigate the adverse impacts, during and after mining, on the
environment, health and socio-economics of people in mining districts; and (c) to ensure long-term sustainable livelihoods for the aected people in mining areas. Care has
been taken to include all aspects of living, to ensure substantial improvement in the quality of life. High priority areas like drinking water supply, health care, sanitation,
education, skill development, women and child care, welfare of aged and disabled people, skill development and environment conservation will get at least 60 % share of the
funds. For creating a supportive and conducive living environment, balance funds will be spent on making roads, bridges, railways, waterways projects, irrigation and
alternative energy sources. This way, government is facilitating mainstreaming of the people from lower strata of society, tribals and forest-dwellers who have no
wherewithal and are aected the most from mining activities.

25. Which of the following are the goals of SABLA scheme?


1.Mainstream out of school adolescent girls into formal/non-formal education.
2.Promote awareness about health, hygiene, nutrition, adolescent reproductive and sexual health (ARSH), and family and child care.
3.Improve nutrition and health status of girls.
4.Organize them into Self Help Groups and induce specic skill sets.
Codes:

(A) 1, 3 and 4
(B) 1, 2 and 3
(C) 2, 3 and 4
(D) All
Selected answer: (Not selected..)
The correct answer is:(B)
Solution:It does not envisage organizing adolescent girls into SHGs.

Information for the solved exam

Started in: March 27, 2017, 12:49 am

Finished in: March 27, 2017, 12:49 am

Marks Obtained: 0

Right Ques.: 0

Wrong Ques.: 0

Unattempted Ques.: 25

Try for a better result? (index.php?section=user&page=continue_exam&id=133436&renew_exam=yes)

Rank List (index.php?section=user&page=ranking&id=37)


Socio - Economic Development Test - 3

1. Consider the following statements regarding tax proposals in Budget 2015-16:


1.No change has been proposed in rate of personal income tax.
2.Proposal has been made to reduce corporate tax from 30% to 25% over the next four years.
3.Rate of Income-tax on royalty and fees for technical services has been reduced from 25% to 10% to facilitate technology inow.
4.Wealth-tax has been replaced with additional surcharge of 2 per cent on super rich with a taxable income of over Rs. 1 crore annually.
Which of the above stated proposals is/are correct?

(A) 2, 3 and 4
(B) 1 and 2
(C) 3 and 4
(D) All
Selected answer: (Not selected..)
The correct answer is:(D)
Solution:.

2. Recently central government has launched Vanbandhu Kalyan Yojana (VKY) for:

(A) Denotied tribes


(B) STs
(C) Forest dwellers in biosphere reserves
(D) Those depending on minor forest produce
Selected answer: (Not selected..)
The correct answer is:(B)
Solution:.

3. Match the following lists:


List-1 (Sector) List-2 (FDI limit)
A.FM Radio 1. 26%
B.Medical devices 2. 100%
C.Civil Aviation 3. 74%
D.Public Sector Banking 4. 20%
Codes:
ABCD

(A) 1324
(B) 1234
(C) 4231
(D) 4321
Selected answer: (Not selected..)
The correct answer is:(B)
Solution:.

4. The government has recently proposed National eHealth Authority (NeHA) for India under Digital India programme. NeHA would be responsible for:
1.To guide the adoption of e-Health solutions at various levels and areas in the country in a manner that meaningful aggregation of health and governance data and
storage/exchange of electronic health records happens at various levels in a cost-eective manner.
2.To facilitate integration of multiple health IT systems through health information exchanges.
3.To oversee orderly evolution of state-wide and nationwide Electronic Health Record Store/Exchange System which ensures that security, condentiality and privacy of
patient data is maintained and continuity of care is ensured.
Which of the above stated functions are correct?

(A) 1 and 2
(B) 1 and 3
(C) 1, 2 and 3
(D) 2 and 3
Selected answer: (Not selected..)
The correct answer is:(C)
Solution:.

5. Match the following lists:


List-1 (Five Year Plan) List-2 (Specication)
A.First Plan 1.Harrod-Domar Model.
B.Second Plan 2.Mahalanobis Plan.
C.Fourth Plan 3.Growth with sta-bility.
D.Fifth Plan 4. Attainment of self reliance.
Codes:
ABCD

(A) 1243
(B) 1234
(C) 2134
(D) 2143
Selected answer: (Not selected..)
The correct answer is:(B)
Solution:.

6. Consider the following statements related to IGMSY:


1.Indira Gandhi Matritva Sahyog Yojana envisages cash transfer to all eligible women who are pregnant and lactating for rst two live births.
2.The beneciaries under IGMSY can be women above 21 years of age and will be paid Rs. 3000 in four installments for cash component to partly compensate for wage loss
incurred during pregnancy.
3.Encouraging early and exclusive breast feeding also forms a part of this scheme.
Which of the above statements are correct?

(A) 1 and 2
(B) 1 and 3
(C) 2 and 3
(D) All
Selected answer: (Not selected..)
The correct answer is:(B)
Solution:The beneciaries under IGMSY can be women above 19 years of age and will be paid Rs. 4000 in three installments for cash component to partly compensate for wage loss
incurred during pregnancy.

7. Consider the following statements about the steps taken by GOI for skill development:
1.GOI is integrating skill enhancement and entrepreneurship in the syllabi at the school level.
2.GOI is setting up of multi-skilling institutions in the Public-Private Partnership (PPP) mode.
3.Government of India has restructured teacher training system.
Which of the above stated initiatives is/are correct?

(A) 1 and 2
(B) 2 and 3
(C) Only 2
(D) All
Selected answer: (Not selected..)
The correct answer is:(D)
Solution:.

8. Under Universal Immunization program, government of India is providing vaccine to prevent which of the following preventable diseases?
1.Diphtheria2.Tetanus
3.Polio4.Hepatitis C
5.Measles6.Tuberculosis
Codes:

(A) 1, 2, 3, 5 and 6
(B) 1, 2, 4 and 5
(C) 2, 3, 4 and 5
(D) All
Selected answer: (Not selected..)
The correct answer is:(A)
Solution:The UIP includes seven vaccine preventable diseases: Diphtheria, Pertussis, Tetanus, Polio, Measles, severe form of childhood Tuberculosis and Hepatitis B.

9. Government of India has recently launched the Beti Bachao Beti Padhao Yojana to improve the child sex ratio (0-6 years). What are the reasons for decline in child sex
ratio?
1.Gender preferences and daughter discrimination.
2.Faulty population policies.
3.Economic considerations associated with daughters.
4.Modernization of technologies.
Codes:

(A) 1, 2 and 3
(B) 2, 3 and 4
(C) 1 and 3
(D) All
Selected answer: (Not selected..)
The correct answer is:(D)
Solution:.

10. Which of the following constitute the features of Atal Pension Yojna?
1.The Atal Pension Yojana is open to all Indians between the age of 18 and 40, the benets of which will be availed at 60 years of age.
2.All existing members of the government's 'Swavalamban Yojana NPS Lite' will automatically be migrated to the Atal Pension Yojana.
3.The pension amount is not xed and the amount of contribution and the individual's age will determine the pension.
Codes:

(A) 1 and 2
(B) 1 and 3
(C) 2 and 3
(D) All
Selected answer: (Not selected..)
The correct answer is:(D)
Solution:.

11. For free and compulsory education for all children until they complete the age of fourteen years which of the following parts of the constitution have provision?
1.Part III - Fundamental Rights
2.Part IV - Directive Principles of State Policy
3.Part IV A - Fundamental Duties
4.Part IX and IX A - Dealing with Rural/Urban local bodies
Codes:

(A) 1 and 3
(B) 1 and 2
(C) 1, 2 and 3
(D) All
Selected answer: (Not selected..)
The correct answer is:(C)
Solution:Part IX and IX-A do not speak about the free and compulsory education for primary school goers.

12. Which of the following services has/have been exempted from service tax as per Budget 2015-16?
1.All ambulance services provided to patients.
2.Services of pre-conditioning, pre-cooling, ripening, etc. of fruits and vegetables.
3.Admission to museum, zoo, national park, wildlife sanctuary and tiger reserve.
4.Transport of goods for export by road from factory to land customs station.
Codes:

(A) Only 1
(B) 1 and 2
(C) 1, 2 and 3
(D) 1, 2, 3 and 4
Selected answer: (Not selected..)
The correct answer is:(D)
Solution:.

13. Which of the following National Health Programmes come under the umbrella of National Rural Health Mission (NRHM)?
1.National Vector Borne Disease Control Programme.
2.National Leprosy Eradication Programme.
3.Revised National Tuberculosis Control Programme.
4.National Iodine Deciency disorder Control Programme.
5.National programme for Prevention and Control of Cancer.
Codes:

(A) 1, 2 and 3
(B) 1, 3, 4 and 5
(C) 1, 2, 3 and 4
(D) All
Selected answer: (Not selected..)
The correct answer is:(C)
Solution:Following National Health Programmes come under the umbrella of National Rural Health Mission (NRHM):
1.National Vector Borne Disease Control Programme
2.National Leprosy Eradication Programme
3.Revised National Tuberculosis Control Programme
4.National Iodine Deciency disorder Control Programme
5.National Programme for Control of Blindness

14. Consider the following statements related to Deen Dayal Upadhyaya Gram Jyoti Yojana (DDUGJY):
1.It has been launched with the objective of separating agriculture and non-agriculture feeders to facilitate distribution companies (Discoms) in the judicious rostering
of supply to agricultural and non-agricultural consumers.
2.Under the new scheme, all Discoms including private sector Discoms are eligible for availing of nancial support.
Which of the statements given above is/are correct?

(A) Only 1
(B) Only 2
(C) Both
(D) None
Selected answer: (Not selected..)
The correct answer is:(C)
Solution:.

15. Which of the following are considered as critical objectives for environment and development policies that follow from the concept of sustainable development?
1. Changing the quality of growth
2. Conserving and enhancing the resource base.

3. Ensuring a sustainable level of population.


4. Merging environment and economics in decision making.
Which of the above statements are correct?

(A) 1, 2 and 3
(B) 2, 3 and 4
(C) 3 and 4
(D) All
Selected answer: (Not selected..)
The correct answer is:(D)
Solution:Critical objectives for environment and development policies that follow from the concept of sustainable development include:
reviving growth;
changing the quality of growth;
meeting essential needs for jobs, food, energy, water, and sanitation;
ensuring a sustainable level of population;
conserving and enhancing the resource base;
reorienting technology and managing risk; and
merging environment and economics in decision making

16. In which of the following years, Five Years Plans could not take o and were treated as annual plans?
1.1990-91
2.1991-92
3.1968-69
4.1979-80
Select the correct code:

(A) Only 1
(B) 1 and 2
(C) 1, 2 and 3
(D) 1, 2, 3 and 4
Selected answer: (Not selected..)
The correct answer is:(C)
Solution:1979-80 was rolling plan.

17. GST in India will replace all indirect taxes levied on goods and services by the Indian Central and State governments. It would be imposed on:
1.Raw material at each production stage
2.Final sale amount
3.First stage of production
4.All stages between production and nal sale
Which of the above statement/s is/are correct?

(A) Only 1
(B) Only 4
(C) 2 and 3
(D) 1 and 3
Selected answer: (Not selected..)
The correct answer is:(B)
Solution:.

18. Consider the following statements regarding Deepak Parekh Committee on Infrastructure:
1. It has recommended for establishment of a PPP model for power distribution, starting rst with the cities.
2. It has recommended that taris should be rationalized with a grading system that distinguishes between consumers depending on their paying capacity.
3. It has recommended that the consumers subject to market based pricing may be allowed to choose from dierent suppliers of electricity.
Which one of the above statements is/are correct?

(A) 1 and 3
(B) 2 and 3
(C) Only 3
(D) All
Selected answer: (Not selected..)
The correct answer is:(D)
Solution:.

19. The PESA (Panchayat Extension to Scheduled Areas) authorizes the State to give the Gram Sabhas power to regulate and restrict which amongst the following?
1.Sale/Consumption of Liquour.
2.Minor forest produce.
3.Identication of beneciaries of schemes.
4.Manage all mineral resources.
Codes:

(A) 1 and 2
(B) 1, 2 and 3
(C) 2 and 3
(D) All
Selected answer: (Not selected..)
The correct answer is:(B)
Solution:Gram Sabhas have been given some power to regulate minor minerals and not all the minerals.

20. India has been growing at a steady rate of economic growth throughout the post-reforms period. Which of the following has not been a feature of this growth?
1. Employment creation
2. Inter-sectoral balance
3. Sustainability
4. Inclusive
Codes:

(A) 1, 2 and 3
(B) 1, 2 and 4
(C) 2, 3 and 4
(D) None
Selected answer: (Not selected..)
The correct answer is:(C)
Solution:.

21. Which of the following is/are salient provisions of Regional Rural Bank (amendment) Bill, 2014?
1. This amendment bill increases the authorised capital of each Regional Rural Bank (RRB) from Rs 5 crore to Rs 2000 crore.
2. The Bill adds provision that any person who is a director of an RRB is also eligible to be on the Board of Directors of another RRB.
3. The bill raises the tenure of directors to 5 years from existing 2 years.
Codes:

(A) 1 and 3
(B) Only 1
(C) 2 and 3
(D) All
Selected answer: (Not selected..)
The correct answer is:(B)
Solution:The Bill adds provision that any person who is a director of an RRB is not eligible to be on the Board of Directors of another RRB.
The bill raises the tenure of directors to 3 years from existing 2 years.

22. Which of the following are the major causes of unemployment in India?
1. Inadequate Employment Planning.
2. Decline of Cottage Industries.
3. Fragmentation of Land.
4. Rapid Population Growth.
Which one of the above statements are correct?

(A) 1, 2 and 3
(B) 2, 3 and 4
(C) 3 and 4
(D) All
Selected answer: (Not selected..)
The correct answer is:(D)
Solution:.

23. Consider the following regarding Urjit R. Patel committee:


1. Committee has suggested that the RBI should adopt the new CPI (consumer price index) as the measure of the nominal anchor for policy com-munication.
2.It has recommended that the monetary policy decision-making should be vested with RBI.
3. The committee asked the Central Government to ensure that the scal decit as a ratio to GDP (gross domestic product) is brought down to 3.0 per cent by 2016-17.
Which one of the above statement/s is/are correct?

(A) 1 and 3
(B) 1 and 2
(C) Only 3
(D) All
Selected answer: (Not selected..)
The correct answer is:(A)
Solution:It has recommended that the monetary policy decision-making should be vested with a Monetary Policy Committee (MPC).

24. The 12th Five Year Plan has emphasized on a shift from primary agriculture to secondary agriculture in the country. What are the main advantages oered by the
secondary agriculture over primary agriculture?
1.Increase in production
2.Adds value
3.Creates jobs
4.Reduces pollution
Choose the correct option:

(A) 1 and 2
(B) 2 and 3
(C) 2, 3 and 4

(D) All
Selected answer: (Not selected..)
The correct answer is:(C)
Solution:The 12th plan would witness a shift from primary to secondary agriculture in the country. Secondary agriculture is very broad as it includes all food and non-food bio-
resource based products for human and industrial use. It aects all sectors of the society with special inclusive impact on the rural population as they majorly depend upon
agro-bio-resource to sustain their livelihoods.
Secondary agriculture provides the following benets:
1. Creates jobs
2. Reduces pollution
3. Adds value
4. Improves farm economy
5. Builds rural agro industries
6. Inclusive growth

25. Consider the following statements regarding Shanta Kumar Committee on Restructuring of FCI:
1. It recommended that FCI should outsource its stocking operations to various agencies such as Central Warehousing Corporation (CWC).
2. It recommends end to end computerization of the entire food management system, starting from procurement from farmers, to stocking, movement and nally
distribution through PDS.
3. It recommended that FCI procurement should focus on eastern belt, where farmers do not get minimum support price.
Which one of the above statement/s is/are correct?

(A) 1 and 2
(B) 2 and 3
(C) Only 1
(D) All
Selected answer: (Not selected..)
The correct answer is:(D)
Solution:.

Information for the solved exam

Started in: March 27, 2017, 12:47 am

Finished in: March 27, 2017, 12:47 am

Marks Obtained: 0

Right Ques.: 0

Wrong Ques.: 0

Unattempted Ques.: 25

Try for a better result? (index.php?section=user&page=continue_exam&id=133430&renew_exam=yes)

Rank List (index.php?section=user&page=ranking&id=94)


Socio - Economic Development Test - 4

1. Which of the following statement/s is/are true in context of Self Help Groups (SHGs):
1.It comprises a group of micro entrepreneurs having homogeneous social and economic backgrounds.
2.It is a village based nancial intermediary committee acting as a means of social as well as nancial inclusion.
3.Each Self Help Group require mandatory registration with village panchayat.
Codes:

(A) 1 and 2
(B) 1 and 3
(C) 2 and 3
(D) 1, 2 and 3
Selected answer: (Not selected..)
The correct answer is:(A)
Solution:SHGs can both be registered and unregistered. No mandatory registration is required.

2. Consider the following statements regarding Frictional unemployment:


1. It is the time period between jobs when a worker is searching for, or transitioning from one job to another.
2. Frictional unemployment exists because both jobs and workers are homogeneous.
3. Frictional unemployment is rarely present in an economy.
Which one of the above statement/s is/are correct?

(A) Only 1
(B) Only 2
(C) Only 3
(D) All
Selected answer: (Not selected..)
The correct answer is:(A)
Solution:Frictional unemployment exists because both jobs and workers are heterogeneous, and a mismatch can result between the characteristics of supply and demand. Such a
mismatch can be related to skills, payment, work-time, location, seasonal industries, attitude, taste, and a multitude of other factors.
Frictional unemployment is always present in an economy, so the level of involuntary unemployment is properly the unemployment rate minus the rate of frictional
unemployment, which means that increases or decreases in unemployment are normally under-represented in the simple statistics.

3. Consider the following statements regarding NITI Aayog:


1. It will be providing strategic and technical advice to the central and the state governments by adopting bottom-up approach.
2. Regional Councils will be formed to address specic issues and contingencies impacting more than one state or a region.
3. Niti Aayog has no power to allocate funds.
Which one of the above statement/s is/are correct?

(A) 1 and 3
(B) 2 and 3
(C) Only 3
(D) All
Selected answer: (Not selected..)
The correct answer is:(D)
Solution:.

4. The envisaged roles for MUDRA Bank include which of the following?
1. Undertake rating and accreditation of MFI entities.
2. Promote appropriate technology solutions for the last mile.
3. Support promotional and development activities in the sector and create a good architecture for last-mile credit delivery.
Which of the statement/s given above is/are correct?

(A) Only 2
(B) 2 and 3
(C) 1 and 3
(D) All
Selected answer: (Not selected..)
The correct answer is:(D)
Solution:.

5. Consider the following statements regarding Multidimensional Poverty Index (MPI):


1. It was developed in 2010 by Oxford Poverty & Human Development Initiative and World Bank.
2. It complements traditional income-based poverty measures by capturing the severe deprivations that each person faces at the same time with respect to education,
health and living standards.
3. The MPI assesses poverty at the individual level.
Which one of the above statement/s is/are correct?

(A) 1 and 2
(B) 2 and 3
(C) 1 and 3
(D) All
Selected answer: (Not selected..)
The correct answer is:(B)
Solution:It was developed in 2010 by Oxford Poverty & Human Development Initiative and the United Nations Development Programme.

6. Recently Lok Sabha passed Negotiable Instrumens Amendments Bill, 2015. Which of the following is true about Negotiable Instruments Act, 1881?
1.The Act denes bills of exchange, promissory notes, cheques and creates penalties for issues such as bouncing of cheques.
2.It also species circumstances under which complaints for cheque bouncing can be led.
Codes:

(A) Only 1
(B) Only 2
(C) Both
(D) None
Selected answer: (Not selected..)
The correct answer is:(C)
Solution:.

7. Which of the following are the features of digital green initiative?


1. It will provide training to farmers to make and show short videos where they can record their problems, share solutions and highlight success stories.
2. It will build on the existing extension systems of public, private, and civil society organizations and strengthens them to be more eective and ecient.
3. It will use an innovative digital platform for community engagement to improve livelihoods of rural communities of India only
Codes:

(A) 1 and 2
(B) 2 and 3
(C) 3 and 4
(D) 1 and 4
Selected answer: (Not selected..)
The correct answer is:(A)
Solution:Uses an innovative digital platform for community engagement to improve livelihoods of rural communities across South Asia and sub-Saharan Africa.

8. Companies operating in SEZ enjoy which of these privileges after withdrawl of STPI (Software Technology Park Initiative) Scheme?
1.Service Tax exemption
2.MAT exemption
3.CST exemption
4.Income Tax exemption
Codes:

(A) 1, 2 and 3
(B) 2, 3 and 4
(C) 1, 3 and 4
(D) All
Selected answer: (Not selected..)
The correct answer is:(A)
Solution:.

9. Consider the following statements regarding soil health card:


1. The SHC contains all basic and crop-wise recommendations of fertilizers or nutrients required for farms of dierent soil types.
2. It provides crop-wise nutrient recommendation and other inputs in order to increase the productivity of farms.
3. Punjab has become the rst state in India to issue Soil Health Cards (SHCs)
Which of the above statements are true?

(A) 1 and 3
(B) 1 and 2
(C) 2 and 3
(D) All
Selected answer: (Not selected..)
The correct answer is:(D)
Solution:.

10. Arrange the following institutions in terms of hierarchy:


1.NABARD
2.RRB
3.State Co-operative Banks
Codes:

(A) 1-2-3
(B) 3-2-1
(C) 2-3-1
(D) 3-1-2

Selected answer: (Not selected..)


The correct answer is:(C)
Solution:.

11. Which of the following is/are correct about MGNREGA Act 2005?
1. It promises minimum 100 days of skilled/ semi-skilled/unskilled manual work.
2. Unemployment allowance, if one can't get work within 30 days.
3. To each rural unskilled person of every rural household.
4. State governments have to appoint district level ombudsman to hear complaints.
Codes:

(A) 2, 3 and 4
(B) 1, 3 and 4
(C) 1, 2 and 4
(D) Only 4
Selected answer: (Not selected..)
The correct answer is:(D)
Solution:It promises minimum 100 days of unskilled manual work.
To each rural household. (not to each person).
Unemployment allowance, if one can't get work within 15 days.
State governments appoint district level ombudsman to hear complaints.

12. Which of the following is/are the core values that guide all the activities under NRLM?
1. Inclusion of the poorest and meaningful role to the poorest in all the processes.
2. Transparency and accountability of all processes and institutions.
3. Ownership and key role of the poor and their institutions in all stages - planning, implementation, and, monitoring
Codes:

(A) Only 2
(B) 2 and 3
(C) 1 and 3
(D) All
Selected answer: (Not selected..)
The correct answer is:(D)
.
Solution:.

13. Which of the following motive is/are not included in the NRUM?
1. Easy loan to street vendors.
2. Shelters for the homeless.
3. Women-SHG in backward districts get loans at cheaper interest rate.
Codes:

(A) 1 and 3
(B) 1 and 2
(C) Only 3
(D) All
Selected answer: (Not selected..)
The correct answer is:(C)
Solution:Under Aajiveeka, Women-SHG in backward districts get loans at cheaper interest rate.

14. Which of the following has been recommended to be the part of regulatory architecture of Financial Sector Legislative Reforms Commission?
1. Unied Financial Regulator
2. Unied Capital markets regulator
3. Financial Sector Appellate Tribunal
4. Financial Redressal Agency
Codes:

(A) 1, 2 and 4
(B) 1, 3 and 4
(C) 1, 2 and 3
(D) All
Selected answer: (Not selected..)
The correct answer is:(B)
Solution:The model of the proposed regulatory architecture will comprise the following agencies:
The central bank as the monetary authority, banking regulator and payment system regulator.
A unied regulator for the rest of the nancial sector.
A deposit insurance-cum-resolution agency.
A public debt management agency.
A nancial redressal agency.
A nancial sector appellate tribunal.
A mechanism for coordination, systemic risk, nancial development and other issues where the role of multiple agencies are involved (FSDC/similar to FSDC)

15. Which of the following statements are true about Pradhan Mantri Suraksha Bima Yojana?
1. It will be operated by Public sector banks.
2. Premium is xed at Rs 12 per annum.
3. It provides a cover of Rs 2,00,000 in case of death.
Codes:

(A) 1 and 2
(B) 1 and 3
(C) 2 and 3
(D) All
Selected answer: (Not selected..)
The correct answer is:(C)
Solution:Both public and private bank

16. Consider the following statements related to P-Notes:


1. Participatory Notes are derivative instruments issued in foreign jurisdictions, by a SEBI registered Foreign Institutional Investor (FII).
2. It is used to invest in Indian companies listed in foreign exchanges.
3. It leads to hedge funds.
Which of the above statements are correct?

(A) 1 and 2
(B) 1 and 3
(C) 2 and 3
(D) All
Selected answer: (Not selected..)
The correct answer is:(B)
Solution:P-Notes are nancial instruments used by investors or hedge funds that are not registered with the Securities and Exchange Board of India to invest in Indian securities.
Indian-based brokerages buy India-based securities and then issue participatory notes to foreign investors. Any dividends or capital gains collected from the underlying
securities go back to the investors.

17. To give a llip to the mining sector, proposal has been made to amend the provisions of the Mines and Minerals (Development & Regulation) Act, 1957 (MMDR) with the
objective of:
1. Providing greater transparency in allotment by auctioning mining leases.
2. Attracting private investment and high technology by promoting easy transferability.
3. Obtaining an increased share for the state governments.
Which of the above stated objectives is/are correct?

(A) 1 and 3
(B) Only 2
(C) 2 and 3
(D) All
Selected answer: (Not selected..)
The correct answer is:(D)
Solution:.

18. Consider the following statements regarding objectives of the "Sfruti", Scheme of Fund for Regeneration of Traditional Industries:
1. It aims at making traditional industries more competitive and productive.
2. It aims at generation of sustainable employment for artisans and rural and urban entrepreneurs.
3. It aims at strengthening the local governance system of Industry Clusters.
Which of the above stated objectives of the scheme is/are correct?

(A) Only 1
(B) 1 and 3
(C) 1 and 2
(D) All
Selected answer: (Not selected..)
The correct answer is:(B)
Solution:Objectives of the Scheme:
Development of Traditional Industries.
Making them more competitive and productive.
Sustainable employment for artisans and rural entrepreneurs.
Innovative Skills-Technology upgradation Developing public private partnership.
Strengthening the local governance system of Industry Clusters.

19. Consider the following statements about Pradhan Mantri Krishi Sinchai Yojana (PMKSY):
1. It aimed at ensuring access to water to every farm and improving water use eciency.
2. It aims at providing end-to-end solutions in the irrigation supply chain, including the water source, the distribution network and farm-level application.
Which of the above statements is/are correct?

(A) Only 1
(B) Only 2
(C) Both
(D) None
Selected answer: (Not selected..)
The correct answer is:(C)
Solution:.

20. Consider the following statements about Paramparagat Krishi Vikas Yojana (Traditional Farming Improvement Programme):
1. It will support and promote organic farming and thereby improving soil health.
2. It will encourage farmers to adopt eco-friendly concept of cultivation and reduce their dependence on fertilizers.
3. One of the objective is to optimally utilize the locally available natural resources for input production.
Which of the above statements is/are correct?

(A) 1 and 2
(B) Only 1
(C) Only 2
(D) All
Selected answer: (Not selected..)
The correct answer is:(D)
Solution:.

21. Government has undertaken a number of steps to improve Ease of Doing Business in India. Which of the following is/are among them?
1.A large number of components of Defence Products' list have been excluded from the purview of Industrial Licencing.
2.Process of obtaining environment and forest clearances has been made online.
3.The validity period of the Industrial Licence and security clearance from Ministry of Home Aairs has been increased.
Choose from the given codes:

(A) Only 1
(B) 2 and 3
(C) All
(D) None
Selected answer: (Not selected..)
The correct answer is:(C)
Solution:Government has undertaken following steps to improve Ease of Doing Business:
A large number of components of Defence Products' list have been excluded from the purview of Industrial Licencing. The application process for Industrial Licence and
Industrial Entrepreneur's Mem-orandum has been made easy by simplication of form and making the process online 24X7. The validity period of the Industrial Licence and
security clearance from Ministry of Home Aairs has been increased. The process of registration with Employees' Provident Fund Organization and Employees' State
Insurance Corporation has been made on line and real-time. Process of obtaining environment and forest clearances has been made online. The Department of Industrial
Policy and Promotion has advised Ministries and State Governments to simplify and rationalize the regulatory environment through business process reengineering and use
of information technology. Government of India services have been integrated with the online single window eBiz portal.

22. Which of the following are correct about the 'Nai Manzil' scheme announced in the budget?
1.The scheme will help in bridging the gap in education and skills of those passing out of madrasas.
2.It will help minority youth without a formal school-leaving certicate to obtain one and nd better employment.
Select the correct option from the codes given below:
Codes:

(A) Only 1
(B) Only 2
(C) Both
(D) None
Selected answer: (Not selected..)
The correct answer is:(C)
Solution:.

23. The "Make in India" initiative is based on four pillars, which have been identied to give boost to entrepreneurship in India, not only in manufacturing but also other
sectors. Which of the following is NOT among the four pillars?

(A) New Processes


(B) New Infrastructure
(C) New Sectors
(D) New Products
Selected answer: (Not selected..)
The correct answer is:(D)
Solution:The four pillars are:
(i) New Processes: 'Make in India' recognizes 'ease of doing business' as the single most important factor to promote entre-preneurship. A number of initiatives have
already been undertaken to ease business environment.
(ii) New Infrastructure: Government intends to develop industrial corridors and smart cities, create world class infrastructure with state-of-the-art technology and high-
speed communication. Innovation and research activities are supported through a fast paced registration system and improved infrastructure for IPR registration. The
requirement of skills for industry are to be identied and accordingly development of workforce to be taken up.
(iii) New Sectors: FDI has been opened up in Defence Production, Insurance, Medical Devices, Construction and Railway infrastructure in a big way. Similarly FDI has been
allowed in Insurance and Medical Devices.
(iv) New Mindset: In order to partner with industry in economic development of the country, Government shall act as a facilitator and not a regulator.

24. Consider the following statements about Mridaprikshak:


1. Mridaparikshak is a digital mobile quantitative mini lab/soil test kit to provide soil testing service at farmers' doorsteps.
2. It provides crop and soil specic fertilizer recommendation directly to farmerss mobile through SMS.
3. Mridaparikshak comes with soil sampling tools, GPS, a Smart Soil Pro, for determining the soil parameters and displaying of fertilizer nutrient recommendations.
Which of the above statements is/are correct?

(A) 1 and 2
(B) 1 and 3
(C) Only 3
(D) All
Selected answer: (Not selected..)
The correct answer is:(D)
Solution:.

25. Which of the following features are true in context of the Rangarajan Committee report on poverty estimation?
1.It takes normative levels for adequate nourishment, clothing, house rent, conveyance and education, alongwith behaviourally-determined levels of other non-food
expenses.
2.Poverty line is estimated on the basis of monthly expenditure of family of ve as against monthly per capita expenditure according to Tendulkar formula.
3.The energy requirement as calculated by Rangarajan is signicantly higher than that used earlier by Lakadwala panel.
Codes:

(A) 1 and 2
(B) 1 and 3
(C) 2 and 3
(D) 1, 2 and 3
Selected answer: (Not selected..)
The correct answer is:(A)
Solution:The energy requirement as calculated by Rangarajan is 2,155 kcal per person per day in rural areas and 2,090 kcal per person per day in urban areas. This is signicantly
lower than the 2,400 kcal in rural areas and slightly less than 2,100 kcal in urban areas used by the earlier Lakdawala panel. The reason given is that the age prole and
working conditions have changed with time.

Information for the solved exam

Started in: March 27, 2017, 12:46 am

Finished in: March 27, 2017, 12:46 am

Marks Obtained: 0

Right Ques.: 0

Wrong Ques.: 0

Unattempted Ques.: 25

Try for a better result? (index.php?section=user&page=continue_exam&id=133428&renew_exam=yes)

Rank List (index.php?section=user&page=ranking&id=95)


Socio - Economic Development Test - 5

1. Which of the following are the objectives of National Agriculture Insurance Scheme?
1. To provide insurance coverage and nancial support to the farmers in the event of failure of any of the notied crop as a result of natural calamities, pest and
diseases.
2. To help stabilize farm incomes, particularly in disaster years.
3. To help stabilize farm incomes, irrespective of disaster or normal years.
Codes:

(A) 1 and 3
(B) 1 and 2
(C) 2 and 3
(D) All
Selected answer: (Not selected..)
The correct answer is:(B)
Solution:.

2. Consider the following statements with respect to National Rural Livelihood Mission (NRLM):
1.NRLM is remodeled Swaranjayanti Grameen Swarojgar Yojna (SGSY) focusing on promoting self-employment and organization of rural poor with the support of IMF.
2.Under NRLM, it is mandatory for an individual to be a part of Self Help Group (SHG) for access to various services under the scheme.
Which of the above statements is/are correct?

(A) Only 1
(B) Only 2
(C) Both
(D) None
Selected answer: (Not selected..)
The correct answer is:(B)
Solution:NRLM is supported by World Bank not IMF.

3. What are the proposed amendments in Child Labor (Prohibition and Regulation) Act, 1986?
1.It bans employment of children below 14 years in any occupation, bringing the law in consistency with the Right to Children to Free and Compulsory Education Act
2009.
2.It prohibits employment of under-14 children in family enterprises only.
3.It prohibits employment of children between 14 to 18 years in hazardous occupations unsuitable to their age.
Codes:

(A) 1 and 2
(B) 1 and 3
(C) 1, 2 and 3
(D) 2 and 3
Selected answer: (Not selected..)
The correct answer is:(B)
Solution:Proposes to ban the employment of children below 14 years in all occupations except family enterprises and the audio-visual entertainment industry, on condition that such
work does not interfere with their education. The amendment proposes to regulate "adolescents" in the 14-18 age group by prohibiting their employment in hazardous
occupations unsuitable to their age.

4. Consider the following statements about the revamped Rashtriya Swasthya Bima Yojana:
1.Revamped RSBY will be responsible for providing secondary health care benets.
2.Rashtriya Swasthya Bima Yojana (RSBY) is set to be transferred to the health ministry as part of the proposed National Health Assurance Mission (NHAM).
3.The beneciary has to pay Rs. 1000 per annum as registration/renewal fee to get biometric-enabled smart card for identifying their eligibility.
Which of the above statements is/are true?

(A) 1 and 2
(B) 1 and 3
(C) Only 2
(D) 2 and 3
Selected answer: (Not selected..)
The correct answer is:(A)
Solution:The beneciary has to pay Rs. 30 per annum as registration/renewal fee to get biometric-enabled smart card for identifying their eligibility.

5. Consider the following statements with respect to AIIB:


1.The purpose of the multilateral development bank is to provide nance to infrastructure projects in the entire world for developing nations.
2.All the prospective founding members of AIIB belong to Asia and India is one of them.
3.The AIIB is headquartered at Beijing, China.
Which of the above statements is/are correct:

(A) 1 and 3
(B) 2 and 3
(C) All
(D) Only 3
Selected answer: (Not selected..)
The correct answer is:(D)
Solution:The purpose of the multilateral development bank is to provide nance to infrastructure projects in the Asian region. All the prospective founding members of AIIB do not
belong to Asia and India is one prospective founding member.

6. Consider the following statements about recently induced labour reforms:


1.Shram Suvidha Unied Portal for Labour and Employment Portal will facilitate consolidated information of Labour Inspection and its enforcement.
2.The Factories bill 2014 identies factory as an entity comprising of 30 workers if power used and 60 if not.
Which of the above statements is/are true?

(A) Only 1
(B) Only 2
(C) Both
(D) None
Selected answer: (Not selected..)
The correct answer is:(A)
Solution:The Factories bill 2014 identies factory as an entity comprising of 20 workers if power used and 40 if not.
Solution:

7. Which of the following statements are correct about National Investment and Infrastructure Fund (NIIF)?
1. It can raise debt, and in turn, invest as equity, in infrastructure nance companies such as IRFC and NHB.
2.Initial capital amounting to Rs. 20,000 crore is planned to be incorporated into the fund.
3.The fund will leverage the corpus to raise low cost-debt through bond issues for investment.
Codes:

(A) 1 and 2
(B) 2 and 3
(C) 1 and 3
(D) All
Selected answer: (Not selected..)
The correct answer is:(D)
Solution:.

8. Which of the following is correct about Pradhan Manti Krishi Sinchai Yojana?
1. It provide assured irrigation to mitigate risk to the farmer since bulk of the farmlands are rainfed and depend on monsoon.
2. At macro level, comprehensive mapping & identication of water bodies will be taken up.
3. Under this rivers will be inter-linked to avoid situations of drought and oods.
Codes:

(A) 1 and 3
(B) Only 2
(C) 2 and 3
(D) All
Selected answer: (Not selected..)
The correct answer is:(D)
Solution:.

9. Which of the following is correct about Atal Innovation Mission (AIM) proposed in the budget?
1. It will provide an Innovation Promotion Platform involving academicians, and drawing upon national and international experiences.
2. It will foster a culture of innovation, research and development in the country.
3. It will be established in Ministry of Finance.
Select the correct code:

(A) 1 and 2
(B) Only 2
(C) 1 and 3
(D) All
Selected answer: (Not selected..)
The correct answer is:(A)
Solution:It will be established in NITI Aayog.

10. Which of the following are correct about the Atal Mission for Rejuvenation and Urban Transformation (AMRUT)?
1.It adopts a project approach to ensure basic infrastructure services relating to water supply, sewerage in cities.
2.Implementation of the Mission will be linked to promotion of urban reforms such as e-governance.
3. This Mission will be implemented in 500 cities and towns each with a population of one lakh and above.
Select the correct code:

(A) 1 and 2
(B) 1 and 3
(C) 2 and 3
(D) All
Selected answer: (Not selected..)

The correct answer is:(D)


Solution:.

11. The thrust areas to be addressed under National Mission for Sustainable Agriculture (NMSA) are:
1. Dryland agriculture
2. Access to information
3. Bio-technology
4. Risk management
Which of the above stated areas is/are correct?

(A) 1, 2 and 3
(B) 1, 2 and 4
(C) 1 and 4
(D) All
Selected answer: (Not selected..)
The correct answer is:(D)
Solution:The National Mission for Sustainable Agriculture (NMSA) is envisaged as one of the eight Missions under the National Action Plan on Climate Change (NAPCC) with the
objective of promoting Sustainable Agriculture. The thrust areas to be addressed under this Mission are dryland agriculture, access to information, bio-technology and risk
management.

12. Consider the following statements regarding ASPIRE: A Scheme for Promoting Innovation and Rural Entrepreneurship:
1.The scheme was formulated to set up a network of technology centres and incubation centers to accelerate entrepreneurship.
2.It aims to promote start-ups for innovation and entrepreneurship in agro-industry.
3.The most important component of scheme is to set up Livelihood Business Incubators (LBI) under National Small Industries Corporation (NSIC).
Which of the above statements is/are correct?

(A) 1 and 2
(B) 2 and 3
(C) 1 and 3
(D) All
Selected answer: (Not selected..)
The correct answer is:(D)
Solution:.
13. Consider the following statements:
1. The First Five year Plan laid down the basic ideas regarding goals of Indian planning.
2. The essence of the FeldmanMahalanobis model adopted during the Second Plan is a shift in the pattern of industrial investment towards building up a domestic
consumption goods sector.
3. The First Five Year Plan stressed investment for capital accumulation in the spirit of the one-sector.
4. The Harrod Domar model suggests that the economic rate of growth depends on the level of savings, and the productivity of investment.
Which of the statements given above are correct?

(A) 1, 2 and 3
(B) 1 and 2
(C) 2, 3 and 4
(D) All
Selected answer: (Not selected..)
The correct answer is:(C)
Solution:India introduced a formal plan model after the First Five Year Plan (1951-1956).

14. Tarapore committee in 2006 was associated with which of the following?

(A) Partial Capital Account Convertibility


(B) Full Capital Account Convertibility
(C) Partial Current Account Convertibility
(D) Full Current Account Convertibility
Selected answer: (Not selected..)
The correct answer is:(B)
Solution:.

15. Which of the following organizations/agencies has agreed to set up an institute in India to train manpower in commodities?

(A) Chicago Board Options Exchange


(B) London Stock Exchange
(C) Tokyo Stock Exchange
(D) New York Stock Exchange
Selected answer: (Not selected..)
The correct answer is:(D)
Solution:.

16. Which of the following are the components of Deendayal Upadhyaya Gram Jyoti Yojana (DDUGJY)?
1. It aims of strengthening and augmentation of sub-transmission and distribution infrastructure in rural areas.
2. Metering in rural areas.
3. Combining agriculture and non-agriculture feeders
Codes:

(A) 1 and 2
(B) 2 and 3
(C) 1 and 3
(D) All
Selected answer: (Not selected..)
The correct answer is:(A)
Solution:Separating agriculture and non-agriculture feeders to facilitate distribution companies (discoms) in the judicious rostering of supply to agricultural and non-agricultural
consumers.

17. Which of the following are the features of Deen Dayal Upadhyaya Grameen Kaushalya Yojana?
1. It provides for demand led skill training at no cost to the rural poor.
2. It provides for mandatory coverage of socially disadvantaged groups for training.
3. It provides funding support for placement linked skilling projects that address the market demand.
Codes:

(A) 1 and 2
(B) 2 and 3
(C) 1 and 3
(D) All
Selected answer: (Not selected..)
The correct answer is:(D)
Solution:.

18. Consider the following statements about Integrated Programme for Older Persons revised in April 2015:
1. The main objective of the Scheme is to improve the quality of life of the Older Persons by providing basic amenities like shelter, food, medical care.
2. Assistance under the scheme will be given to the Panchayati Raj Institutions / local bodies and NGOs are excluded from it.
3. It provides for Institutional as well as Non Institutional Care / Services to Older Persons.
Which of the statements given above are correct?

(A) 1 and 2
(B) 2 and 3
(C) 1 and 3
(D) All
Selected answer: (Not selected..)
The correct answer is:(C)
Solution:Assistance under the scheme will be given to the Panchayati Raj Institutions / local bodies and eligible Non-Governmental Voluntary Organizations.

19. Consider the following statements about the Eklavya Model Residential Schools (EMRS) for ST students:
1. The objective of EMRS is to provide quality education only upto the middle level to the Scheduled Tribe (ST) students in remote areas.
2. Comprehensive physical, mental and socially relevant development of all students is the major objective.
3. Admission to these schools is based on the BPL status of the family.
Which of the statements given above is/are correct?

(A) Only 1
(B) Only 2
(C) Only 3
(D) All
Selected answer: (Not selected..)
The correct answer is:(B)
Solution:The objective of EMRS is to provide quality middle and high level education to Scheduled Tribe (ST) students in remote areas, not only to enable them to avail of reservation
in high and professional educational courses and as jobs in government and public and private sectors but also to have access to the best opportunities in education at par
with the non ST population. Admission to these schools will be through selection/competition with suitable provision for preference to children belonging to Primitive Tribal
Groups, rst generation students, etc.

20. Consider the following statements about the Integrated Power Development Scheme" (IPDS):
1. It focuses on strengthening of sub-transmission and distribution network in the urban areas.
2. It will help in reduction in AT&C losses.
3. It will help in establishment of IT enabled energy accounting / auditing system.
Which of the statements given above are correct?

(A) 1 and 2
(B) 2 and 3
(C) 1 and 3
(D) All
Selected answer: (Not selected..)
The correct answer is:(D)
Solution:.

21. Which of the following statements is/are correct about Indradhanush Mission?
1.It is to achieve full immunization coverage for all children by 2025.
2.This mission will cover all un-vaccinated or partially vaccinated children in India against seven vaccine preventable diseases.
3.The vaccine will be covered against diphtheria, whooping cough, tetanus, polio, pneumonia, measles and hepatitis B diseases.
Codes:

(A) 1 and 2
(B) 2 and 3
(C) Only 2
(D) All
Selected answer: (Not selected..)
The correct answer is:(C)
Solution:\ is to achieve full immunization coverage for all children by 2020
Diseases including diphtheria, whooping cough, tetanus, polio, tuberculosis, measles and hepatitis B.

22. Which of the following statements is/are correct about Indradhanush Mission?
1.It is to achieve full immunization coverage for all children by 2025.
2.This mission will cover all un-vaccinated or partially vaccinated children in India against seven vaccine preventable diseases.
3.The vaccine will be covered against diphtheria, whooping cough, tetanus, polio, pneumonia, measles and hepatitis B diseases.
Codes:

(A) 1 and 2
(B) 2 and 3
(C) Only 2
(D) All
Selected answer: (Not selected..)
The correct answer is:(C)
Solution:It is to achieve full immunization coverage for all children by 2020
Diseases including diphtheria, whooping cough, tetanus, polio, tuberculosis, measles and hepatitis B.

23. Which of the following statements is/are correct about Rashtriya Arogya Nidhi?
1.It aims to provide for nancial assistance to patients living below poverty line.
2.As per the scheme, person who are suering from major life threatening diseases is elligible to receive medical treatment at any of the super specialty
hospitals/institutes or other Gover-ment hospitals.
3.Financial assistance is released in the form of two times grant to the Medical Superintendent of the hospital in which the treatment is being received.
Codes:

(A) Only 1
(B) 1 and 3
(C) 1 and 2
(D) All
Selected answer: (Not selected..)
The correct answer is:(C)
Solution:Financial assistance is released in the form of "one times grant" to the Medical Superintendent of the hospital in which the treatment is being received.

24. Which of the following ministries do not work as a multisectoral intervention under Beti Bachao Beti Padhao Yojna ?
1.Ministry of Women and Child Welfare.
2.Ministry of Health and Family Welfare.
3.Ministry of Human Resource Development.
4.Ministry of Social .Justice and Empowerment.
Codes:

(A) Only 3
(B) 3 and 4
(C) Only 4
(D) All
Selected answer: (Not selected..)
The correct answer is:(C)
Solution:.

25. Consider the following statements:


1.One stop centre are being established across the country to provide integrated support and assistance under one roof to women aected by violence, both in private
and public spaces.
2.One stop centre are being established across the country to provide integrated support and assistance under one roof to women aected by violence in public spaces.
3.One stop centre is to provide free access to medical and psychological support only.
4.One stop centre is to provide medical, psycholgical and legal support.
Which of the above statements are correct?

(A) 1 and 3
(B) 1 and 4
(C) 2 and 3
(D) 2 and 4
Selected answer: (Not selected..)

The correct answer is:(B)


Solution:.

Information for the solved exam

Started in: March 27, 2017, 12:44 am

Finished in: March 27, 2017, 12:44 am

Marks Obtained: 0

Right Ques.: 0

Wrong Ques.: 0

Unattempted Ques.: 25

Try for a better result? (index.php?section=user&page=continue_exam&id=133421&renew_exam=yes)

Rank List (index.php?section=user&page=ranking&id=96)


Socio - Economic Development Test - 6

1. Which of the following is /are correct objective of SABLA scheme?


1.Promote awareness about health, hygiene, nutrition, adolescent reproductive and sexual health (ARSH) and family and child care.
2.Upgrade home-based skills, life skills and integrate with the National Skill Development Program (NSDP) for vocational skills.
3.Vocational Training for girls aged 18 and above under NSDP.
Codes:

(A) 1 and 2
(B) Only 1
(C) Only 3
(D) All
Selected answer: (Not selected..)
The correct answer is:(A)
Solution:Vocational Trainings for girls aged 16 and above under NSDP.

2. Which of the following is not included in package of services under Integrated Child Development Mission?

(A) Pre School Education


(B) Supplementary Nutrition and health education.
(C) Referral services.
(D) Primary school education.
Selected answer: (Not selected..)
The correct answer is:(D)
Solution:.

3. What were the most appropriate fundamental problems with second plan out of the following options?
1.It ignored agricultural sector.
2.It was inationary in nature.
3.It couldnt improve export competitiveness.
4.It focussed on productivity gains but not on production.
Codes:

(A) 1 and 2
(B) 2, 3 and 4
(C) 1, 2 and 4
(D) All
Selected answer: (Not selected..)
The correct answer is:(C)
Solution:.

4. Which of the following committee suggested regarding improving the condition of small and medium industries?

(A) Omkar Goswami Committee


(B) Kelkar Committee
(C) Deepak Pareekh Committee
(D) Abid Hussain Committee
Selected answer: (Not selected..)
The correct answer is:(D)
Solution:.

5. Integrated Child Development Services (ICDS) Scheme aims to providing many services to children that includes:
1.Basic Education
2.Supplementary Nutrition
3.Primary Healthcare facilities
Select the correct answer using the codes given below:
Codes:

(A) 1 and 2
(B) 1 and 3
(C) 2 and 3
(D) 1, 2 and 3
Selected answer: (Not selected..)
The correct answer is:(C)
Solution:.

6. Which of the following area is not covered under recently launched Beti Bachao Beti Padhao Yojana?

(A) Safety
(B) Education
(C) Female infanticide
(D) Vocational Training
Selected answer: (Not selected..)
The correct answer is:(D)
Solution:.

7. Shanta Kumar committee has submitted its report on restructuring of Food Corporation of India (FCI) to the central government. Which of the following
recommendations of the committee are correct?
1.FCI procurement should focus on the Western belt, where farmers get the minimum support price.
2.There should be greater exibility to FCI with business orientation, to operate in OMSS and export markets and a transparent liquidation policy to manage surplus buer
stocks.
3.Proposed uniform tax of minimum 3% and maximum 4% on wheat and rice and the same to be included in the MSP.
Codes:

(A) 1 and 2
(B) 1 and 3
(C) 2 and 3
(D) All
Selected answer: (Not selected..)
The correct answer is:(C)
Solution:FCI procurement should focus on eastern belt, where farmers could not get minimum support price of their crops.

8. Consider the following statements related to RBI's guidelines on Payment Banks:


1.Mobile telephone companies, super-market chains, companies, real sector cooperatives; that are owned and controlled by residents and public sector entities may
apply to set up payments banks.
2.The minimum paid-up equity capital for Payment banks shall be Rs. 100 crores.
3.Payment Banks are eligible for issuing ATM/debit cards/credit cards as other banks.
Which of the above statements is/are correct?

(A) 1 and 2
(B) 2 and 3
(C) 1 and 3
(D) All
Selected answer: (Not selected..)
The correct answer is:(A)
Solution:Payment Banks are eligible for issuing ATM/debit cards but not credit cards.

9. Which of the following is/are included in the major policy goals of the National Policy for Farmers?
1.To mainstream the human and gender dimension in farm policies and programmes.
2.To develop and introduce a social security system for farmers.
3.To complete the unnished agenda in land reforms.
4.To introduce measures which can help attract and retain youths in farming.
Codes:

(A) 1 and 4
(B) 2 and 3
(C) Only 2
(D) All
Selected answer: (Not selected..)
The correct answer is:(D)
Solution:.

10. The oil price has fallen by more than 40% since June 2014 and the trend continues. Which of the following statements correctly explain the reason for the present
decline in oil prices?
1.The global economic slowdown, as China's national growth projected at around 7% per year and Europe is also suering from sluggish growth.
2.The world markets are ushed with supplies for which there are not enough consumers.
3.Shale gas revolution in USA has provided an alternative fuel options thus reducing oil imports.
Codes:

(A) Only 1
(B) Only 2
(C) 1 and 3
(D) All
Selected answer: (Not selected..)
The correct answer is:(D)
Solution:.

11. Consider the following statements about the New Urea Policy for 2015-19 approved by Union Cabinet:
1. Urea producers are mandated to produce a minimum of 75% of domestic urea as neem coated.
2. Movement of P&K fertilizers has been freed, which will allow companies to sell these fertilizers in any part of the country.
3. Subsidies on fertilizers will be payable to suppliers only after the receipt of the fertilizers.
Which of the statements given above are correct?

(A) 1 and 2
(B) 1 and 3
(C) 2 and 3
(D) All
Selected answer: (Not selected..)
The correct answer is:(D)
Solution:.

12. Concept of income elasticity of demand is useful in analysis of which of the following economic phenomenon?
1.To analyze the impact of subsidy provided.
2.To analyze the impact of tax imposed.
3.To calculate the basket of essential commodities.
Codes:

(A) Only 1
(B) 1 and 2
(C) 2 and 3
(D) All
Selected answer: (Not selected..)
The correct answer is:(D)
Solution:If income elasticity is high it would mean that it will help non-targeted population instead of targeted population.
If income elasticity is low for a commodity; the tax imposed on it would be regressive.
Essential commodities generally have low income elasticity and high price elasticity.

13. The central government has relaunched the Kisan Vikas Patra (KVP) investment scheme to tap household savings for funding infrastructure development in the
country. Consider the following statements related to it:
1.The minimum amount that can be invested is Rs 1000 with no maximum limit.
2.Kisan Vikas Patra oer income tax benets on interest earned by the investors.
3.Under this, the amount invested get double in ve year span.
Which of the above stated features is/are correct?

(A) Only 1
(B) Only 2
(C) 1 and 3
(D) All
Selected answer: (Not selected..)
The correct answer is:(A)
Solution:Kisan Vikas Patra does not oer income tax benets to the investors. Under this, the amount invested get doubles in 100 months.

14. Which of the following statements is/are false about agricultural credit puzzle in India?
1.Agricultural credit is growing at the same rate as the overall credit creation in India.
2.Most of the Agricultural credit is issued from urban and metropolitan area bank branches.
3.Increase in Agricultural credit is not able to increase the asset creation in the primary sector.
Codes:

(A) Only 1
(B) 1 and 2
(C) 1 and 3
(D) All
Selected answer: (Not selected..)
The correct answer is:(D)
Solution:.

15. What are Free Goods?

(A) Goods which do have rivalry and excludability.


(B) Goods with no opportunity cost.
(C) Goods provided by the state for free for greater social benet.
(D) Goods where people may under-estimate costs of consuming it.
Selected answer: (Not selected..)
The correct answer is:(B)
Solution:Free goods are goods with no opportunity cost like Breathing Air.

16. Which of the following statements related to Ease of Doing Business Index is/are incorrect?
1.A low ease of doing business ranking means the regulatory environment is more conducive to the starting and operation of a local rm.
2.It calculates index on the basis of strict employment regulations so that human rights issues can be maintained.
3.The index is prepared by IMF.
Codes:

(A) Only 3
(B) 2 and 3
(C) 1 and 3
(D) All
Selected answer: (Not selected..)
The correct answer is:(B)
Solution:It favors exible employment regulations. The index is prepared by World Bank.

17. The Central and State governments over a period of time, evolved strategies to reduce drop-out rates and improve levels of achievements in schools. Which of the
following steps has been initiated by them in this direction?
1.Creating parental awareness and community mobilization.
2.Involvement of Panchayati Raj Institutions in implementation of schemes.
3.Economic incentives such as free education, free books and free uniforms.
4.Amending Constitution to bring education at forefront.
Codes:

(A) 2 and 3
(B) 1, 3 and 4
(C) 2, 3 and 4
(D) All
Selected answer: (Not selected..)
The correct answer is:(D)
Solution:.

18. Which of the following expert committees is not related to the nancial sector?

(A) Arvind Mayaram Committee


(B) Nachiket Mor Committee
(C) Pulin J. Nayak Committee
(D) Urjit Patel Committee
Selected answer: (Not selected..)
The correct answer is:(A)
Solution:.

19. Which of the following tax is not shared by the Centre and the States?

(A) Sales Tax


(B) Corporation Tax
(C) Income Tax
(D) Union Excise Duties
Selected answer: (Not selected..)
The correct answer is:(A)
Solution:.

20. Consider the following statements with reference to the chronology of events in foreign exchange system in India:
1.Post independence, Indias exchange rate policy was of par value system which was xed with respect to Gold.
2.With the breakdown of the Bretton Woods System in 1971, the Rupee was pegged to a basket of currencies.
3.In the Post Structural Reforms of 1991, the Liberalized Exchange Rate Management System (LERMS) were put in place.
Which of the above given stated events is/are incorrect?

(A) 1, 2 and 3
(B) 2 and 3
(C) 1 and 3
(D) Only 2
Selected answer: (Not selected..)
The correct answer is:(D)
Solution:India's exchange rate policy has evolved over time in line with the gradual opening up of the economy as part of the broader strategy of macroeconomic reforms and
liberalization since the early 1990s. In the post independence period, India's exchange rate policy has seen a shift from a par value system to a basket-peg and further to a
managed oat exchange rate system. With the breakdown of the Bretton Woods System in 1971, the rupee was linked with pound sterling. In order to overcome the
weaknesses associated with a single currency peg and to ensure stability of the exchange rate, the rupee, with eect from September 1975, was pegged to a basket of
currencies till the early 1990s.

21. Recently the Ministry of Urban Development has ranked 75 cities under the mission Swachh Survekshan. Which of the following parameters are dened under the
mission?
1.Information, Education and Behaviour Change Communication (IEBC) activity.
2.Distribution and establishment of dustbins across the city.
3.Construction of household individual toilets.
4.Sweeping, door to door collection and transportation (of solid waste).
Codes:

(A) 2, 3 and 4
(B) 1, 2 and 4
(C) 1, 3 and 4
(D) All
Selected answer: (Not selected..)
The correct answer is:(C)
Solution:Parameters for evaluation:
Strategy for Open Defecation Free town (ODF) and Integrated Solid Waste Management (SWM).
Information, Education and Behaviour Change Communication (IEBC) activity.
Sweeping, door to door collection and transportation (of solid waste).
Processing and disposal of solid waste.
Provision of public & community toilet seats.
Construction of household individual toilets.

22. With respect to Unnat Jyoti by Aordable LEDs for all schemes, which of the following statements is/are correct?
1.The scheme aims to improve energy eciency and reduce power consumption of the states.
2.The scheme aimed at greenhouse gas emission reductions.
3.The scheme is implemented by Energy Eciency Services Limited (EESL).
Codes:

(A) 1 and 2
(B) 1 and 3
(C) Only 3
(D) All
Selected answer: (Not selected..)
The correct answer is:(D)
Solution:The Union Government has launched National LED programme -Unnat Jyoti by Aordable LEDs for All (UJALA).
The UJALA scheme is being implemented by Energy Eciency Services Limited (EESL), a joint venture of PSUs under the Union Ministry of Power.
The scheme will help reduce electricity bills of consumers, contribute to the energy security of India and also help in environment protection.
Benets of UJALA Scheme:
Electricity savings
Reduction of load
Reduction of consumer bills
Greenhouse gas emission reductions

23. Which of the following statements related to Swachh Bharat Cess is/are incorrect?
1.Rate of Swachh Bharat cess is 0.5 % on all services.
2.Swachh Bharat cess is levied on service tax.
3.The amount will be used in nancing and promoting toilets in rural areas only to reduce open defecation.
Select the correct answer using below codes:
Codes:

(A) Only 1
(B) 1 and 2
(C) 1, 2 and 3
(D) None
Selected answer: (Not selected..)
The correct answer is:(C)
Solution:It is correct that cess is levied on tax. And it is tax on tax. But Swachh Bharat Cess is somewhat dierent. It is levied on 'Value of Services', not on service tax. So the eective
rate of service tax is 14.05 %( including 0.5 % Swachh Bharat Cess). Swachh Bharat Cess is not leviable on services which are fully exempt from service tax or those covered
under the negative list of services.

24. Which of the following committee is constituted to review and recommend on FRBM act?

(A) N.K. Singh Committee


(B) Arvind Subramaniam Committee
(C) Urjit Patel Committee
(D) Sumit Bose Committee
Selected answer: (Not selected..)
The correct answer is:(A)
Solution:The Union Government has constituted a ve member committee to comprehensively review and give recommendations on the Fiscal Responsibility and Budget
Management (FRBM) roadmap for the future.
The committee will be headed by Former Revenue Secretary and RajyaSabha MP, NK Singh. Its members are former Finance and Revenue Secretary Sumit Bose, Chief
Economic Adviser (CEC) Dr. Arvind Subramanian, Deputy Governor of RBI Urijit Patel and Director of National Institute Public Finance and Policy (NIPFP) Rathin Roy.

25. Consider the following statements about SARFAESI Act:


1.Under this act, Bank has to power to take possession of Defaulter assets without court order.
2.If the price fetched in auction of property exceeds the banks dues, the excess amount is given to the borrower.
3.SARFAESI Act is eective only for secured loans where bank can enforce the underlying security.
Which of the above statements is/are correct?

(A) 1 and 2
(B) Only 2
(C) Only 3
(D) All
Selected answer: (Not selected..)
The correct answer is:(D)
Solution:The Securitisation and Reconstruction of Financial Assets and Enforcement of Security Interest Act, 2002, allows banks and nancial institutions to auction
properties(residential and commercial) when borrowers fail to repay their loans. It enables banks to reduce their non-performing assets (NPAs) by adopting measures for
recovery or reconstruction.
Bank can take possession of Defaulter assets without requiring court order both Commercial or residential, xed or moving assets.
SARFAESI applies only to loans above Rs.10 lakhs.
If the price fetched exceeds the bank's dues, the excess amount is given to the borrower.

Information for the solved exam

Started in: March 27, 2017, 12:40 am

Finished in: March 27, 2017, 12:40 am

Marks Obtained: 0

Right Ques.: 0

Wrong Ques.: 0

Unattempted Ques.: 25

Try for a better result? (index.php?section=user&page=continue_exam&id=133411&renew_exam=yes)

Rank List (index.php?section=user&page=ranking&id=100)


Socio - Economic Development Test - 7

1. Which agency is mandated to conducts investigations for possible contravention of FDI norms/policy?

(A) Enforcement Directorate.


(B) Cabinet Committee on Economic Aairs.
(C) Reserve Bank of India.
(D) Director General for Foreign Trade.
Selected answer: (Not selected..)
The correct answer is:(A)
Solution:Enforcement Directorate conducts investigations under the Foreign Exchange Management Act, 1999 (FEMA) for possible contravention of FDI norms/ policy by various
entities as and when any credible information is received in this regard. Based on the outcome of the investigation, appropriate action under the provision of FEMA is taken
in such cases.

2. Which of the following statement with regard to Sansad Adarsh Gram Yojna is incorrect?

(A) Members of Parliament will adopt village to create model village.


(B) A Rajya Sabah MP has to choose a gram panchayat in the state from which he or she was elected.
(C) District Collector will act as coordinator.
(D) Separate fund have been allotted for the scheme to improve the delivery process.
Selected answer: (Not selected..)
The correct answer is:(D)
Solution:No new funds have been allocated for the Yojana. Resources may be raised through:
Funds from existing schemes, such as the Indira Awas Yojana, Pradhan Mantri Gram Sadak Yojana, Mahatma Gandhi National Rural Employment Guarantee Scheme, and
Backward Regions Grant Fund, etc.
The Member of Parliament Local Area Development Scheme (MPLADS),
The gram panchayat's own revenue.
Central and State Finance Commission Grants, and
Corporate Social Responsibility funds.

3. Which of the following ministries deal with Urban Local Government?


1.Ministry of Urban development.
2.Ministry of Housing and Urban poverty alleviation.
3.Ministry of Defence.
4.Ministry of Home Aairs.
Codes:

(A) 1 and 2
(B) 2, 3 and 4
(C) 1, 3 and 4
(D) All
Selected answer: (Not selected..)
The correct answer is:(C)
Solution:Ministry of Defence (In case of Cantonment boards) , Ministry of Home Aairs (UT), Ministry of Urban Development in case of all other towns.

4. Consider the following statements related to organisation of Panchayats and select the correct answer:
1.A Gram Sabha consists of all the adults i.e. voters living in the area of a Gram Panchayat.
2.Gram Sabha is executive committee but not a legal body.
3.Meetings of Gram Sabha are held in every two months.
Which of the above statements is/are correct?

(A) Only 1
(B) 1 and 2
(C) 2 and 3
(D) All
Selected answer: (Not selected..)
The correct answer is:(A)
Solution:The Village Panchayat or Gram Panchayat is the executive committee of Gram Sabha. A Gram Sabha or Village Assembly consists of all the adults i.e. voters (persons above
the age of 18 years) living in the area of a Gram Panchayat i.e., village or a group of small villages. The Gram Sabha has now been recognized as a legal body.

5. National Land Record Modernisation Programme has been revamped in Budget 2016-17. Which of the following are the benets of NLRMP?
1.Real-time land ownership records will be available to the citizen.
2.Free accessibility to the records will reduce interface between the citizen and the Government functionaries, thereby reducing rent seeking.
3.Automatic and automated mutations will signicantly reduce the scope of fraudulent property deals.
Codes:

(A) 1 and 2
(B) 2 and 3
(C) 1 and 3

(D) All
Selected answer: (Not selected..)
The correct answer is:(D)
Solution:Benets of NLRMP Project are:
A modern, comprehensive and transparent land records management system in each state.
A single window system to handle land records, including maintenance and updating of textual records, maps, survey and settlement operation and registration of
immovable property.
Up-dated land records and push them into public domain so that people can access the records with ease.
Integration of the diverse processes in land administration and provide an integrated land records information system.
Land value assessment.
Preparation of eld level soil health cards.
Smart cards for farmers to facilitate e-governance and e-banking.
Settlement of compensation claims
Land acquisition and rehabilitation
Crop insurance
Grant of agricultural subsidies
Community/ village resource centres
Precision farming etc.
6. Consider the following statements related to the Krishi Kalyan Cess:
1.It includes the cess of 0.5% on all taxable services.
2.Proceeds of cess will be exclusively used by NABARD for irrigation development.
Which of the above statements is/are correct?

(A) Only 1
(B) Only 2
(C) Both
(D) None
Selected answer: (Not selected..)
The correct answer is:(A)
Solution:The proceeds of tax will be exclusively used for nancing initiatives relating to improvement of agriculture and welfare of farmers.

7. The National Social Assistance Programme (NSAP) aims at ensuring minimum national standard for social assistance. What assistances are available under this
programme?
1. Old Age Pension
2. Widow Pension
3. Lump sum cash assistance on death of the primary bread winner of family
4. Subsidized foodgrain
5. Disability Pension
Select the correct answer using the codes given below:
Codes:

(A) 1, 2 and 5
(B) 1, 2, 4 and 5
(C) 1, 2, 3 and 5
(D) All
Selected answer: (Not selected..)
The correct answer is:(D)
Solution:At present NSAP comprises NOAPS, NWPS, NDPS, NFBS& Annapurna. The NFBS provides a lump sum family benet of Rs. 20000 to the bereaved household in case of death
of the primary bread winner irrespective of the cause of death. This scheme is applicable to all the eligible persons in the age group of 18-64.
Annapurna Scheme aimed at providing food security to meet the requirement of those senior citizens who, though eligible had remained uncovered under the NOAPS. This
Scheme 10 kgs of food grains per month is provided free of cost to the beneciary.

8. Consider the following statements with reference to Pradhan Mantri Gram Sadak Yojana (PMGSY):
1.The primary focus of the Programme is on construction only of weathered roads to the unconnected villages.
2.This programme will be implemented by Ministry of Roads Transport and Highway.
Which of the above statements is/are correct?

(A) Only 1
(B) Only 2
(C) Both
(D) None
Selected answer: (Not selected..)
The correct answer is:(D)
Solution:This programme include construction as well as maintenance of existing roads in rural regions.
The Rural Roads constructed /upgraded under this Programme will be maintained by the concerned Panchayati Raj Institution (District Panchayat/ Intermediate Panchayat).
The concerned Panchayati Raj Institutions would need to be identied while submitting the project for approval and the State Authorities will be required to furnish an
Undertaking that they would remit (to the identied Panchayati Raj Institution), from the State Government funds, the requisite cost of maintenance. The State Governments
will also oer an Undertaking for the release of maintenance costs, alongwith their project proposals. The Ministry of Rural Development would oversee the implementation
of this undertaking.

9. The responsibility to plan, monitor and coordinate 'Sustainable Development Goals' (SDG) eorts across Central Ministries and State governments in India is entrusted
upon:

(A) Prime Minister's Oce.


(B) Ministry of Planning.
(C) Ministry of Statistics and Programme Implementation.
(D) NITI Aayog
Selected answer: (Not selected..)
The correct answer is:(D)
Solution:NITI Aayog has been entrusted with the responsibility to plan, monitor and coordinate SDG eorts across Central Ministries and State governments.

10. 'Jan Swasthya Abhiyan' often seen in the news is:

(A) A phrase axed to the on-going health movement in India, spearheaded by NGOs which seek to address the problem of rising disparities in
health status among people in India.
(B) A government scheme which seeks to boost rural public health infra-structure.
(C) The Indian regional circle of the global People's Health Movement (PHM)
(D) A government scheme which seeks to boost both urban & rural public health infrastructure.
Selected answer: (Not selected..)
The correct answer is:(C)
Solution:The Jan Swasthya Abhiyan (JSA) was formed in 2001, with the coming together of 18 national networks that had organised activities across the country in 2000, in the lead up
to the First Global Peoples Health Assembly, in Dhaka, in December 2000. The JSA forms the Indian regional circle of the global People's Health Movement (PHM).

11. Gender Budgeting is a powerful tool for achieving gender mainstreaming so as to ensure that benets of development reach women. Gender budgeting helps in:
1.Governments to decide how policies need to be made, adjusted and reprioritized for women and children.
2.Tracking public expenditure against gender and development policy commitments.
3.Helps in checking both nancially and the physical deliverables of the programmes and policies.
Which of the above stated benets is/are correct?

(A) 1 and 2
(B) 2 and 3
(C) Only 1
(D) All
Selected answer: (Not selected..)
The correct answer is:(D)
Solution:.
12. Consider the following statements regarding Street Vendors Act, 2014:
1.The bill aimed at providing social security and livelihood rights to street vendors in both urban as well as rural areas.
2.All street vendors above eighteen years of age will be granted a certicate of vending.
3.The Ministry of Urban Development has prepared the draft of the Street Vendors Bill for livehood rights of Urban vendors.
Which of the above statements is/are correct?

(A) Only 1
(B) 1 and 3
(C) All
(D) None
Selected answer: (Not selected..)
The correct answer is:(D)
Solution:All street vendors above fourteen years of age will be granted a certicate of vending.
The bill aimed at providing social security and livelihood rights to street vendors in urban areas.
The Ministry of Housing and Urban Poverty Alleviation has prepared the draft of the Street Vendors Bill.

13. Consider the following statements regarding the Mid Day Meal Scheme:
1.Every child is eligible for 100 gms food grains at primary stage and 150 gms at upper primary stage on each working day.
2.The cost of foodgrains is shared between the Government of India and the State in the ratio of 75:25.
3.Calorie intake per child per day at primary stage is 450g and 700g at upper primary stage.
Which of the above statements is/are incorrect?

(A) Only 3
(B) 2 and 3
(C) 1 and 3
(D) Only 2
Selected answer: (Not selected..)
The correct answer is:(D)
Solution:The objective of the scheme is to provide hot cooked meal to children of primary and upper primary classes. Eligibility; Government, Govt. Aided, Local Body, EGS and AIE
Centres, Madarsa and Maqtabs supported under Sarva Shiksha Abhiyan and National Child Labour Project (NCLP) Schools run by Ministry of Labour. Every child is eligible for
100 gmsfoodgrains at primary stage and 150 gms at upper primary stage on each working day. The cost of foodgrains is fully borne by Government of India.
Caloric and nutrition value and food norm per child per day-
ITEMS PRIMARY UPPER PRIMARY
Calorie 450 700
Protein 12 20

14. Which of the following are the constituents of 15 Point Programme?


1.Enhancement of adequate credit supply to Regional Rural Banks.
2.Prevention and control of communal disharmony and violence.
3.Improving the conditions of living of minorities.
4.Connecting rural roads to the districts headquarters.
Which of the above stated constituents are correct?

(A) 1 and 2
(B) 3 and 4
(C) 2 and 4
(D) 2 and 3
Selected answer: (Not selected..)
The correct answer is:(D)
Solution:15 Point Programme was launched in 2006 for the welfare of minorities. Its objectives were to Prevention and control of communal disharmony and violence, improving the
conditions of living of minorities, to ensure government schemes benets to the disadvantaged minorities, to promote education and employment to the minorities.

15. Which of the following are the objectives of the Multi Sectoral Development Programme (MsDP)?
1.MsDP aims at to bridge the gap of inequalities between rural and urban India by providing economic facilities to vulnerables.
2.MsDP aims at to extend urban civic amenities to the rural India specially to backward classes.
3.MsDP aims at improving the socio-economic condition of women of scheduled Tribes and providing basic amenities to them.
4.MsDP aims at to improve the socio-economic condition of minorities and providing basic amenities to them.
Codes:

(A) 1 and 2
(B) 2 and 3
(C) Only 4
(D) 3 and 4
Selected answer: (Not selected..)
The correct answer is:(C)
Solution:Multi Sectoral Development Programme MsDP aims at to improve the socio-economic condition of minorities and providing basic amenities to them for improving the
quality of life of the people and reducing imbalance in the identied minority concentration areas. Education is one of the main activities which has been given priority under
the programme.

16. Consider the following statements regarding the Skill Developments initiatives in India:
1.India has various skill development programmes but it does not have any placement-linked skill development scheme for rural youth.
2.Deen Dayal Upadhyaya Grameen Kaushalya Yojana focus on the occupational needs of rural youth.
3.DDU-GKY mandates mandatory coverage of socially disadvantaged groups (SC/ST/women only).
Which of the above statements is/are correct?

(A) All
(B) None
(C) Only 2
(D) 1 and 3
Selected answer: (Not selected..)
The correct answer is:(C)
Solution:The Deen Dayal Upadhyaya Grameen Kaushalya Yojana (DDU-GKY) is a placement-linked skill development scheme for rural youth who are poor. DDU-GKY bridges this gap
by funding training projects benchmarked to global standards, with an emphasis on placement, retention, career progression and foreign placement. The Ministry of Rural
Development implements DDU-GKY to drive this national agenda for inclusive growth, by developing skills and productive capacity of the rural youth from poor families.
Deen Dayal Upadhyaya Grameen Kaushalya Yojana does focus on the occupational needs of rural youth. Deen Dayal Upadhyaya Grameen Kaushalya Yojana has a
mandatory coverage of socially disadvantaged groups (SC/ST 50%; Minority 15%; Women 33%).

17. Consider the following statements regarding the 'Hamari Dharohar initiative' which was in news recently;
1.It is a bilateral agreement between India and Japan which aims at preserving the rich heritage of Indian holy cities.
2.It aims to preserve rich heritage of both linguistic & religious minority communities.
Which of the above statements is/are correct?

(A) Only 1
(B) Only 2
(C) Both
(D) None
Selected answer: (Not selected..)
The correct answer is:(D)
Solution:Hamari Dharohar:-The Scheme aims to preserve rich heritage of minority communities under over all context of Indian culture. The Ministry of Minority Aairs in
collaboration with UNESCO ParzorFoundation, has approved three travelling iconic Exhibitions, 'The Everlasting Flame' along with academic and cultural programmes of
international magnitude.

18. Consider the following statements regarding the safeguards for the Scheduled Caste in India:
1.Constitution of India does not mention about the Scheduled Castes.
2.Ministry of Social Justice and Empowerment noties the Scheduled Castes.
3.President after consultation with the Governor of the concerned state may also notify the Scheduled Castes.
4.Parliament may by law include in or exclude the castes from the list of Scheduled Castes.
Which of the above statements are correct?

(A) 2 and 3
(B) 3 and 4
(C) 1, 2 and 4
(D) 1, 2 and 3
Selected answer: (Not selected..)
The correct answer is:(B)

Solution:Constitution of India does mention about the SCs in Article 341 of the constitution. Article 366 of the Indian Constitution go ahead and even dene the SCs.
Article 341. Scheduled Castes
1.The President may with respect to any State or Union Territory, and where it is a State, after consultation with the Governor thereof, by public notication, specify the
castes, races or tribes or parts of or groups within castes, races or tribes which shall for the purposes of this Constitution be deemed to be Scheduled Castes in relation to
that State or Union Territory, as the case may be.
2.Parliament may by law include in or exclude from the list of Scheduled Castes specied in a notication issued under clause (1) any caste, race or tribe or part of or group
within any caste, race or tribe, but save as aforesaid a notication issued under the said clause shall not be varied by any subsequent notication
Article 366. Denitions
In this Constitution, unless the context otherwise requires, the following expressions have the meanings hereby respectively assigned to them, that is to say -
(24) "Scheduled Castes" means such castes, races or tribes or parts of or groups within such castes, races or tribes as are deemed under article 341 to be Scheduled Castes
for the purposes of this Constitution;

19. Which of the following statements regarding District Planning Committee (DPC) is/are correct?
1.The provision related to its composition, functions may be made by the Governor in consultation with the President.
2.A minimum of 4/5th of the DPC shall be elected by, and from amongst, the elected members of the District Panchayat and Municipalities out of these 75% must be
municipality members.
3.It prepares a draft plan for the district as a whole.
Codes:

(A) 1 and 3
(B) Only 3
(C) 2 and 3
(D) All
Selected answer: (Not selected..)
The correct answer is:(B)
Solution:As per constitution a minimum of 2/3rd of the MPC shall be constituted of elected representatives from the metropolitan region, and a minimum of 4/5th of the DPC shall be
elected by, and from amongst, the elected members of the District Panchayat and Municipalities in the district in proportion to the ratio between the population of the rural
areas and of the urban areas in the district. Currently, DPCs are dysfunctional in most States.

20. Consider the following statements regarding the present scenario of Rural Electrication in India:
1.A village is deemed electried if it had a single light bulb connection.
2.Under this atleast ten per cent of homes to be electried including all common or public areas such as schools and clinics.
3.Deen Dayal Upadhyaya Gram Jyoti Yojana was launched to target for o-grid (decentralized) electrication in rural India.
4.There is no feeder segregation in rural areas to provide uninterrupted supply for irrigation.
Which of the above statements is/are correct?

(A) Only 2
(B) 2 and 3
(C) Only 3
(D) All
Selected answer: (Not selected..)
The correct answer is:(A)
Solution:A village was deemed electried if it had a single light bulb connection. Subsequently, the denition was upgraded requiring at least ten per cent of homes to be electried
including all common or public areas such as schools and clinics. Based on this denition, the agship Central government programme Rajiv Gandhi Grameen Vidyutikaran
Yojana (RGGVY) electried over 18,000 villages annually for several years, leaving out only some ve per cent remote and distant villages which were being targeted for o-
grid (decentralized) electrication. There is a scheme of feeder segregation in rural areas to separate households from pump set supply, allowing them uninterrupted supply.
This was originally implemented successfully in Gujarat, and now at the national-level by the PanditDeendayalUpadhyaya Feeder Segregation programme.

21. Consider the following statements regarding the National Food Security Act (NFSA), 2013:
1.The Act provides for coverage of 100% of the rural population and upto 75% of the urban population for receiving subsidized foodgrains.
2.The entitlements for the existing Antyodaya Anna Yojana (AAY) households will be replaced and rationalized in consonance with the NFSA.
3.Under NFSA, States/UTs are obliged to provide entitlements in foodgrains only and not in the form of direct cash transfer.
4.The work of identication of eligible households for each State is to be done by the Ministry of Food, Public Distribution on the basis of the NSS Household Consumption
data.
Which of the above statements are correct?

(A) 1, 2 and 3
(B) 2, 3 and 4
(C) 3 and 4
(D) None
Selected answer: (Not selected..)
The correct answer is:(D)
Solution:As passed by the Parliament, Government has notied the National Food Security Act, 2013 with the objective to provide for food and nutritional security in human life cycle
approach, by ensuring access to adequate quantity of quality food at aordable prices to people to live a life with dignity. The Act provides for coverage of upto 75% of the
rural population and upto 50% of the urban population for receiving subsidized foodgrains under Targeted Public Distribution System (TPDS), thus covering about two-thirds
of the population. The eligible persons will be entitled to receive 5 Kgs of foodgrains per person per month at subsidised prices of Rs. 3/2/1 per Kg for rice/wheat/coarse
grains. The existing Antyodaya Anna Yojana (AAY) households, which constitute the poorest of the poor, will continue to receive 35 Kgs of foodgrains per household per
month. 28 States/UTs, namely Andhra Pradesh, Assam, Bihar, Chandigarh, Chhatisgarh, Daman & Diu, Delhi, Goa, Haryana, Himachal Pradesh, Jharkhand, Karnataka,
Lakshadweep, Madhya Pradesh, Maharashtra, Odisha, Puducherry, Punjab, Rajasthan, Sikkim, Telangana, Tripura, Uttarakhand, West Bengal, Uttar Pradesh, Meghalaya,
Jammu & Kashmir and Andaman & Nicobar are implementing the Act at present. Out of these, Chandigarh and Puducherry are implementing the Act in DBT mode i.e. they
are providing direct cash transfer of food subsidy to the beneciaries. Identication of Households: the work of identication of eligible households is to be done by
States/UTs.
22. Consider the following statements regarding the National Mission for Urban Housing:
1.National Mission for Urban Housing seeks to address the housing requirement of urban poor including slum dwellers.
2.Under the mission the cut-o date on which beneciaries need to be resident of that urban area for being eligible to take benets would be decided by the Central
Government.
3.Mission will be implemented as a Central Sector Scheme except for the component of credit linked subsidy which will be implemented as a Centrally Sponsored Scheme
(CSS).
4.The private sector is strictly prohibited under the policy.
Which of the above statements are incorrect?

(A) 1 and 2
(B) Only 4
(C) 2, 3 and 4
(D) None
Selected answer: (Not selected..)
The correct answer is:(C)
Solution:National Mission for Urban Housing seeks to address the housing requirement of urban poor including slum dwellers. Housing for All by 2022" Mission - National Mission for
Urban Housing aimed for urban areas with following components/options to States/Union Territories and cities:-
Slum rehabilitation of Slum Dwellers with participation of private developers using land as a resource;
Promotion of aordable housing for weaker section through credit linked subsidy;
Aordable housing in partnership with Public & Private sectors and
Subsidy for beneciary-led individual house construction or enhancement.
The scheme will be implemented as a Centrally Sponsored Scheme except the credit linked subsidy component, which will be implemented as a Central Sector Scheme.

23. Which of the following are the major programs under the Ministry of Rural Development?
1. Pradhan Mantri Awas Yojna.
2. Deen Dayal Upadhyaya - Grameen Kaushlaya Yojna.
3. Swachh Bharat Mission.
4. National Social Assistance Programme (NSAP).
Select the correct answer using the codes given below:
Codes:

(A) 2 and 4
(B) 1, 2 and 4`
(C) 2, 3 and 4
(D) All
Selected answer: (Not selected..)
The correct answer is:(A)
Solution:Swatch Bharat Abhiyan is under Ministry of Drinking Water and Sanitation./ Pradhan Mantri Awas Yojana is under Ministry of Housing and Urban Poverty Alleviation

24. Consider the following statements regarding the 'USTTAD':


1.USTTAD is 100% Central Sector Scheme of Government of India under the Ministry of HRD.
2.It aims at establishing linkages of traditional skills with the global market.
3.It aims at improving employability of school dropouts.
4.It aims at design development and Research in traditional arts/crafts.
Which of the above statements are correct?

(A) All
(B) 2 and 4
(C) 1, 2 and 3
(D) 3 and 4
Selected answer: (Not selected..)
The correct answer is:(B)
Solution:Ministry of Minority Aairs launches a new scheme "USTTAD? as 100% Central Sector Scheme. This scheme will be implemented from 2014-15 onwards during 12th Five Year
Plan.
Objectives:
To build capacity of master craftsmen/artisans and training of young generation through the master craftsmen/ artisans for traditional arts/crafts.
Set up standards of identied arts/ crafts and their documentation.
To establish linkages of traditional skills with the global market.
To improve employability of existing workers, school dropouts etc.
To generate means of better livelihood for marginalized minorities and bring them in the mainstream.
To enable minorities to avail opportunities in the growing market.
To ensure dignity of labour.
Design development and Research in traditional arts/crafts.

25. Consider the following statements related to the funding pattern of AMRUT:
1.Ministry of Urban Development will allot funds on per city basis in consensus with State government.
2.Incentives for Reforms include 10% of the annual budgetary allocation.
3.The state government will provide 2.5% of the fund for building green spaces.
Which of the above statements is/are incorrect?

(A) Only 2
(B) 1 and 3
(C) All
(D) Only 3
Selected answer: (Not selected..)
The correct answer is:(B)
Solution:.

Information for the solved exam

Started in: March 27, 2017, 12:39 am

Finished in: March 27, 2017, 12:39 am

Marks Obtained: 0

Right Ques.: 0

Wrong Ques.: 0

Unattempted Ques.: 25

Try for a better result? (index.php?section=user&page=continue_exam&id=133408&renew_exam=yes)


Rank List (index.php?section=user&page=ranking&id=103)
Socio - Economic Development Test - 8

1. Which of the following statements is/are correct with respect to Sukanya Samriddhi Account?
1. The account can be opened in any post oce branch and designated public sector banks.
2. The rate of interest for the scheme is xed and subject to revision biannually.
3. The proceeds and maturity amount would be fully exempted from tax and can be used only for educational purposes.
Select the correct answer using the codes given below:
Codes:

(A) Only 1
(B) 1 and 2
(C) 2 and 3
(D) 1 and 3
Selected answer: (Not selected..)
The correct answer is:(A)
Solution:Under the scheme, a parent or legal guardian can open an account in the name of the girl child until she attains the age of ten years. As per the government notication on
the Scheme, the account can be opened in any post oce branch and designated public sector banks. The rate of interest for the scheme is an attractive 9.2 per cent which
will be compounded annually. This rate, however, will be revised every year by the government and will be announced at the time of the Union Budget. The minimum
deposit that needs to be made every year is Rs 1,000, and the maximum amount that can be deposited in a year is Rs 1,50,000. There is no limit on the number of deposits
either in a month or in a nancial year.
The account will be valid for 21 years from the date of opening, after which it will mature and the money will be paid to the girl child in whose name the account had been
opened. If the account is not closed after maturity, the balance amount will continue to earn interest as specied for the scheme from time to time. The account will also
automatically close if the girl child gets married before the completion of the tenure of 21 years.
To meet the nancial requirements of the account holder for the purpose of higher education and marriage, account holder can avail partial withdrawal facility after
attaining 18 years of age.

2. Which of the following statements is/are true?


1. Government has initiated a training program under "Himmat" in all states with an aim to provide digital literacy to non IT literate citizens.
2. "Disha" is a scheme for Leadership Development of Minority Women launched by ministry of minority aairs.
Codes:

(A) Only 1
(B) Only 2
(C) Both
(D) None
Selected answer: (Not selected..)
The correct answer is:(D)
Solution:Government has initiated training program under "Digital Saksharta Abhiyan (DISHA)" in all states with an aim to provide digital literacy to non IT literate citizens.
The objective of NAi Roshni scheme is to empower and instill condence among minority women, including their neighbours from other communities living in the same
village/locality, by providing knowledge, tools and techniques for interacting with Government systems, banks and other institutions at all levels

3. With regard to the types of work which can be undertaken under MGNREGA, which of the following statements is/are true?
1. The act has been modied to ensure that at least 60 % of works being taken in a district in term of cost shall be for productive assets directly linked to agriculture.
2. Housing construction in rural areas has been removed as a permissible activity under MGNREGA.
Select the correct answer using codes given below:
Codes:

(A) Only 1
(B) Only 2
(C) Both
(D) None
Selected answer: (Not selected..)
The correct answer is:(A)
Solution:To devise a better mechanism to improve the quality and durability of assets created under MGNREGS, Schedule I to the MGNREG Act, 2005 has been modied to provide
that at least 60% of the works being taken in a district in terms of cost shall be for creation of productive assets directly linked to agriculture and allied activities through
development of land, water and trees. It has also been notied that the 60:40 ratio of labour to material component will be maintained at the district level (instead of Block
level) for the works to be executed by implementing agencies other than Gram Panchayats. Housing construction in rural areas is included.

4. Which of the following pairs related to types of urban governments is/are correct?
1.Notied Area Committee: All the members of the committee are nominated by the Governor.
2.Cantonment Board: It is created and administered by the Central Government.
3.Port Trust: It consists of both elected and nominated members to provide civic amenities.
Codes:

(A) Only 1
(B) 2 and 3
(C) Only 3
(D) All
Selected answer: (Not selected..)
The correct answer is:(B)
Solution:All members of the notied area committee are nominated by the state government

5. Consider the following statements regarding the Rural Electrication under DDUGJY:
1.Deen Dayal Upadhyaya Gram Jyoti Yojana (DDUGJY) focuses on Feeder separation to ensure sucient power to farmers and regular supply to other consumers.
2.Deen Dayal Upadhyaya Gram Jyoti Yojana will work as a component under the Rurban Mission.
3.Grant portion of the Scheme is 85% for special category States and 60% for other States.
Which of the above statements is/are correct?

(A) 1 and 2
(B) 1 and 3
(C) Only 1
(D) All
Selected answer: (Not selected..)
The correct answer is:(B)
Solution:The DDUGJY is one of the agship programmes of the Ministry of Power and will facilitate 24x7 supply of power. The DDUGJY Scheme approved by the Union Government
draws its inspiration from the similar pioneering scheme implemented by the Government of Gujarat. This scheme will enable to initiate much awaited reforms in the rural
areas. It focuses on feeder separation (rural households & agricultural) and strengthening of sub-transmission & distribution infrastructure including metering at all levels in
rural areas. This will help in providing round the clock power to rural households and adequate power to agricultural consumers .The earlier scheme for rural electrication
viz. Rajiv Gandhi Grameen Vidyutikaran Yojana (RGGVY) has been subsumed in the new scheme as its rural electrication component. The major components of the scheme
are feeder separation; strengthening of sub-transmission and distribution network; Metering at all levels (input points, feeders and distribution transformers); Micro grid and
o grid distribution network & Rural electrication- already sanctioned projects under RGGVY to be completed.

6. Consider the following statements regarding the 'Nai Roshni' scheme which was in news recently:
1.Nai Roshni aims at leadership Development of Minority Women with an aim to empower them.
2.The scheme will be implemented through the National Commission for Women.
Which of the above statements is/are correct?

(A) Only 1
(B) Only 2
(C) Both
(D) None
Selected answer: (Not selected..)
The correct answer is:(A)
Solution:Nai Roshni aims at Leadership Development of Minority Women with an aim to empower and instill condence in women by providing knowledge, tools and techniques for
interacting with Government systems, banks and intermediaries at all levels.
The scheme is implemented through Non -Governmental Organizations (NGOs), Civil Societies, Trust etc. The scheme provides one week training programmes followed by
Handholding for one year. The training is provided on various Training modules covering issues relating to women viz. Leadership of Women through participation in
decision making, Educational Programmes for women, Health and Hygiene, Legal rights of women, Financial Literacy, Digital Literacy, Swachch Bharat, Life Skills and
Advocacy for Social and Behavioural change.

7. Skilled attendance at delivery is an important indicator in monitoring progress towards reduction of maternal mortality ratio. Consider the following statements
regarding the schemes to promote Institutional Delivery:
1.Janani Suraksha Yojana (JSY) entitles all pregnant women delivering in public health institutions to absolutely free delivery services.
2.Janani Suraksha Yojana (JSY) stipulates free drugs, diagnostics, blood besides free transport from home to the institution.
3.Janani Shishu Suraksha Karyakaram (JSSK) is a conditional cash transfer scheme to promote institutional deliveries.
4.The Central government is empowered to get states compelled to implement health laws as health is in Union list.
Which of the above statements are correct?

(A) 1, 3 and 4
(B) None
(C) 1, 2 and 3
(D) 3 and 4
Selected answer: (Not selected..)
The correct answer is:(B)
Solution:Janani Shishu Suraksha Karyakaram (JSSK) which entitles all pregnant women delivering in public health institutions to absolutely free delivery services. Janani Shishu
Suraksha Karyakaram (JSSK) stipulates free drugs, diagnostics, blood, diets besides free transport from home to institution for all the pregnant women in public health
institutions. It is the Janani SurakshaYojana (JSY) which is a conditional cash transfer scheme to promote institutional deliveries not JSSK. Health is a state subject, the Central
government enacts laws but implementation rests with a state government.

8. Which of the following avenues are available for city people to engage in urban planning process apart from the constitutional provisions?
1.Area Sabha cover the citizens who are voters in one or more polling stations, but preferably not covering more than, say, 2500 voters.
2.Wards Committees are constituted consisting of one or more Wards, within the territorial area of a Municipality having a population of three lakhs or more.
Codes:

(A) Only 1
(B) Only 2
(C) Both
(D) None
Selected answer: (Not selected..)
The correct answer is:(B)
Solution:Area Sabha has been recommended by Second ARC but it has not been formed til yet.

9. Which of the following provisions for the rehabilitation of the identied manual scavengers have been stated in the Manual Scavenging Act?
1.Eliminate the insanitary latrines and provide one-time cash assistance.
2.Allotment of residential plot and nancial assistance for house construction of a ready built house.
3.Training in a livelihood skill with payment of stipend of at least Rs 3000 per month.
4.Provision for subsidy, along with concessional loans, to at least one adult member of the family.
Codes:

(A) 1, 2 and 3
(B) 2, 3 and 4
(C) 2 and 3
(D) All
Selected answer: (Not selected..)
The correct answer is:(D)
Solution:The act has provisions for the following measures for the rehabilitation of the identied manual scavengers
An initial one-time cash assistance
Scholarship to the children of manual scavenger
Allotment of residential plot and nancial assistance for house construction of a ready built house
Training in a livelihood skill with payment of stipend of at least Rs 3000 per month
Provision for subsidy, along with concessional loans, to at least one adult member of the family

10. India has the highest number of undernourished people. Which of the following initiatives has been taken by the government to address the issue?
1.Promotion of appropriate infant and young child feeding practices.
2.Setting up of Nutrition Rehabilitation Centres in each village to treat children with severe acute malnutrition.
3.Launch of Mother and Child Protection Card for addressing the nutrition concerns.
Select the correct answer using the codes given below:
Codes:

(A) 1 and 2
(B) 2 and 3
(C) 1 and 3
(D) All
Selected answer: (Not selected..)
The correct answer is:(C)
Solution:Treatment of children with severe acute malnutrition at special units called the Nutrition Rehabilitation Centres (NRCs), set up at public health facilities. Presently 891 such
centres are functional all over the country.

11. Which of the following are the benets of Pradhan Mantri Fasal Bima Yojana?
1.Farmers will pay uniform and low premium for all crops whether Rabi or Kharif.
2.The protection base has been widened as the term Disaster includes Flooding of Crops and Damage after Harvest.
3.Time Bound Payment of Losses will prevent delays as subsidy will be shared equally between centre and state.
Select the correct answer using the codes given below:
Codes:

(A) 1 and 2
(B) Only 3
(C) 2 and 3
(D) All
Selected answer: (Not selected..)
The correct answer is:(C)
Solution:Farmers will pay uniform premium of 2 per cent for all Kharif crops and 1.5 percent for all Rabi crops. In case of annual horticultural and commercial crops, farmers will pay
5 per cent premium.

12. Welfare of Persons with Disabilities has which of the following Relevant Constitutional Provisions?
1.Preamble.
2.Fundamental Rights.
3.Directive Principles of State Policy.
4.Eleventh Schedule
5.Twelfth Schedule
Select the correct answer using the codes given below:
Codes:

(A) 1, 2 and 3
(B) 2, 3, 4 and 5
(C) 1, 2, 4 and 5
(D) All
Selected answer: (Not selected..)
The correct answer is:(D)
Solution:The Constitution of India through its Preamble, inter-alia seeks to secure to all its citizens; justice, social, economic and political; liberty of thought, expression, belief, faith
and worship; equality of status and of opportunity.
Part-III of the Constitution provides for a set of six Fundamental Rights to all the citizens (and in a few cases to non citizens also). These include - Right to Equality; Right to
Freedom; Right against Exploitation; Right to Freedom of Religion; Cultural and Educational Rights and Right to Constitutional Remedies. All these rights are also available to
the Persons with Disabilities even though no specic mention of such persons appear in this Part of the Constitution.
Article 41: Right to work, to education and to public assistance in certain cases, provides: "The State shall, within the limits of its economic capacity and development, make
eective provision for securing the right to work, to education and to public assistance in case of unemployment, old age, sickness and disablement and in other cases of
undeserved want.
Eleventh Schedule to Article 243- G and Twelfth Schedule to Article 243-W, which pertain to the powers and responsibilities of the Panchayats and Municipalities respectively
with respect to implementation of schemes for economic development and social justice, include welfare and safeguarding the interests of Persons with Disabilities among
other weaker sections of the society.

13. Which of the following is not an objective of National Youth Policy?

(A) Create a productive workforce that can make a sustainable contribution to India's economic development.
(B) Instil social value and promote community service to build national ownership.
(C) To develop scientic temper and spirit of inquiry in the Indian youth.
(D) Facilitate participation and civic engagement at level of Governance.
Selected answer: (Not selected..)
The correct answer is:(C)
Solution:It is a Fundamental Duty.
Objectives are:
Create a productive work force that can make a sustainable contribution to Indias economic development.
Develop a strong and healthy generation equipped to take on future challenges.
Instil social values and promote community service to build national ownership.
Facilitate participation and civic engagement at levels of governance.
Support youth at risk and create equitable opportunity for all disadvantaged and marginalised youth.

14. Which of the following initiatives will ensure nancial inclusion?


1.Enrollment in Aadhaar.
2.Reinventing post oces as "lender of last mile".
3.Strengthening prudential norms of banks.
4.MUDRA scheme.
5.Rationalising subsidies.
Codes:

(A) 1, 2 and 3
(B) 1, 2, 4 and 5
(C) 2 and 4
(D) All
Selected answer: (Not selected..)
The correct answer is:(C)
Solution:Only enrollment in Aadhaar does not ensure nancial inclusion. Aadhaar aids other initiatives of nancial inclusion(e.g opening bank account etc.)
Prudential norms are to bring discipline in the management of banks.
Rationalizing subsidies decreases scal burden but does not directly ensure nancial inclusion.

15. Which of the following statements is/are incorrect?


1.National Commission for SC submits annual report to Parliament, which ensures accountability of Government on issues related to SCs.
2.National Commission on ST looks into the issues of Anglo Indians as well.
3.National Commission for Backward Classes is a statutory body, which tenders advice to the Government regarding inclusion and exclusion of communities as backward.
Codes:

(A) Only 3
(B) Only 2
(C) 1 and 2
(D) None
Selected answer: (Not selected..)
The correct answer is:(C)
Solution:National commission on SC gives report to the President.
National commission on SC also looks into the issues with Anglo Indians.

16. Which of the following is/are correct regarding poverty estimation in India?
1.Poverty estimation by Government does not consider social and political deprivation.
2.Mixed recall period involves consumption in the past 7 days and the preceding month.
3.In the recent Rural Development Report, Odisha had the highest percentage of "very poor" across major states.
Codes:

(A) 1 and 3
(B) 1 and 2
(C) All
(D) Only 1
Selected answer: (Not selected..)
The correct answer is:(D)
Solution:The MRP measures consumption of ve low-frequency items (clothing, footwear, durables, education and institutional health expenditure) over the previous year, and all
other items over the previous 30 days.
Chhattisgarh tops the list of states with highest percentage of extremely poor in India.

17. The criteria generally adopted for specication of a community as a Scheduled Tribe are:
1.Indications of primitive traits.
2.Distinctive culture.
3.Extreme Economic backwardness.
4.Extremely low educational levels.
Which of the above stated criterias are correct?

(A) 1 and 2
(B) 1, 2 and 3
(C) 1, 3 and 4
(D) All
Selected answer: (Not selected..)
The correct answer is:(A)

Solution:The criteria generally adopted for specication of a community as a Scheduled Tribe are:
Indications of primitive traits;
Distinctive culture;
Shyness of contact with the community at large;
Geographical isolation i.e. backwardness.

18. Consider the following statements related to the recent insurance schemes launched by GOI:
1.Atal Pension Yojana is for accidental death and full disability.
2.Pradhan Mantri Suraksha Bima Yojna cover for expenses incurred during hospitalisation due to illness or surgery.
3.Pradhan Mantri Jeevan Jyoti Bima Yojana is a life insurance cover for the people in the age group of 18 to 50 and having a bank account.
Which of the above statements are incorrect?

(A) 1 and 2
(B) 2 and 3
(C) 1 and 3
(D) All
Selected answer: (Not selected..)
The correct answer is:(A)
Solution:Atal Pension Yojana provides people with a monthly income when they are no longer earning. A Subscriber receives pension based on accumulated contribution out of his
current income. Under the Atal Pension Yojna Scheme (APY), the subscribers ,under the age of 40, would receive the xed monthly pension of Rs. 1000 to Rs 5000 at the age
of 60 years, depending on their contributions. To make the the pension scheme more attractive, government would co-contribute 50 per cent of a subscriber's contribution
or Rs 1,000 per annum, whichever is lower to each eligible subscriber account for a period of of 5 years from 2015-16 to 2019-20.
Pradhan Mantri Suraksha Bima Yojana oers Accidental death and disability cover.

19. Consider the following statements:


1.The Fifth Schedule under Article 244 (1) of Constitution denes "Tribal Areas" as such areas as the President may by Order declare to be Scheduled Areas after
consultation with the Governor of the state.
2.The Sixth Schedule under Article 244 (2) of the Constitution relates to those areas in the states of Assam, Meghalaya, Tripura and Mizoram which are declared as
"Scheduled Areas" and provides District Councils and/or Regional Councils for such Areas.
Select the correct answer using the codes given below:
Codes:

(A) Only 1
(B) Only 2
(C) Both
(D) None
Selected answer: (Not selected..)
The correct answer is:(D)
Solution:The Fifth Schedule under Article 244 (1) of Constitution denes "Scheduled Areas" as such areas as the President may by Order declare to be Scheduled Areas after
consultation with the Governor of the state.
The Sixth Schedule under Article 244 (2) of the Constitution relates to those areas in the states of Assam, Meghalaya, Tripura and Mizoram which are declared as "Tribal
Areas" and provides for District Councils and/or Regional Councils for such Areas.

20. Which of the following indicators are used in determining the Multidimensional Poverty Index?
1.Child mortality
2.Nutrition
3.Access to credit
4.Cooking Fuel
5.Electricity
Codes:

(A) 1, 2 and 3
(B) 2, 3 and 4
(C) 1, 2, 4 and 5
(D) All
Selected answer: (Not selected..)
The correct answer is:(C)
Solution:Indicators (Child Mortality, Nutrition, Years of schooling, School attendance, Cooking ,fuel, Toilet, Water, Electricity, Floor Assets).

21. The Ministry of Women and Child Development is incharge of the administration of the which of the following Acts?
1.The Dowry Prohibition Act, 1961 as amended in 1986.
2.Juvenile Justice (Care and Protection of Children) Act 2000.
3.Right of Children to Free and Compulsory Education (RTE) Act, 2009.
4.The Protection of Human Rights Act, 1993.
Select the correct answer using the codes given below:
Codes:

(A) 1 and 2
(B) 1, 2 and 3
(C) All
(D) 1, 2 and 4
Selected answer: (Not selected..)
The correct answer is:(A)
Solution:Right of Children to Free and Compulsory Education (RTE) Act, 2009 is under Human Resources Ministry.
The Protection of Human Rights Act, 1993 is under Home Ministry.

22. Which of the following statement correctly denes the Perinatal mortality?

(A) It refers to the number of stillbirths and deaths in the rst week of life.
(B) It encompasses any death of a fetus after 16 weeks of gestation.
(C) It is the death of a live newborn in the rst 28 days of life.
(D) It refers to a death of a live-born baby within the rst seven days of life.
Selected answer: (Not selected..)
The correct answer is:(A)
Solution:The perinatal period commences at 22 completed weeks (154 days) of gestation and ends seven completed days after birth. Perinatal and maternal health are closely linked.
Perinatal mortality refers to the number of stillbirths and deaths in the rst week of life (early neonatal mortality).

23. Consider the following statements regarding the provisions related to Minorities in India:
1.As per the government notication Minority communities in India include Muslims, Sikhs, Buddhists and Anglo-Indians.
2.By Supreme Court judgment transgender have got minorities' status and reservation.
3.According to the Indian Constitution for preservation of linguistic minority a Special Ocer have to be appointed.
Which of the above statements is/are correct?

(A) 1 and 3
(B) 2 and 3
(C) Only 3
(D) Only 1
Selected answer: (Not selected..)
The correct answer is:(C)
Solution:The Ministry of Minority Aairs, a ministry of the Government of India established in 2006. It is the apex body for the central government's regulatory and developmental
programmes for the minority communities in India, which include Muslims, Sikhs, Christians, Buddhists, Zoroastrians (Parsis) and Jains notied as minority communities
under Section 2 (c) of the National Commission for Minorities Act, 1992.
Linguistic Minorities, according Indian Constitution should have a Special Ocer appointed as per Constitutional Article: 350B. Transgenders to enjoy reservations under
OBC quota.

24. Which of the following pairs of land capability classication is/are correctly matched?
1.Land for grazing and forestry - Class VI.
2.Strip cropping - Class II.
3.Wildlife habitat - Class VIII.
Codes:

(A) 1 and 3
(B) Only 2
(C) Only 3
(D) All
Selected answer: (Not selected..)
The correct answer is:(D)
Solution:The classication is as follows:
Class 1 soils have slight limitations that restrict their use.
Class 2 soils have moderate limitations that restrict the choice of plants or that require moderate conservation practices. Used for strip cropping, contour farming, etc.
Class 3 soils have severe limitations that restrict the choice of plants or that require special conservation practices, or both.
Class 4 soils have very severe limitations that restrict the choice of plants or that require very careful management, or both.
Class 5 soils are subject to little or no erosion but have other limitations, impractical to remove, that restrict their use mainly to pasture, rangeland, forestland, or wildlife
habitat.
Class 6 soils have severe limitations that make them generally unsuitable for cultivation and that restrict their use mainly to pasture, rangeland, forestland, or wildlife
habitat.
Class 7 soils have very severe limitations that make them unsuitable for cultivation and that restrict their use mainly to grazing, forestland, or wildlife habitat.
Class 8 soils and miscellaneous areas have limitations that preclude commercial plant production and that restrict their use to recreational purposes, wildlife habitat,
watershed, or esthetic purposes.

25. Which of the following scheme and institution with respect to the livestock sector have been introduced in the recent budget?
1.Nakul Swasthya Patra.
2.Pashudhan Sanjivani.
3.E-Pashudhan Haat.
4.National Genomic Centre for indigenous breeds.
Codes:

(A) Only 1
(B) 1, 2 and 3
(C) 1, 2 and 4
(D) All
Selected answer: (Not selected..)
The correct answer is:(D)
Solution:The budget has provided for Rs. 850 crore in next few years for spending on the 'Pashudhan Sanjivani', an animal wellness programme and provision of Animal Health Cards
('Nakul Swasthya Patra'); 2nd an Advanced breeding technology; 3rd, Creation of 'E-Pashudhan Haat', an e-market portal for connecting breeders and farmers; and 4th, a
National Genomic Centre for indigenous breeds.

Information for the solved exam

Started in: March 27, 2017, 12:36 am

Finished in: March 27, 2017, 12:36 am

Marks Obtained: 0

Right Ques.: 0

Wrong Ques.: 0

Unattempted Ques.: 25
Try for a better result? (index.php?section=user&page=continue_exam&id=133403&renew_exam=yes)

Rank List (index.php?section=user&page=ranking&id=104)


Socio - Economic Development Test - 9

1. Consider the following statements with reference to the Reservation of Seats in the Lok Sabha:
1.The Constitution provides for the reservation of seats for Scheduled Castes and Scheduled Tribes in the Lok Sabha on the basis of population ratios.
2.The Constitution provides the system of communal representation in allocation of seats for minorities.
3.A separate electorate to be constituted for the purpose of election of seats reserved for Scheduled Castes and Scheduled Tribes.
4.A member of Scheduled Castes and Scheduled Tribes is debarred from contesting from a non-reserved seat.
Which of the above statements are incorrect?

(A) 1 and 2
(B) 2, 3 and 4
(C) 2 and 4
(D) 3 and 4
Selected answer: (Not selected..)
The correct answer is:(B)
Solution:The Constitution has abandoned the system of communal representation, it provides for the reservation of seats for Scheduled Castes and Scheduled Tribes in the Lok
Sabha on the basis of population ratios. Though seats are reserved for scheduled castes and scheduled tribes, they are elected by all the voters in a constituency, without
any separate electorate. A member of scheduled castes and scheduled tribes is also not debarred from contesting a general (non-reserved) seat.

2. Which of the following steps have been taken by the government for 'ensuring availability of quality medicines at aordable prices to all'?
1.Price control of Scheduled Drugs through the National Pharmaceutical Pricing Authority.
2.Government has xed a uniform and low rate of 4% VAT on medicines in the country.
3.Selling generic medicines through the dedicated stores under Jan Aushadhi initiative.
Codes:

(A) 1 and 2
(B) Only 3
(C) 2 and 3
(D) All
Selected answer: (Not selected..)
The correct answer is:(D)
Solution:Accordingly, 'ensuring availability of quality medicines at aordable prices to all', has been a key objective of the Government. Some of the important steps taken to enable
this are:
Price control of Scheduled Drugs through the National Pharmaceutical pricing authority (NPPA): Under the Drug Price Control Order, 1995 NPPA has been given the
mandate to control and x the maximum retail prices of a number of scheduled/listed bulk drugs and their formulations, in accordance with well dened criteria and
methods of accounting, relating to costs of production and marketing
Price regulation of Non-Scheduled Drugs: Apart from the scheduled medicines under DPCO, 1995, the NPPA monitors the prices of other medicines not listed in the DPCO
schedule, such that they do not have a price variation of more than 10% per annum.
Uniform VAT of 4% on medicines: Government has xed a uniform and low rate of 4% VAT on medicines in the country.
This policy has been adopted, in almost all the States in the country, and has reduced the incidence of sales tax on medicines and thereby assisted in keeping their prices
low.
The government has decided to launch a country wide Jan Aushadhi Campaign.

3. Union Human Resource Development Ministry has released 'India Rankings 2016' for the survey of educational institutions commissioned by the government. Which of
the following parameters have been used under the survey?
1.Teaching, Learning & Resources available.
2.Research, Professional Practice & Collaborative Performance of the institute.
3.Outreach & Inclusivity.
4.Industrial and vocational training.
5.Perception about the college.
Codes:

(A) 1, 2 and 3
(B) 1, 2, 3 and 4
(C) 1, 2, 3 and 5
(D) All
Selected answer: (Not selected..)
The correct answer is:(C)
Solution:The criteria for ranking included teaching/learning resources, research, graduation outcomes, outreach/inclusivity and perception. The data for the rst four parameters,
which account for 90% of the weightage, was submitted by the institutions and veried by National Institutional Ranking Framework (NIRF), a body constituted by the HRD
ministry last year to conduct annual surveys. For the perception criterion, various stakeholders including parents, teachers, and alumni were engaged to give their feedback.

4. Which of the following statements are correct with respect to the achievement of Goal 1 of Millenium Development Goals by India?
1.India has successfully achieved Goal 1, to eradicate Extreme Poverty and Hunger.
2.India has achieved the target of reducing poverty by half, with 21.9% of its 1.2 billion people living below the poverty line.
3.Estimation of Poverty for this Goal was based on Global Poverty Line of less than $1.0 a day.
Select the correct answer using the codes given below:
Codes:

(A) 1 and 3
(B) 2 and 3
(C) Only 1
(D) Only 2
Selected answer: (Not selected..)

The correct answer is:(D)


Solution:India has partially achieved Goal 1 of Eradicate Extreme Poverty and Hunger.
India has been moderately successful in reducing poverty. In 1990, the all India Poverty Head Count Ratio (PHCR) was estimated to be 47.8%. In order to meet the 2015
target, the PHCR level has to be 23.9%. In 2011-12, the PHCR was 21.9%. This indicates that, India has achieved the poverty reduction target, however, progress is uneven.
Faster reduction in poverty since the mid-2000s helped India halve the incidence of poverty.
However in case of eradicating hunger remains a key challenge. In 1990, when the MDGs were formulated, 53.5 percent of all Indian children were malnourished. Since then,
progress has been slow. In India, the proportion of underweight children below three years has declined marginally between 1998-99 and 2005-06 to 46 percent. In 2015,
malnourishment declined to 40 percent. This is still below the target of reducing malnourishment to 26 percent.
Global Poverty Line was $1.25 a day for MDGs but recently World Bank changed it to $1.9 a day.h'

5. What is the concept of "Tithi bhojan" proposed by the Government?

(A) Government provides food to all village poor on a special occasion.


(B) Involving local community and charitable institutions to improve the quality of food in Mid Day Meal programme.
(C) Providing culturally acceptable food in schools.
(D) It is a scheme for improving nutritional security in tribal areas.
Selected answer: (Not selected..)
Selected answer: (Not selected..)
The correct answer is:(B)
Solution:Under this initiative, food is voluntarily served among school children by villagers in several forms like sweet and namkeen with the regular midday meal and supplementary
nutritive items like sprouted beans.
The objective is to inculcate the feeling of equity and brotherhood among the children of all communities and reduction of the gap between the school administration and
the community.

6. The excessive food stock in the central pool has been optimized through various policy measures.Which of the following measure can be used to optimize the central
food stock?
1.Generating competitiveness in the domestic grain market.
2.Open market sale.
3.Lowering rate of imposition of levy for rice in certain states.
4.Liquidation of excess stock through export.
Codes:

(A) 1 and 2
(B) 2, 3 and 4
(C) 1, 3 and 4
(D) All
Selected answer: (Not selected..)
The correct answer is:(D)
Solution:Due to an open ended procurement policy of food grains providing price support to every farmer, who wants to sell his produce to the Government at MSP, usually
Government agencies end up in procuring more food grains. This has led to accumulation of excess stock under central pool as against the required buer norms. As excess
stock involve carrying cost which adds to the burden of food subsidy on the Government, it is necessary that they are liquidated from time to time either through sale in the
open domestic market or through exports. Liquidation of excess stocks under Open Market Sale Scheme-Domestic (OMSS-D) also helps in keeping food ination under
check and controlling prices of food grains during o season, especially in the decit areas.

7. Trans fats are a type of unsaturated fats that have high health side-eects then also why is it used in food processing industry?
1.It give products a longer shelf life.
2.It helps in improving the texture and avour of the food.
3.It help in preserving the milk products.
Codes:

(A) Only 1
(B) 1 and 2
(C) 2 and 3
(D) All
Selected answer: (Not selected..)
The correct answer is:(B)
Solution:It is used mainly for frying to give longer shelf life & enhancing texture & avour.

8. Recently 62 new Navodaya Vidyalayas have been proposed in Union Budget to promote quality education. Consider the following statements related to it:
1.Its objective is to educate children of the Central Government Employees who are often posted to remote locations to provide uniform education.
2.These are fully residential and co-educational schools aliated to Central Board of Secondary Education (CBSE).
3.Navodaya Vidyalayas exist all over India except Jammu and Kashmir.
Which of the above statements are incorrect?

(A) 1 and 2
(B) Only 3
(C) 1 and 3
(D) Only 1
Selected answer: (Not selected..)
The correct answer is:(C)
Solution:The objective of Kendriya Vidyalaya is to educate children of the Central Government Employees who are often posted to remote locations to provide uniform education.
JNVs exist all over India, with the exception of Tamil Nadu.

9. Which of the following modications have been brought in the 2010 HDI report over the 1990 report?
1.Income measure was changed from Gross National Income to Gross Domestic Product.
2.The 2010 report considers mean years of schooling and expected years of schooling for calculation of knowledge parameter.
Codes:

(A) Only 1
(B) Only 2
(C) Both
(D) None
Selected answer: (Not selected..)
The correct answer is:(B)
Solution:It measures living standards by Gross National Income Per Capita.

10. Which of the following committees have been formed for the Poverty Estimation In India?
1.Alagh Committee
2.Saxena Committee
3.Hashim Committee
4.Naresh Chandra Committee
Select the correct answer using the codes given below:
Codes:

(A) 1, 2 and 3
(B) 2 and 3
(C) 1, 3 and 4
(D) All
Selected answer: (Not selected..)
The correct answer is:(A)
Solution:Naresh Chandra Committee was formed in 2012 to reform Defence Sector.
Following are the committees formed for determination of poverty:
Alagh Committee (1977)
Lakdawala Committee (1989)
Tendulkar Committee (2005)
Saxena committee (2009)
Hashim Committee (2012)
C. Rangrajan Committee (2012)
11. Consider the following statements about "Global Monitoring Report":
1.It examines the impact of demographic change on achieving World Development Goals.
2.The report in 2015 was jointly produced by World Bank and IMF.
3.With the revised poverty line, the recent report stated that the number of people below poverty line has almost doubled in comparison to 2012 gure.
Which of the above statements is/are correct?

(A) Only 1
(B) 1 and 2
(C) All
(D) 2 and 3
Selected answer: (Not selected..)
The correct answer is:(B)
Solution:Global Monitoring Report 2015 , produced jointly by the World Bank and International Monetary Fund, details the progress the world has made towards global development
goals and examines the impact of demographic change on achieving these goals.
The report details the decline of those living in global poverty, which is reclassied as living on $1.90 or less a day, to a forecast 9.6 percent of the world's population in 2015 -
- a projected 200 million fewer people living in extreme poverty than in 2012. It also revises world economic growth projections for 2015 down to 3.3 percent on the basis of
lower growth prospects in emerging markets.

12. Which of the following is/are correct about SABLA scheme?


1.It is implemented by Ministry of Human Resource Development.
2.The scheme targets at adolescent girls for improvement of their nutrition and health status.
3.It aims at improving various vocational skills by integrating with National Skill Development Program.
4.It has provisions for subsidized bank loans for adolescent girls to start self employment business.
Codes:

(A) 1, 3 and 4
(B) 1, 2 and 4
(C) 1, 2 and 3
(D) All
Selected answer: (Not selected..)
The correct answer is:(C)
Solution:The objectives of the program are:
Enable the Adolescent girls for self-development and empowerment
Improve their nutrition and health status.
Promote awareness about health, hygiene, nutrition, adolescent reproductive and sexual health (ARSH) and family and child care.
Upgrade home-based skills, life skills and integrate with the National Skill Development Program (NSDP) for vocational skills.
Mainstream out of school adolescent girls into formal/non formal education.
Provide information/guidance about existing public services such as PHC, CHC, Post Oce, Bank, Police Station, etc.

13. Which of the following Indices are presented by Human Development Report?
1. Gender Development Index.
2. Multi Dimensional Poverty Index.
3. Gender Inequality Index.
4. Inequality Adjusted HDI.
Codes:

(A) 1 and 4
(B) 1, 2 and 4
(C) 2 and 4
(D) All
Selected answer: (Not selected..)
The correct answer is:(D)
Solution:To measure human development more comprehensively, the Human Dev-elopment Report also presents four other composite indices such as:
The Inequality-adjusted HDI discounts the HDI according to the extent of inequality.
The Gender Development Index compares female and male HDI values.
The Gender Inequality Index highlights women's empowerment.
The Multidimensional Poverty Index measures non income dimensions of poverty.

14. Consider the following statements with respect to National Family Health Survey- 4 (NFHS-4):
1.There is a decline in number of Under 5 wasting.
2.Number of people suering from hypertension is more in urban areas than in rural areas.
3.Alcohol and tobacco consumption has fallen since NFHS-3.
4.There is a fall in number of fully immunized children in rich states like Haryana and Maharashtra.
Which of the above statements is/are incorrect?

(A) Only 1
(B) Only 2
(C) 1 and 3
(D) None
Selected answer: (Not selected..)
The correct answer is:(B)
Solution:Under 5: The survey found that there were reduced levels of children under 5 who were wasted., decreased under 5 mortality, increased institutional deliveries, rise in fully
immunized children, increase in children who were breast feeded., increased ante-natal care.
However, the issues which are a cause of concern are slow reduction in child stunting, anaemia, rapid increase in male obesity.
Non communicable disease: Consistent with the burden of non-communicable diseases in India, over-nutrition or obesity among adults has emerged as a major concern. At
least three in 10 women are overweight or obese in the Andaman and Nicobar Islands, Andhra Pradesh, Goa.
Hypertension: While obesity levels have shot up in the country since the last NFHS survey in 2005-06, the number of people suering from hypertension in rural India is, in
many cases, higher than in urban parts.
Among women, obesity Levels shot up from 13.92 per cent in 2005-06 to 19.56 per cent in 2015-16. For men, the rise from the last decade has been from 10.35 per cent to
18.04 per cent.
Alcohol: Men and women across India smoke and drink less than what they used a decade ago.
Tobacco use fallen from 50% to 47%. India has a part of global campaign against tobacco use has introduced a series of measures to control and discourage the use-
increase social awareness, ban on smoking in public places and working areas., establishing smoking zones in hotels, airports and restaurants. (India is a party to WHO
Framework Convention on Tobacco control(FCTC)).

15. The World Development Report (World Bank) highlighted that "To get the most out of the digital revolution, countries need to work on the "analog complements".
Which of the following comes under the analog complements?
1.Strengthening regulations that ensure competition among businesses.
2.Adapting workers' skills to the demands of the new economy.
3.Ensuring that institutions are accountable.
Codes:

(A) Only 1
(B) 1 and 2
(C) All
(D) (d) None
Selected answer: (Not selected..)
The correct answer is:(C)
Solution:Digital technologies have spread rapidly in much of the world. Digital dividends-the broader development benets from using these technologies-have lagged behind. In
many instances digital technologies have boosted growth, expanded opportunities, and improved service delivery. Yet their aggregate impact has fallen short and is
unevenly distributed. For digital technologies to benet everyone everywhere requires closing the remaining digital divide, especially in internet access. But greater digital
adoption will not be enough. To get the most out of the digital revolution, countries also need to work on the "analog complements"-by strengthening regulations that
ensure competition among businesses, by adapting workers'skills to the demands of the new economy, and by ensuring that institutions are accountable.

16. What is "Digital dividend " as emphasized by World Bank?

(A) It is the high rate of return investors get on their investment in long term digital infrastructures.
(B) It is the income generated by Government through improved service delivery using digital technology.
(C) It is the improvement in growth rate, jobs and services in return to investment in digital technology.
(D) It is the extra income that internet service providers accrue due to higher internet use.
Selected answer: (Not selected..)
The correct answer is:(C)

Solution:What are the digital dividends?


Growth,jobs, and services are the most important returns to digital investments. With digital technologies help businesses become more pro- ductive; people nd jobs and
greater opportunities; and govern-ments deliver better public services to all.
By reducing information costs, digital technologies greatly lower the cost of economic and social transactions for rms, individuals, and the public sector. They promote
innovation when transaction costs fall to essentially zero. They boost eciencyas existing activities and services become cheaper, quicker, or more convenient. And they
increase inclusionas people get access to services that previously were out of reach.

17. Consider the following statements related to Human Development Report, 2015 with respect to India:
1.India's HDI has improved 5 notches to 130.
2.The improvement is mainly because of improvement in Life expectancy and Per Capita Income.
3.India still falls in "Low Human Development category".
Which of the above statements is/are correct?

(A) 1 and 2
(B) 2 and 3
(C) 1 and 3
(D) All
Selected answer: (Not selected..)
The correct answer is:(A)
Solution:India continued to rank low in the Human Development Index (HDI), but climbed ve notches to the 130th rank in the latest UNDP report on account of rise in life
expectancy and per capita income.
India ranked 130 among 188 countries in 2014 in Human Development Report 2015 by the United Nations Development Programme (UNDP). The country's rank was 135
according to the 2014 report.
Between 1980 and 2014, India's HDI value increased from 0.362 to 0.609, an increase of 68.1 per cent or an average annual increase of about 1.54 per cent. India is in
medium human development category.

18. What are the main objectives of "policy on food grain stocking"?
1.To meet the prescribed minimum food grain stocking norms for food security.
2.To release food grains for supply through TPDS monthly.
3.To meet emergency situation arising out of unexpected crop failure and natural disasters.
4.For market intervention to augment supply.
Codes:

(A) Only 1
(B) 2 and 3
(C) 1, 2 and 3
(D) All
Selected answer: (Not selected..)
The correct answer is:(D)
Solution:Buer stock of food grains in the Central Pool is maintained by the Government of India (GOI)/Central Government for:
Meeting the prescribed minimum buer stock norms for food security,
Monthly release of food grains for supply through Targeted Public Distribution System (TPDS) and Other Welfare Schemes (OWS),
Meeting emergency situations arising out of unexpected crop failure, natural disasters, etc., and
Price stabilisation or market intervention to augment supply so as to help moderate the open market prices.

19. Which of the following statements are correct about Wheat Based Nutrition programme?
1.The scheme is implemented by Ministry of Food and Civil Supplies.
2.The foodgrains allotted under this scheme are utilized by States under ICDS.
Codes:

(A) Only 1
(B) Only 2
(C) Both
(D) None
Selected answer: (Not selected..)
The correct answer is:(B)
Solution:This Scheme is implemented by the Ministry of Women & Child Development. The foodgrains allotted under this Scheme are utilized by the States/UTs under the Integrated
Child Development Scheme (ICDS) for providing nutritious/ energy food to children below 6 years of age and expectant /lactating women from disadvantaged sections.

20. Which of the following is/are part of 5 fold principles of tribal development in India, put forward by Verrier Elwin?
1.People should develop along the line of their own genius and nothing should be imposed on them from outside.
2.Tribal rights to land and forest should be respected.
3.Tribals should be gradually assimilated in mainland development process.
4.Tribal lands should not be over administered.
Codes:

(A) 1 and 2
(B) 2, 3 and 4
(C) 1, 2 and 4
(D) All
Selected answer: (Not selected..)
The correct answer is:(C)

Solution:The principles are:


People should develop along the line of their own genius and we should avoid imposing anything on them. We should try to encourage in every way their own traditional
arts and culture.
Tribal rights to land and forest should be respected.
We should try to train and build up a team of their own people to do the work of administration and development. Some technical personnel from outside will no doubt, be
needed, especially in the beginning. But we should avoid introducing too many outsiders into tribal territory.
We should not over administer these areas or overwhelm them with a multiplicity of schemes. We should rather works through, and not in rivalry to, their own social and
cultural institutions.
We should judge results, not by statistics or the amount of money spent, but by the quality of human character that is evolved.

21. According to the United Nations, World Water Development Report 2016 which of the following are the likely impacts of water scarcity?
1.Unsustainable management of water may lead to reversing many poverty reduction, job creation and hard-won development gains.
2.Water scarcity will aect geopolitical security due to prompting of migration at various scales.
3.Reduced water availability will impact the women empowerment initiatives.
Codes:

(A) 1 and 3
(B) Only 2
(C) 1 and 2
(D) 1, 2 and 3
Selected answer: (Not selected..)
The correct answer is:(D)
Solution:Water scarcity is likely to limit opportunities for economic growth and the creation of decent jobs in the upcoming years and decades. Unless there is sucient infrastructure
to manage and store the water, as is the case in many developed countries, water availability might vary signicantly, leaving (parts of) countries 'water scarce' for extended
periods. Water availability is also highly dependent on water quality. Poor quality water may not be t for several uses and the cost of the required treatment may be a
prohibiting factor, thus contributing to the burden of economic water scarcity.
Reduced water availability will further intensify competition for water among users, including agriculture, maintenance of ecosystems, human settlements, industry and
energy production. This will aect regional water, energy and food security, and potentially geopolitical security, prompting migration at various scales. The potential impacts
on economic activity and the job market are real and possibly severe. Many developing economies are located in hotspots of water-related stress, particularly in Africa, Asia,
Latin America and the Middle East.

22. Consider the following statements regarding the comparison of India's human development indices with that of other Asian nations:
1.India's Healthy life expectancy at birth is signicantly lower than China, Vietnam and Bangladesh.
2.Indonesia has much higher purchasing power parity (PPP) adjusted per capita national income than India.
3.India's under-ve mortality level is markedly higher than Pakistan.
Which of the above statements is/are correct?

(A) Only 1
(B) Only 2
(C) 2 and 3
(D) 1 and 2
Selected answer: (Not selected..)
The correct answer is:(D)
Solution:India's under-ve mortality level is markedly higher than in all these countries except Pakistan; four times higher than China's and over double Vietnam's.

23. Which of following statements is/are correct regarding Biwako Millennium Framework ?
1.It is action by Governments in the African region to achieve an inclusive, barrier-free and rights-based society for Persons with Disabilities in the new decade, 2015-
2030.
2.This regional framework for action focusses on rights based society for Persons with Disability.
Codes:

(A) Only 1
(B) Only 2
(C) Both
(D) None
Selected answer: (Not selected..)
The correct answer is:(B)
Solution:Biwako framework is for promoting an inclusive, barrier-free and rights-based society for people with disabilities in the Asian and Pacic region in the twenty-rst century,
by which it proclaimed the extension of the Asian and Pacic Decade of Disabled Persons, 1993-2002, for another decade, 2003-2012.

24. Which of following statements is/are correct regarding 'Gift Basket Diplomacy'?
1.It is an approach to multilateral negotiation aimed at pushing forward progress on a particular issue without the requirement of consensus.
2.It is extension of Gujaral doctrine where Developed Nation gives more to the developing nation.
3.It is collective action agreed by smaller groups of participants in multilateral fora without support of major nations.
Codes:

(A) 1 and 3
(B) Only 1
(C) Only 3
(D) 2 and 3
Selected answer: (Not selected..)
The correct answer is:(A)
Solution:Gift basket diplomacy is an approach to multilateral negotiation aimed at pushing forward progress on a particular issue without the requirement of consensus. The policy is
most often seen in United Nations style diplomatic meetings where a particular group of countries wishes to take action or make a joint statement but is unable to do so
without the consensus of all parties involved. Gift basket diplomacy fundamentally is collective action agreed by smaller groups of participants that goes beyond the lowest
common denominator consensus that larger groups often reach in large multilateral fora. The United States rst introduced Gift basket diplomacy in 2011 during the
Nuclear Security Summit preparation process and more than 30 countries participated in Gift basket diplomacy statements at the 2012 Nuclear Security Summit in Seoul.

25. Which of the following institutes are engaged in food and public distribution?
1.Food Corporation of India.
2.Central Warehousing Corporation.
3.Indian Grain Storage Management & Research Institute.
4.Rice Research Institute.
Codes:

(A) 1 and 2
(B) 1 and 3
(C) 1, 2 and 3
(D) All
Selected answer: (Not selected..)
The correct answer is:(C)
Solution:.

Information for the solved exam

Started in: March 27, 2017, 12:35 am

Finished in: March 27, 2017, 12:35 am

Marks Obtained: 0

Right Ques.: 0

Wrong Ques.: 0

Unattempted Ques.: 25

Try for a better result? (index.php?section=user&page=continue_exam&id=133400&renew_exam=yes)

Rank List (index.php?section=user&page=ranking&id=106)


3/27/2017 IASScore:PracticeTests

Socio - Economic Development Test - 1

1. With reference to the Sustainable Development Goal to End poverty in all its forms everywhere, consider the following statements:
1.The Goal state to eradicate extreme poverty for all people everywhere which is currently measured as people living on less than $1.25 a day by 2030.
2.The Goal state to reduce by 2030, at least by half the proportion of men, women and children of all ages living in poverty in all its dimensions according
to global denitions.
3.The Goal state to ensure by 2030, that all men and women, in particular the poor and the vulnerable, have equal rights to economic resources.
Which of the above statements is/are correct?

(A) Only 1
(B) 1 and 2
(C) 1 and 3
(D) All
Selected answer: (Not selected..)
The correct answer is:(C)
Solution:At the United Nations Sustainable Development Summit on 25 September 2015, world leaders adopted the 2030 Agenda for Sustainable Development,
which includes a set of 17 Sustainable Development Goals (SDGs) to end poverty, ght inequality and injustice, and tackle climate change by 2030.
Targets:
By 2030, eradicate extreme poverty for all people everywhere, currently measured as people living on less than $1.25 a day. By 2030, ensure that all men
and women, in particular the poor and the vulnerable, have equal rights to economic resources, as well as access to basic services, ownership and control
over land and other forms of property, inheritance, natural resources, appropriate new technology and nancial services, including micronance. By 2030,
reduce at least by half the proportion of men, women and children of all ages living in poverty in all its dimensions according to national denitions.

2. Consider the following statements regarding the 'The Everlasting Flame'which was in news recently:
1.It is an ambitious project of the Government in collaboration with UNESCO to preserve the rich heritage of the Muslim minority community.
2.This is an ambitious project of the Government under the new scheme "Hamari Dharohar", to preserve the rich heritage of the Parsi minority
community.
Which of the above statements is/are incorrect?

(A) Only 1
(B) Only 2
(C) Both
(D) None
Selected answer: (Not selected..)
The correct answer is:(A)
Solution:The Ministry of Minority Aairs in collaboration with UNESCO Parzor Foundation, has approved three travelling iconic Exhibitions, 'The Everlasting Flame'
along with academic and cultural programmes of international magnitude, to be held from March to May, 2016. It will show-case the civilization and
culture of the Parsis. The exhibition will bring alive Parsi heritage so that all Indians can understand and be proud of India as the world's nest exemplar
of unity in diversity. The Finance Minister has also announced support to the "Everlasting Flame" in the Union Budget 2015-16, keeping Government's
commitment to preserve rich heritage of minority communities of India. This will be the rst ambitious project of the Government under the new scheme
"Hamari Dharohar", to preserve the rich heritage of the Parsi minority community.

3. Consider the following statements regarding the National Commission for SCs:
1.National Commission for SCs is a constitutional body, responsible for the welfare of SCs.
2.National Commission for SCs has power of both civil and criminal court while investigating the matter.
3.National Commission for SCs has power of civil court only while investigating the matter.
Which of the above statements is/are correct?

(A) Only 1
(B) 1 and 2
(C) 1 and 3
(D) All
Selected answer: (Not selected..)
The correct answer is:(C)
Solution:National Commission for SCs is a constitutional body, responsible for the welfare of SCs. It was set up under article 338 of the constitution. Constitution of
India under Article 338 has assigned the following duties and functions to the Commission. The Commission shall, while investigating any matter referred
inquiring into any complaint referred to shall have all the powers of a civil court.

4. Which of the following statement correctly denes the objective of 'Cyber Gram' which was in news recently?

(A) Cyber Gram' aims to impart digital literacy to students of Madarsas and government Schools in identied Minority Concentration
Areas.
(B) Cyber Gram' aims to impart digital literacy to students of tribal areas and government Schools in identied Maoist Left wing
aected areas.
(C) Cyber Gram' aims to impart digital literacy to students of SC communities in states having high concentration of SC population.
(D) Cyber Gram' aims to impart digital literacy to students of government Schools in all villages having 80% sanitation facilities.
Selected answer: (Not selected..)

The correct answer is:(A)


Solution:Cyber Gram' programme aims to impart digital literacy to students of Madarsas and government Schools in identied Minority Concentration Areas. The
programme is implemented through M/s CSC e-Governance Services Ltd., a Special Purpose Vehicle (SPV) created by the Ministry of Electronics & IT, Govt.
of India for implementation of their Common Service Centre (CSC) scheme. As on 31.10.2015, projects for imparting digital literacy to 3,71,657 students
have been approved in the States of West Bengal, Uttar Pradesh, Tripura and Rajasthan.

5. Which of the following are the shortcomings of "Master plan approach" to urban planning?
1.It focuses on peripheral issues like nancing and operating strategy and misses the core issues.
2.Many a times, it is used as a regulatory mechanism rather than being a blueprint of development.
3.It freezes the land use pattern and oor-space index, without considering future urban growth.
Codes:

(A) Only 1
(B) 1 and 2
(C) 2 and 3
(D) All
Selected answer: (Not selected..)
The correct answer is:(C)
Solution:The 'Master Plan' approach generally focuses on only the core area of the city, has little linkages to any nancial and operating strategy and, in many
cases has been used as a regulatory tool instead of being a blue print for development of dynamic and smart cities. A master plan typically freezes the
land use pattern and building byelaws and so on determines the permissible limits of Floor Space Index (FSI) and minimum set-back areas. Often these
provisions do not take into account the potential of the city to grow, especially where trunk infrastructure has been laid. This results in sub-optimal use of
=133375 1/6
3/27/2017 IASScore:PracticeTests
provisions do not take into account the potential of the city to grow, especially where trunk infrastructure has been laid. This results in sub-optimal use of
land.

6. Which of the following statements related to "land readjustment" concept is/are correct?
1.Under this land is purchased from private owners and pooled in a land bank.
2.Under this municipality authority create infrastructure by pooling money from the private stakeholders.
3.Under this the remaining land, whose value has increased due to provision of infrastructure, is reallocated back to participating private land- owners.
Codes:

(A) Only 2
(B) 1 and 2
(C) Only 3
(D) None
Selected answer: (Not selected..)
The correct answer is:(C)
Solution:Land readjustment (LR) is gaining acceptance as an alternative to land acquisition as it has many advantages for land assembly. Under this process, a
compact area is selected in consultation with the land owners for urban expansion/renewal. The municipal authorities provide infrastructure which is
funded by exploiting a part of land. The remaining land, whose value has increased due to provision of infrastructure, is reallocated back to participating
private land- owners. In essence a participatory tool, LR avoids public discontent and protests to a great extent. It also reduces the need for raising large
amounts of money for acquiring land. However, successful LR is grounded in three main enablers:
Fairly well-dened property rights.
Streamlined, independent and transparent evaluation processes.
Strong judicial system to address public concerns.

7. Arrange these sources of credit for rural households in increasing order of uptake?
1. Moneylenders
2. Government
3. Cooperative banks/societies
Select the correct order using the codes below:
Codes:

(A) 1-2-3
(B) 2-1-3
(C) 2-3-1
(D) 3-21
Selected answer: (Not selected..)
The correct answer is:(C)
Solution:According to Indian Council for Research on International Economic Relations the source of rural credit according to 2013 data are: Moneylenders (33.2%);
Government (1.2%) and Cooperative banks/societies (24.8%).

8. Which of the following steps can be employed to ensure quality and aordable urban transport?
1.Promoting high speed urban rail.
2.Integration of dierent modes through smart card.
3.Disincentivising private transport
4.Social and gender auditing of transport projects.
5.Promote non motorised transport.
Codes:

(A) 1, 2 and 3
(B) 1, 2, 3 and 5
(C) 1, 4 and 5
(D) All
Selected answer: (Not selected..)
The correct answer is:(D)
Solution:The measures recommended for the urban transport sector under 12FYP are:
Strengthen Urban Transport Wing in MoUD.
Constitution of National Urban Rail Transit Authority
Setting up of a Research Centre for Rail-based Urban Mass Transport System
Promote High Speed Urban Rail and the Regional Rapid Transit System
Intelligent Transport System and Seamless Integration of Dierent Modes through Smart Card
Promote Non-motorised Transport (NMT)
Social and Gender Auditing of Transport Projects
Institute a Safety Commission for Rail/ Guided and Road Transport
Promote PPP Arrangements, where Appropriate

9. What are the benets of adopting of "strategic densication" in the context of Urban development?
1.It makes the city compact and ecient.
2.It generates revenue for further urban infrastructure development by pricing higher Floor-Space Indexes.
3.It frees space for providing other amenities.
Codes:

(A) Only 2
(B) Only 1
(C) 1 and 3
(D) All
Selected answer: (Not selected..)
The correct answer is:(D)
Solution:Strategic densication of cities through higher FSI has numerous advantages: it makes the cities compact and ecient and frees space for
accommodating more people as well as for providing urban amenities. Pricing of higher FSI also generates resources for funding urban infrastructure
projects.

10. APJ Abdul Kalam Amrut Yojana relates to:


(A) Providing one cooked meal to pregnant, lactating women in tribal areas.
(B) Improving the vaccination rates in rural India to tackle preventable diseases.
(C) Registering all Mid-Day meal serving NGOs and providing them single window administrative clearances.
(D) None of the above.
Selected answer: (Not selected..)
The correct answer is:(A)
Solution:It was recently approved by Maharashtra government to curb malnutrition among tribal children by providing nutrition to pregnant women for 6 months
when the child's growth is at its peak. The scheme is an initiative of Tribal Development department of state government.
Implementation will be done by Anganwadis falling under Women and Child Welfare Department in 16 districts of states having tribal population will
implement it. The scheme Plan is to provide one full hot cooked nutritious food to every pregnant (in third trimester) and lactating mother (rst three
=133375 2/6
3/27/2017 IASScore:PracticeTests
implement it. The scheme Plan is to provide one full hot cooked nutritious food to every pregnant (in third trimester) and lactating mother (rst three
months post-delivery) in tribal areas.

11. With regard to panchayats in India, which of the following statements is/are incorrect?
1. The President shall, after every ve years, constitute a Financial Commission to review the nancial position of panchayats.
2. The Central Finance Commission shall suggest measures needed to augment the Consolidated Fund of State to supplement the resources of the
panchayats in the states by consulting the President and Governor.
Select the correct answer using codes given below:
Codes:

(A) Only 1
(B) Only 2
(C) Both
(D) None
Selected answer: (Not selected..)
The correct answer is:(A)
Solution:Article 280 of the Constitution of India requires the Constitution of a Finance Commission every ve years, or earlier. The Finance Commission is also
required to recommend on 'the measures needed to augment the Consolidated Fund of a State to supplement the resources of the Panchayats and
Municipalities in the State on the basis of the recommendations made by the Finance Commission of the State'.
FFC has recommended distribution of grants to States for local bodies using 2011 population data with weight of 90% and area with weight of 10%. The
grants to States will be divided into two, a grant to duly constituted Gram Panchayats and a grant to duly constituted Municipal bodies, on the basis of
rural and urban population.
FFC has recommended grants in two parts; a basic grant, and a performance grant, for duly constituted Gram Panchayats and municipalities. The ratio of
basic to performance grant is 90:10 with respect to Panchayats and 80:20 with respect to Municipalities.

12. India is said to be taking steps in order to be at the forefront of the 'war against Neglected Tropical Diseases (NTDs)'. What are NTDs?

(A) A group of infectious diseases caused by bacterias, viruses which primarily aect the poorest sections of society, in both rural
and urban areas.
(B) A group of infectious diseases which primarily aect the tribal population.
(C) A group of infectious diseases which aect people regardless of their socio-economic status and caused by bacterias.
(D) None of the above.
Selected answer: (Not selected..)
The correct answer is:(A)
Solution:The neglected tropical diseases (NTDs) are a group of infectious diseases which primarily aect the poorest sectors of society, especially the rural poor
and the most disadvantaged urban populations. Nearly one billion people in the world suer from NTDs, which are referred to as "neglected" because
they are characterized by little attention from policymakers, lack of priority within health strategies, inadequate research, limited resource allocation and
few interventions

13. Which of the following statements about 'barefoot' technicians involved in better implementation of MGNREGA is/are correct?
1.Under this the technicians would be trained in basic concepts of civil engineering.
2.Any person above the age of 21 is eligible for training and will be selected from the local SC/ST MGNREGA worker households.
Select the correct answer using codes given below:
Codes:

(A) Only 1
(B) Only 2
(C) Both
(D) None
Selected answer: (Not selected..)
The correct answer is:(A)
Solution:A barefoot technician is an educated person, identied from the local SC/ST MGNREGA worker's households and specially trained in civil engineering
concepts, using the customised training modules.
Only an educated individual is eligible to become a barefoot technician, not 'any' person.

14. Which of the following are the main factors behind lack of people's participation in Urban Governance?
1.Representation ratio between citizens and their elected representatives is very high in urban areas as compared to rural.
2.Ward committees have not been constituted in many areas.
3.More uid nature of Urban dwellers,who do not feel stake in Governance.
Codes:

(A) Only 2
(B) 1 and 3
(C) All
(D) 2 and 3
Selected answer: (Not selected..)
The correct answer is:(C)
Solution:For increment in public participation Ward Committees and Area Sabhas should be set up for eectively carrying out the functions devolved to ULBs
under the Twelfth schedule of the Constitution. For this purpose, Area Sabhas would be constituted by comprising all citizens in one or two polling station
areas who should elect, once in a ve year, a small Committee of representatives. Further, Ward Committees should be set up in every electoral ward of
Municipalities and Panchayats by drawing representatives from Area Sabhas. Together, these institutions will ensure that executive power is located at
the ULB level, while the deliberative powers are vested with the Ward Committee. These structures will institutionalize participatory and accountability
mechanisms.

15. With reference to Lead Bank Scheme, consider the following statements:
1. It is applicable in both rural and urban areas.
2. Eradication of unemployment is one of its objectives.
3.Usha Thorat Committee has recommended to revitalise it to increase nancial inclusion.
Which of the above statements is/are correct?

(A) Only 1
(B) Only 2
(C) 2 and 3
(D) All
Selected answer: (Not selected..)
The correct answer is:(D)
Solution:Lead Bank Scheme (LBS) was introduced in 1969, based on the recommendations of the Gadgil Study Group.
Objectives of Lead Bank Scheme: Eradication of unemployment and under employment Appreciable rise in the standard of living for the poorest of the
poor Provision of some of the basic needs of the people who belong to poor sections of the society. Usha Thorat Committee has recommended that LBS
should be continued to accelerate nancial inclusion in the unbanked areas of the country.
=133375 3/6
3/27/2017 IASScore:PracticeTests
should be continued to accelerate nancial inclusion in the unbanked areas of the country.

16. Which of the following parameters are included under "Automatic Inclusion" category under the recently conducted Socio Economic Caste Census?
1.Destitute, living on alms.
2.Households with only one room, kuccha walls and kuccha roof.
3.Manual scavenger families.
4.Primitive tribal groups.
5.Legally released bonded labor.
Select the correct answer using the codes given below:
Codes:

(A) 1, 2 and 3
(B) 3, 4 and 5
(C) 2, 3, 4 and 5
(D) 1, 3, 4 and 5
Selected answer: (Not selected..)
The correct answer is:(D)
Solution:Under SECC household with any of the following will be included automatically:
Households without shelter
Destitute/ living on alms
Manual scavengers
Legally released bonded labourers
Primitive tribal groups

17. Which of the following schemes aim to reduce poverty among rural poor by enabling them to access gainful and sustainable employment?
1.Deen Dayal Upadhyaya Antyodaya Yojana.
2.Himayat.
3.Nai Roshini.
4.Start up Village Entrepreneurship Programme (SVEP).
Select the correct answer using the codes given below:
Codes:

(A) 1, 2 and 3
(B) 1 and 4
(C) 1, 2 and 4
(D) 1, 2, 3 and 4
Selected answer: (Not selected..)
The correct answer is:(C)
Solution:Deen Dayal Upadhyaya Antyodaya Yojana or DAY is a Government of India scheme for the helping the poor by providing skill training.
Himayat is a training-cum-placement programme for unemployed youth in Jammu and Kashmir.
Nai Roshni objective is to Empower and install condence in women of minority communities by equipping them with knowledge, tools and techniques to
interact with government systems, banks and intermediaries.
Village Entrepreneurship Programme to foster 1.82 lakh entrepreneurs over a period of four years, in 40 blocks across 14 states.

18. The Shyama Prasad Mukherji Rurban Mission (SPMRM) will develop a cluster of Smart Villages. Which of the following statements is/are true regarding
rurban clusters?
1. The Ministry of Rural Development would identify the clusters in accordance with the Framework for Implementation prepared through scientic
analysis.
2. The States would prepare comprehensive Integrated Cluster Action Plans detailing out the strategy for the cluster and Critical Gap Funding required for
it.
Select the correct answer using the code given below:
Codes:

(A) Only 1
(B) Only 2
(C) Both
(D) None
Selected answer: (Not selected..)
The correct answer is:(B)
Solution:The State Governments would identify the clusters in accordance with the Framework for Implementation prepared by the Ministry of Rural Development.
The clusters will be geographically contiguous Gram Panchayats with a population of about 25000 to 50000 in plain and coastal areas and a population of
5000 to 15000 in desert, hilly or tribal areas. There would be a separate approach for selection of clusters in Tribal and Non-Tribal Districts. As far as
practicable, clusters of village would follow administrative convergence units of Gram Panchayats.
For the selection of clusters, the Ministry of Rural Development is adopting a scientic process of cluster selection which involves an objective analysis at
the District, Sub District and Village level, of the demography, economy, tourism and pilgrimage signicance and transportation corridor impact. While the
Ministry, following this analysis, would provide a suggestive list of sub districts to the State, the State Governments would then select the clusters
following a set of indicated principles included in the Framework for Implementation.

19. Which of the following form the non-tax revenue of Municipalities?


1.User charges
2.Rent
3.Interest
4.Prots/Dividends
Codes:

(A) 1, 2 and 3
(B) Only 1
(C) 1, 2 and 4
(D) All
Selected answer: (Not selected..)
The correct answer is:(D)

Solution:Receipts in case of an urban local body can be broadly classied as follows:


Tax Revenue - proper ty tax, advertisement tax etc.
Non-Tax Revenue - income in terms of rent, royalty, interest, fees and prots/dividends, user charges for public utilities such as water, sewage etc.
Devolution of funds from the State Government.
Grants from Union and State Governments for development schemes.
Borrowings

20. The price stabilisation fund (PSF) that had been announced in July 2014 budget with corpus of Rs 500 crore will be used to support market
interventions for managing prices of which of the following commodities?
1. Pulses
2. Tea and Coee
3. Potato
4. Onion
Select the correct answer using the codes given below:
=133375 4/6
3/27/2017 IASScore:PracticeTests
Select the correct answer using the codes given below:
Codes:

(A) Only 1
(B) 3 and 4
(C) Only 2
(D) 2, 3 and 4
Selected answer: (Not selected..)
The correct answer is:(B)
Solution:The fund, with a corpus of Rs. 500 crore, will be used to support market interventions for managing prices of perishable agri-horticultural commodities.
Initially, the fund is proposed to be used for onion and potato only. Losses incurred, if any, in the operations will be shared between the Centre and the
States.
The PSF will be used to advance interest-free loans to State governments and Central agencies to support their working capital and other expenses on
procurement and distribution interventions for such commodities.
These commodities will be procured directly from farmers or farmers organisations right at the farm gate or mandi levels and be made available at
reasonable prices to consumers.

21. The benets of MGNREGA in rural areas are:


1.It increases the wage rate in rural areas and strengthen the rural economy through the creation of infrastructure assets.
2.It ensures sustainable development by developing the natural resources of land and water.
3.PRIs get strengthen and improvise Human Development Index.
Which of the above stated benets are correct?

(A) 1 and 2
(B) 2 and 3
(C) 1 and 3
(D) All
Selected answer: (Not selected..)
The correct answer is:(D)
Solution:.

22. Which of the following initiatives of Ministry of Health and Family Welfare have been correctly matched?
1.Daksh - for improving the skills of healthcare providers in RMNCH+A services.
2.Kayakalp - for appraising public health facilities in cleanliness, hygiene parameters.
3.Kilkari - to initiate awareness among pregnant women about Anti Natal Care.
Codes:

(A) 1 and 3
(B) 2 and 3
(C) Only 3
(D) All
Selected answer: (Not selected..)
The correct answer is:(D)
Solution:Daksh: For improving the skills of healthcare providers and to enhance their capacity to provide quality (Reproductive, Maternal, Neonatal, Child &
Adolescent Health) RMNCH+A services, Government of India has established ve National Skills lab ''Daksh''.
Kilkari & Mobile Academy: To create proper awareness among pregnant women, parents of children and eld workers about the importance of Anti Natal
Care (ANC), institutional delivery, Post-Natal Care (PNC) and immunization, it was decided to implement the Kilkari and Mobile Academy services in pan
India in phased manner.
Kayakalp- initiative has been launched to promote cleanliness, hygiene and infection control practices in public health facilities.

23. Consider the following statements related to the KIDSRIGHTS INDEX:


1.The KidsRights Index is the annual global index which ranks how countries adhere to and are equipped to improve children's rights.
2.It comprises a ranking for all UN member states.
3.It calculates index in ve dierent domains: the right to life, health, education, protection and child rights environment.
Which of the above statements is/are correct?

(A) 1 and 3

(B) Only 3
(C) 2 and 3
(D) All
Selected answer: (Not selected..)
The correct answer is:(A)
Solution:The KidsRights Index is the annual global index which ranks how countries adhere to and are equipped to improve children's rights. The Kids Rights Index
is an initiative of the Kids Rights Foundation, in cooperation with Erasmus University Rotterdam; Erasmus School of Economics and the International
Institute of Social Studies. It comprises a ranking for all UN member states that have ratied the UN Convention on the Rights of the Child and for which
sucient data is available, a total of 165 countries.

24. Consider the following statements related to the pattern of urbanization in India:
1.India's urban growth is largely concentrated in Class I cities and the number of metropolitan cities has increased from 35 in 2001 to 53 in 2011.
2.The share of the population in Class II-IV+ cities are increasing due to rural-urban migration to 31% in 2015.
Which of the above statements is/are correct?

(A) Only 1
(B) Only 2
(C) Both
(D) None
Selected answer: (Not selected..)
The correct answer is:(A)
Solution:The population growth in smaller cities has tended to stagnate or slow down, with the share of the population in Class II-IV+ cities decreasing from 31% in
2001 to 28% in 2015.

25. The Government of India as well as states has adopted various approaches to tackle the country's urban development challenges. Which of the
following is/are part of it?
1.Development of Industrial Corridors
2.Make in India
3.Regeneration of Heritage Cities
4.Clean India Mission
5.Country specic tie-ups as with China, Japan and USA.
Codes:

=133375 5/6
3/27/2017 IASScore:PracticeTests
(A) 1, 2 and 3
(B) 1, 3, 4 and 5
(C) 1, 3 and 4
(D) All
Selected answer: (Not selected..)
The correct answer is:(D)
Solution:Projects include the development of ve industrial corridors, of which only the Delhi-Mumbai Industrial Corridor (DMIC) is under development. Private
urban development projects include the building of whole new towns such as Lavasa and Palava. Since taking oce, the new Government of India has
announced several policies and urban development initiatives. The most prominent is the 100 Smart Cities programme, although other initiatives have
been launched such as the Redevelopment and Urban Renewal of 500 Cities, the Regeneration of Heritage Cities and various country-specic tie-ups.
Furthermore, policies relevant to urban development, such as the Make in India programme and the Clean India Campaign, have been announced.

Information for the solved exam

Started in: March 27, 2017, 12:20 am

Finished in: March 27, 2017, 12:20 am

Marks Obtained: 0

Right Ques.: 0

Wrong Ques.: 0

Unattempted Ques.: 25

Try for a better result? (index.php?section=user&page=continue_exam&id=133375&renew_exam=yes)

Rank List (index.php?section=user&page=ranking&id=36)

=133375 6/6
3/27/2017 IASScore:PracticeTests

Socio - Economic Development Test-17

1. Which of the following statement related to the Swachh Bharat Kosh is correct?

(A) The Swachh Bharat Kosh has been established to collect funds from non-governmental sources.
(B) The Swachh Bharat Kosh has been established to collect funds from governmental budget as well as non-governmental sources.
(C) The Swachh Bharat Kosh has been established to collect funds from only Corporate Social Responsibility criteria as mentioned in
the Companies Act.
(D) The Swachh Bharat Kosh has been established to collect funds from common people for providing hygiene and sanitation
services.
Selected answer: (Not selected..)
The correct answer is:(A)
Solution:.

2. Inclusive growth is an important objective of the development process. In this context, which of the following would lead to inclusive growth?
1. Poverty reduction
2. e-literacy
3. Aordable health insurance
4. Skill development
Select the correct option using the codes given below:
Codes:

(A) 1, 2 and 3
(B) 1, 3 and 4
(C) 2, 3 and 4
(D) All
Selected answer: (Not selected..)
The correct answer is:(D)
Solution:.

3. The Model Act of APMC has the provision for contract farming. What are the risk factors involved in contract farming?
1.To qualify for contract farming, the Model Act imposes the conditionality that the agriculturist has to have sold agricultural produce in the market for
two years.
2.In contract farming, farmers have to depend on external sources for food because sponsors favour growing commercial crops rather than food crops.
3.On the failure of crop sole responsibility lies on the farmer.
Which of the above stated risk factors is/are correct?

(A) Only 1
(B) Only 2
(C) 2 and 3
(D) All
Selected answer: (Not selected..)
The correct answer is:(B)
Solution:.

4. Consider the following statements related to the Housing for All scheme:
1.This scheme aims at Slum rehabilitation of Slum Dwellers with participation of private developers using land as a resource.
2.The house ownership would be in the name of female or joint holding.
3.The denition of Economically Weaker Sections and the Lower Income group has been redened under this scheme.
Which of the above statements is/are correct?

(A) 1 and 2
(B) Only 1
(C) 2 and 3
(D) All
Selected answer: (Not selected..)
The correct answer is:(D)
Solution:.

5. The Mid-day Meal Scheme is a school meal programme of the government of India designed to improve the nutritional status of school-age children
nationwide. The scheme covers students of which of the following entities?
1.Primary and upper primary government schools.
2.National Child Labour Project Schools.
3.Madarsas under Sarva Siksha Abhiyan.
Codes:

(A) Only 1
(B) 1 and 2
(C) 1 and 3
(D) All

Selected answer: (Not selected..)


The correct answer is:(D)
Solution:.

6. Food Security Allowance in National Food Security Mission implies:

(A) Pregnant women and lactating mothers will be entitled to receive allowance of not less than Rs. 6,000.
(B) Pregnant women and lactating mothers and children in the age group of 6 months to 14 years will be entitled to allowance in
form of meal as per prescribed nutritional norms.
(C) It is the penalty on public servant or authority in case of failure to comply with the relief recommended by the District Grievance
Redressal Ocer.
(D) This is the allowance entitled to beneciaries in case of non-supply of entitled foodgrains or meals.
Selected answer: (Not selected..)
The correct answer is:(D)
Solution:.

7. Which of the following provision is incorrect for identication of villages in Sansad Adarsh Gram Yojana?

(A) MPs can select any gram panchayat, other than their own village or that of their spouse, to be developed as an Adarsh Gram.
=133376 1/5
3/27/2017 IASScore:PracticeTests
(A) MPs can select any gram panchayat, other than their own village or that of their spouse, to be developed as an Adarsh Gram.
(B) Nominated members can choose a village from any district of the country.
(C) MPs which represent urban constituencies need not to select a village, they can transfer their money to other MPs.
(D) Lok Sabha MPs can choose a village from their constituency.
Selected answer: (Not selected..)
The correct answer is:(C)
Solution:.

8. The National Florence Nightingale Award instituted by the Ministry of Health and Family Welfare confers awards to:

(A) Nursing personnels.


(B) Doctors in government hospitals.
(C) Topper in medicine in government colleges.
(D) Teacher of government colleges.
Selected answer: (Not selected..)
The correct answer is:(A)
Solution:.

9. Rashtriya Madhyamik Shiksha Abhiyan (RMSA) aims at enhancing access to secondary education and improving its quality by:
1.Eradicating gender dierences.
2.Achieving universal retention by 2025.
3.Establishing secondary school within a radius of 5 km.
Which of the above statements is/are correct?

(A) Only 3
(B) 1 and 3
(C) Only 2
(D) All
Selected answer: (Not selected..)
The correct answer is:(B)
Solution:It aims at achieving universal retention by 2020.

10. Union MOHFW has launched the National De-Worming initiative. Which of the following problems due to worming will the scheme tackle?
1.Malnutrition
2.Wasting
3.Cognitive impairment
4.Tissue damage that may require corrective surgery.
Choose the correct option using the codes below:

(A) 1 and 4
(B) 2 and 3
(C) 1, 3 and 4
(D) All
Selected answer: (Not selected..)
The correct answer is:(D)
Solution:.

11. Recently which scheme for girls has been launched along with, Beti Bachao, Beti Padhao campaign?

(A) Sukanya Samriddhi Yojana.


(B) Best Sele with daughter award in Haryana villages.
(C) Kanya Jeevanshree.
(D) Kanya Suraksha Yojana.
Selected answer: (Not selected..)
The correct answer is:(A)
Solution:.

12. Consider the following statements related to the Rural Entrepreneurship and Awareness Development Yojana:
1.It involves linkage of farmers with scientists to improve their entrepreneurship skill.
2.It involves developing entrepr-eneurship among youth through skill development of students in project mode in dierent agricultural universities.
3.It aims to check the migration of rural youth and retain their interest in agriculture through creation of new employment opportunities in linkage with
scientic knowledge.
Which of the above statements is/are correct?

(A) Only 1
(B) Only 2
(C) 2 and 3
(D) All
Selected answer: (Not selected..)
The correct answer is:(B)
Solution:Student READY (Rural Entrepreneurship and Awareness Development Yojana) program in agricultural education for developing entrepreneurship among
youth through skill development of students in project mode will be implemented in dierent agricultural universities (Budget allocation Rs. 50 crores).

13. Consider the following statements about 'Beti Bachao Beti Padhao' campaign:
1.It will be implemented all over the country.
2.The key interventions under the scheme include improved access to ICDS services, increasing institutional deliveries and registration of births.
3.It will only work for Gender biased sex selective elimination.
Which of the above statements is/are correct?

(A) Only 2
(B) 1 and 2
(C) 2 and 3
(D) All
Selected answer: (Not selected..)
The correct answer is:(A)
Solution:The Beti Bachao Beti Padhao Scheme will be implemented in 100 selected districts, which will cover all states and UTs and is a joint initiative of Women
and Child Development, Human Resource Development and Health Ministries. These districts have Child Sex Ratio, far below the national ratio of 918 girls
per 1000 boys.

14. The National Food Security Act, aims to provide subsidized foodgrains to approx-imately two thirds of population. The responsibility of Central
Government under PDS system are:
1.Storage of foodgrains.
=133376 2/5
3/27/2017 IASScore:PracticeTests
1.Storage of foodgrains.
2.Issue ration cards.
3.Identication of families of BPL
4.Allocation of foodgrains.
Which of the above stated responsibilities are correct?

(A) 1, 2 and 3
(B) 1 and 4
(C) 1, 3 and 4
(D) All
Selected answer: (Not selected..)
The correct answer is:(B)
Solution:The Central Government has taken the responsibility for procurement, storage, transportation and bulk allocation of foodgrains, etc. The responsibility for
distributing the same to the consumers through the network of Fair Price Shops (FPSs) rests with the State Governments.

15. Inclusive development approaches of the UNDP to achieve the Millennium Development Goals (MDGs) includes:
1.Building eective and ecient social safety nets to protect those who cannot work, or, who earn too little.
2.Creating productive and gainful employment.
3.People sharing the benets of development and participating in decision-making.
Which of the above statements are correct?

(A) 1 and 2
(B) 2 and 3
(C) 1 and 3
(D) All
Selected answer: (Not selected..)
The correct answer is:(D)
Solution:.

16. A new Scheme National Sports Talent Search Scheme (NSTSS) has been formulated for identication of Sporting Talent amongst children (both Boys
and Girls) in the age group of:

(A) 12-16 years.


(B) 8-12 years.
(C) 11-15 years.
(D) 12-15 years.
Selected answer: (Not selected..)
The correct answer is:(B)
Solution:National Sports Talent Search Scheme (NSTSS) aims at identication of talented sportspersons in the age group of 8 - 12 years in schools all over the
country through a battery of tests and nurturing of identied talented sportspersons in sports schools will help broaden the pool of sportspersons in the
country.

17. Which of the following statements are correct about National Health Family Survey?
1.Survey is coordinated by Ministry of AYUSH.
2.Survey provides information on important emerging health and family welfare issues.
3.The NFHS-4 for the rst time is collecting data from all 29 states and all 7 Union Territories, also at the district level.
4.NFHS-4 exclude data on HIV as it will be covered by National AIDS Control Organization.
Select the correct answer using below codes:
Codes:

(A) 2 and 3
(B) 1 and 4
(C) 1, 2 and 3
(D) All
Selected answer: (Not selected..)
The correct answer is:(A)
Solution:The Ministry of Health and Family Welfare, Government of India, designated the International Institute for Population Sciences (IIPS), Mumbai, as the
nodal agency responsible for providing coordination and technical guidance for the survey. IIPS collaborated with a number of eld organizations for
survey implementation.
The NFHS has had two specic goals: to provide essential data on health and family welfare needed by the Ministry of Health and Family Welfare and
other agencies for policy and program purposes, and to provide information on important emerging health and family welfare issues.
The NFHS-4 is for the rst time collecting data from all 29 states and all 7 Union Territories. For the rst time, the NFHS-4 will provide estimates at the
district level. Given the wide intra-state variations, the disaggregated data at the district level helps in better understanding of the data and future policy
formulation.
NFHS- will provide updates and evidence of trends in key population, health and nutrition indicators, including HIV prevalence. The information will
enable the GOI to provide national and international agencies to monitor and evaluate policies and programmes related to population, health, nutrition,
and HIV/AIDS.

18. Which of the following is/are the objectives of Khelo India?


1.Mass participation of youth in annual sports competitions.
2.Identication of talent.
3.Creation of Sports Infrastructure at mofussil, Tehsil, District, State levels, etc.
4.Guidance and nurturing of the talent through sports academies set up only by the central government.
Select the correct answer using the codes given below:
Codes:

(A) 1, 2 and 3
(B) 2 and 3
(C) Only 2
(D) All
Selected answer: (Not selected..)
The correct answer is:(A)
Solution:Khelo India, a national programme for development of sports, a prestigious initiative by the Government. Rajiv Gandhi KhelABhiyan (RGKA) with the Khelo
India programme in 2016.
Objectives of Scheme:
Mass participation of youth in annual sports competitions through a structured competition.
Identication of talent.
Guidance and nurturing of the talent through existing sports academies and new set up either by the central Government or State Government or in PPP
mode.
Creation of Sports Infrastructure at mofussil, Tehsil, District, State levels, etc.

19. Which of the following are the objectives of the Rashtriya Uchchatar Shiksha Abhiyan?
1.The scheme aims at improving the overall quality of existing central higher educational institutions.
2.It aims at ensuring adequate availability of quality faculty in all state higher educational institutions.
3.It aims at integrating skill development eorts of the government with the conventional higher education system through optimum interventions.
Select the correct answer using the codes given below:
Codes:

(A) 1 and 2
(B) 2 and 3
(C)
=133376 3/5
3/27/2017 IASScore:PracticeTests
(C) 1 and 3
(D) All
Selected answer: (Not selected..)
The correct answer is:(B)
Solution:Central government is implementing the Centrally Sponsored Scheme of Rashtriya Uchchatar Shiksha Abhiyan (RUSA) for providing nancial support to
proposals contained in State Higher Education Plan (SHEP) approved by theState Higher Education Councils (SHECs) in order to achieve the objectives of
equity, access and quality.

20. Recently the government of India has approved the Pradhan Mantri Ujjwala Yojana. Consider the following statements related to it:
1.It will provide subsidised Improved Cooking Stoves (ICS) to households using solid bio-fuels.
2.The beneciaries will be identied in consultation with the State Governments and the Union Territories.
3.It has been launched by Ministry of Petroleum and Natural Gas for benetting crores of women belonging to the poorest households.
Which of the above statements is/are correct?

(A) 1 and 2
(B) 2 and 3
(C) Only 3
(D) All
Selected answer: (Not selected..)
The correct answer is:(B)
Solution:The Union Cabinet Committee on Economic Aairs (CCEA) has approved Pradhan Mantri Ujjwala Yojana (PMUY) for providing free of cost LPG (cooking
gas) connections to women from BPL Households.

21. Consider the following statements regarding the Pradhan Mantri Ujjwala Yojana:
1.Under this scheme, over 5 Crore BPL women would be provided LPG connections in the next three years.
2.The scheme will be implemented using the money saved in LPG subsidy through the Give-it-Up campaign.
3.Under this connection will be provided in the name of women or girl child in BPL household.
Which of the statements given above are correct?

(A) 1 and 2
(B) 2 and 3
(C) 1 and 3
(D) All
Selected answer: (Not selected..)
The correct answer is:(A)
Solution:India currently has 16.64 Crore active LPG consumers, mostly in Urban and Semi Urban areas. Poor have limited access to Clean fuel (LPG).
UjjwalaYojana is aimed at providing free LPG connections in the name of women in BPL (Below Poverty Line) households across the country. The main
objectives of the scheme are as follows:
Empowering women and protecting their health.
Preventing young children from signicant number of acute respiratory illnesses caused due to indoor air pollution by burning the fossil fuels.
Reducing the number of deaths in India due to unclean cooking fuels, which is about 5 lakh deaths per year.
The scheme will be implemented by the Ministry of Petroleum & Natural Gas.
Consumers are currently entitled to 12 cylinders of 14.2 kg each or 34 bottles of 5 kg each in a year at subsidised rates.

22. Consider the following statements with respect to the Shala Asmita Yojana:
1.It is a government initiative to address public grievances by engaging concerned ocials through digital technology.
2.It is programme initiated by the Ministry of Home Aairs for better implementation of programmes & policies.
Which among the above given statements is/are correct?

(A) Only 1
(B) Only 2
(C) Both
(D) None
Selected answer: (Not selected..)
The correct answer is:(D)
Solution:The Union Ministry of Human Resource Development (HRD) has decided to launch ASMITA, a student tracking system programme. ASMITA is acronym for
All School Monitoring Individual Tracing Analysis and shall be launched under Shala Asmita Yojana (SAY). Key facts SAY aims to track the educational
journey of school students from Class I to Class XII across the 15 lakhs private and government schools in the country. ASMITA will be an online database
which will carry information of student attendance and enrolment, learning outcomes, mid-day meal service and infrastructural facilities among others.

23. Recently, World Bank has extended $50 million credit for the government's Nai Manzil scheme. Which of the following statements are correct about
'Nai Manzil Scheme'?
1.The scheme aims to provide benet to minority people who are school-dropouts or educated in the community education institutions like Madarsas, by
providing them an integrated input of formal education.
2.The scheme is administered by Ministry of Human Resource and Development.
3.Minimum 30% seats are reserved for minority girls under this scheme.
Codes:

(A) 1 and 3
(B) 1 and 2
(C) 2 and 3
(D) 1, 2 and 3
Selected answer: (Not selected..)
The correct answer is:(A)
Solution:Ministry of Minority Aairs is the Nodal body for Nai Manzil Scheme.

24. Consider the following statements about 'National Family Health Survey-4 (NFHS-4)':
1.In addition to the 29 states, NFHS-4 will also include all six UTs for the rst time.
2.The International Institute for Population Sciences (IIPS) will serve as the implementing agency for NFHS-4.
3.NFHS provide updates and evidence of trends in key population, health and nutrition indicators, including HIV prevalence.
Which of the above statements are correct?

(A) 1 and 2
(B) 2 and 3

(C) 1 and 3
(D) All
Selected answer: (Not selected..)
The correct answer is:(D)
Solution:The Ministry of Health and Family Welfare released the results from the rst phase of the National Family Health Survey (NFHS-4), 2015-16. The National
Family Health Survey (NFHS) is a large-scale, multi-round survey conducted in a representative sample of households throughout India. The survey
provides state and national information for India on fertility, infant and child mortality, the practice of family planning, maternal and child health,
reproductive health, nutrition, anaemia, utilization and quality of health and family planning services.

25. Consider following statements about 'Human Development Index (HDI)':


1.It is published by United Nations Development Program (UNDP).
2.It is a composite statistic of life expectancy, standard of living and income per capita indicators.

=133376 4/5
3/27/2017 IASScore:PracticeTests
2.It is a composite statistic of life expectancy, standard of living and income per capita indicators.
3.Inequality-adjusted HDI measures inequality among dierent nations on the above stated parameters.
Which of the above statements is/are incorrect?

(A) Only 1
(B) 2 and 3
(C) Only 2
(D) Only 3
Selected answer: (Not selected..)
The correct answer is:(B)
Solution:It is a composite statistic of life expectancy, Education, and income per capita indicators. The IHDI takes into account not only the average achievements of
a country on health, education and income, but also how those achievements are distributed among its population by "discounting" each dimension's
average value according to its level of inequality.

Information for the solved exam

Started in: March 27, 2017, 12:21 am

Finished in: March 27, 2017, 12:21 am

Marks Obtained: 0

Right Ques.: 0

Wrong Ques.: 0

Unattempted Ques.: 25

Try for a better result? (index.php?section=user&page=continue_exam&id=133376&renew_exam=yes)

Rank List (index.php?section=user&page=ranking&id=157)

=133376 5/5
3/27/2017 IASScore:PracticeTests

Socio - Economic Development-16

1. Pandit Madan Mohan Malviya National Mission for Teachers Training will focus on which of the following areas?
1. Teaching methodology development.
2. Research infrastructure in higher education.
3. Develop professional cadre of teachers.
4. Changes in Curriculum design.
5. Technology requirements for bringing inclusiveness.
Codes:

(A) 1, 3, 4 and 5
(B) 1, 2, 4 and 5
(C) 1, 2, 3 and 4
(D) 1, 3 and 4
Selected answer: (Not selected..)
The correct answer is:(D)
Solution:An umbrella scheme to create synergies among the various ongoing initiatives on Teachers and Teaching was launched.
The Scheme will address all issues related to teachers, teaching, teacher preparation, professional development, curriculum design.
Develop a strong professional cadre of teachers by setting performance standards and creating top class institutional facilities for innovative teaching.
The scheme will also address the need to induct qualied teachers, attracting talent into teaching profession and raising the quality of teaching in schools
and colleges.

2. Which of the following is the main reason behind decline in poverty rate to 12.4 per cent from the 2011-12 estimate of 21 per cent, according to new
data released by World Bank in 2015?

(A) Rural electrication and infrastructure development.


(B) Eective Implementation of MGNREGA.
(C) Employment Generation in Urban Area.
(D) Employment generation in Rural Area.
Selected answer: (Not selected..)
The correct answer is:(A)
Solution:India has reduced its poverty rate to 12.4 per cent from the 2011-12 estimate of 21 per cent, according to new data released by World Bank, which
identied rural electrication as an important driving factor for everything from greater rural spending to schooling for girls. The global poverty line is the
ability to live on $1.90 (Rs 123.5) per day, up from $1.25 (Rs 81). The change was made to reect dierences in cost of living across countries based on
2011 prices-in other words, to adjust for ination and other economic variables.
The World Bank said improved infrastructure, specically rural electrication, has had far-ranging eects, changed earnings, consumption and even
encouraged schooling for girls.

3. Which of the following statements is/are correct with respect to Rotavac, a Rotavirus medicine launched in India?
1. This vaccine will boost eorts to combat infant mortality due to diarrhea and this is the rst of its kind vaccine available globally against Rotavirus.
2. This was developed under the joint collaboration between India and the Israel in the area of medical research.
Codes:

(A) Only 1
(B) Only 2
(C) None
(D) Both
Selected answer: (Not selected..)
The correct answer is:(C)
Solution:This is the third such vaccine available globally against Rotavirus. This is joint collaboration of US and India. It was developed under the public-private
partnership (PPP) model that involved Union Ministry of Science and Technology, institutions of the US Government and NGOs in India supported by the
Bill and Melinda Gates Foundation.

4. Which of the following is the objective of 'Sehat' Initiative?

(A) This is telemedicine initiative by the central government in collaboration with Apollo Hospitals for urban slums in Delhi/NCR on
pilot basis.
(B) This is a initiative to promote sale of generic drugs through collaboration with Health Ministry by setting up 'Jan Aushadhi' Stores.
(C) This is telemedicine initiative by the central government in collaboration with Apollo Hospitals for rural areas.
(D) This is telemedicine initiative by the central government in collaboration with Medanta Hospitals for urban slums in Delhi/NCR
on pilot basis.
Selected answer: (Not selected..)
The correct answer is:(C)
Solution:Government of India launched a telemedicine initiative 'Sehat' for providing healthcare facilities in rural areas. It is collaboration of Apollo Hospitals and
GoI. The initiative was launched by Communication and IT Ministry and Health Ministry. Under this facility people can consult doctors through video link
and also order generic drugs. It will also help in providing quality and aordable healthcare in rural areas.

5. Consider the following statements about Udaan scheme launched by Ministry of Human Resource Development:
1. The Scheme is to promote the admission of girl students in primary education.
2. The aim is to address the teaching gap by providing free online supplementary lessons in form of videos, practice testa etc. science subjects.

3. It seeks to enhance the enrolment of girl students in prestigious technical education institutions through incentives & academic support.
Which of the above statements is/are correct?

(A) 1 and 2
(B) Only 1
(C) 2 and 3
(D) All
Selected answer: (Not selected..)
The correct answer is:(C)
Solution:The Scheme is dedicated to the development of girl child education, so as to promote the admission of girl students.
951 students have been selected by the CBSE.
The aim is to address the teaching gap between school education and engineering entrance examinations.
It seeks to enhance the enrolment of girl students in prestigious technical education institutions through incentives & academic support.
It oers free online supplementary lessons in physics, chemistry and maths specially prepared in the form of videos, text and practice tests on a tablet.

6. Which of the following statements correctly denes the term Maternal Mortality Rate (MMR) in India?

(A) It is the annual number of female deaths per 100,000 live births from any cause related to or aggravated by pregnancy or its
management.
(B) It is the annual number of female deaths per 1000 live births from any cause related to or aggravated by pregnancy or its
management.
(C) It is the annual number of female deaths per 100,000 live births after 42 days of termination of pregnancy or accidental or
=133378 1/6
3/27/2017 IASScore:PracticeTests
(C) It is the annual number of female deaths per 100,000 live births after 42 days of termination of pregnancy or accidental or
incidental causes.
(D) It is the annual number of deaths of pregnant females per 100,000 live births.
Selected answer: (Not selected..)
The correct answer is:(A)
Solution:.

7. Which of the following statements is/are incorrect with respect to MPI (Multidimensional Poverty Index)?
1. The Index had been developed in 2010 by the World Bank.
2. The index uses the same three dimensions as the Human Development Index: health, education, and standard of living.
3. The dimensions have been divided in dierent indicators where each Indicator and dimension have been assigned dierent weightage according to
their importance in livelihood maintenance.
Codes:

(A) 1 and 3
(B) 2 and 3
(C) 1 and 2
(D) All
Selected answer: (Not selected..)
The correct answer is:(A)
Solution:This was developed in 2010 by the Oxford Poverty & Human Development Initiative (OPHI) and the United Nations Development Programme. Each
dimension and each indicator within a dimension is equally weighted.

8. What are the challenges faced by Mid Day Meal Programme?


1. Poor quality of food.
2. Exclusion of out of school children.
3. Inadequate arrangement for cooking and serving.
4. Issues related to additional work for school teachers.
Codes:

(A) Only 1
(B) 1 and 2
(C) 2, 3 and 4
(D) All
Selected answer: (Not selected..)
The correct answer is:(D)
Solution:.

9. According to the NSSO report which of the following statements regarding Rural & Urban food consumption pattern is/are correct?
1. Rural people consume more calories than their urban counterpart.
2. Urban people consume more protein than the rural counterpart.
3. An average rural person consumes more fat than the Urban counterpart.
Codes:

(A) Only 1
(B) 1 and 2
(C) Only 3
(D) All
Selected answer: (Not selected..)
The correct answer is:(A)

Solution:Year 2010
Calories (kcal)-Rural 2147
Calories (kcal)-Urban 2123
Protein-Rural 59.3
Protein-Urban 58.8
Fat (Rural 43.1
Fat-urban 53.0

10. Which of the following health interventions have not been proposed in the Union Budget 2016-17?

(A) LPG connection in the name of women members to BPL families over the next three years.
(B) Cess on cars to reduce pollution in urban areas.
(C) Increased allocation for Swatchch Bharat Abhiyan and National Rural Drinking Water Programme.
(D) The public expenditure on health has been increased to 1.3%.
Selected answer: (Not selected..)
The correct answer is:(D)
Solution:.

11. Consider the following statements related to TB:


1. Tuberculosis (TB) is an infectious disease caused by a Bacterium, Mycobacterium tuberculosis.
2. HIV-infected people are at increased risk of getting infected by TB.
3. MDRTB refers to strains of the bacterium which are proven in a laboratory to be resistant to the anti-TB drugs.
Which of the above statements are correct?

(A) 1 and 3
(B) 1 and 2
(C) 2 and 3
(D) All
Selected answer: (Not selected..)
The correct answer is:(D)
Solution:.

12. According to WHO, which of the following parameters lead to food security in long term?
1. Availability of food
2. Accessibility of food
3. Absorption of food
4. Aordability of food
Codes:

(A) 1 and 2
(B) 1, 3 and 4
(C) 2 and 3
(D) All
=133378 2/6
3/27/2017 IASScore:PracticeTests
(D) All
Selected answer: (Not selected..)
The correct answer is:(D)
Solution:.

13. With the recommendation of the NITI AAYOG rationalization of Centrally Sponsored Scheme related to education have been done. Which of the
following schemes have been included under National Education Mission?
1. Rashtriya Uchchatar Shiksha Abhiyan.
2. Rashtriya Madhyamik Shiksha Abhiyan.
3. Navodaya Vidyalaya.
4. Support For Educational Development Including Teachers Training.
Codes:

(A) 1 and 2
(B) 1, 2 and 3
(C) 1, 2 and 4
(D) All
Selected answer: (Not selected..)
The correct answer is:(C)
Solution:Sarva Shiksha Abhiyan, Rashtriya Uchchatar Shiksha Abhiyan, Rashtriya Madhyamik Shiksha Abhiyan, support for educational development including
teachers training and Adult education have been placed under National Education Mission.

14. Consider the following statements regarding constituents of Indira Gandhi Matritva Sahyog Yojana (IGMSY):
1. It is a conditional cash transfer scheme for pregnant and lactating women.
2. It attempts to partly compensate for wage loss to pregnant and lactating women only prior to the delivery of child.
3. It covers both the organized and unorganized sector employees.
4. It encourages the women to follow (optimal) nutrition and feeding practices.
Which of the above statements are correct?

(A) 1, 2 and 4
(B) 1 and 4

(C) 3 and 4
(D) All
Selected answer: (Not selected..)
The correct answer is:(B)
Solution:IGMSY is a conditional cash transfer scheme for pregnant and lactating women launched. It attempts to partly compensate for wage loss to pregnant and
lactating women both prior and after the delivery of child. It covers only the unorganized sector employees. All organized sector employees are excluded
from the scheme as the entitled for paid maternity leave.

15. Which of the following reasons are responsible for the increment in the incidence of feminization of poverty in the developing nations?
1. The increasing prevalence of female-headed households due to rural-urban migration.
2. Gender biases often deprive women of opportunities to independently pursue education or careers.
3. Wage discrimination and deprivation of decision making power in women.
Codes:

(A) 2 and 3
(B) Only 3
(C) 1 and 3
(D) All
Selected answer: (Not selected..)
The correct answer is:(D)
Solution:.

16. What is the objective of the Swami Vivekananda Scholarship launched by UGC recently?

(A) To provide scholarship to Single Girl Child for research in Social Sciences.
(B) To provide scholarships to students from North East Region whose parental income is below Rs. 4.5 lakh per annum for studying
at under graduate level in Colleges/ Universities of the country.
(C) To provide scholarships to students from left-wing extremism region whose parental income is below Rs. 4.5 lakh per annum for
studying at under graduate level in Colleges/ Universities of the country.
(D) To provide scholarships to students from left-wing extremism region for preparing for medical and engineering entrance
examinations.
Selected answer: (Not selected..)
The correct answer is:(A)
Solution:.

17. Which of the following is/are true about the latest World bank poverty line calculation?
1. The poverty line has been increased to $1.90 from $ 1.25 using 2011 prices.
2. It uses Uniform Recall Period to estimate poverty.
3. The World Bank projects that the 9.6% of the global population is living in poverty.
Codes:

(A) Only 1
(B) 2 and 3
(C) 1 and 3
(D) All
Selected answer: (Not selected..)
The correct answer is:(C)
Solution:It has raised its ticket size to measure poverty to $1.90 from the older $1.25 since the price of goods and services across the world has risen
Modied mixed reference period, or MMRP.(METHOD ADOPTED BY WB)
In this method, for some food items, instead of a 30-day recall, only a 7-day recall is collected. Also, for some low-frequency items, instead of a 30-day
recall, a 1-year recall is collected.
This is believed to provide a more accurate reection of consumption expenditures. When such data was collected, consumption expenditures for
people in both urban and rural areas went up by 10 per cent to 12 per cent.
This happened essentially because people could better recall their food expenditure over a shorter, 7-day period than what they might have done over
the longer 30-day period. The higher expenditures, combined with the high population density around the poverty line, essentially meant that the poverty
rate for India (for 2011-12) came down sharply.

18. According to the World Employment and Social Outlook: Trends 2016, which of the following short term measures will help in improving the quantity
and quality of jobs and tackling income inequality?
1. Changes in National macroeconomic policies.
2. Financing necessary infrastructure projects.
3. Strengthening labour market institutions.
4. Achieving the Sustainable Development Goals (SDGs).
5. Financial reforms with focus on Banking Sector.
Codes:
=133378 3/6
3/27/2017 IASScore:PracticeTests
Codes:

(A) 1 and 2
(B) 1, 3, 4 and 5
(C) 1, 2 and 5
(D) 1, 2, 3 and 4
Selected answer: (Not selected..)
The correct answer is:(A)

Solution:Source: World Employment and Social Outlook: Trends 2016


In the short term, there is room for manoeuvre in macroeconomic policies in many countries. In light of historically low interest rates, countries could
nance necessary infrastructure projects, which would have important multiplier eects without imposing a big burden on the public purse.
The need to address these long-term trends adds urgency to the calls by the ILO for a shift in economic and employment policies. It is particularly
important to strengthen labour market institutions and ensure that social protection systems are well designed, in order to prevent further increases in
long-term unemployment, underemployment and working poverty. A rebalancing in reform eorts is also needed. In particular, nancial reforms need to
ensure that banks perform their role of channelling resources into the real economy and into investment for sustainable enterprise expansion and job
creation.
In the medium to long term, achieving the sustainable development goals (SDGs), particularly decent and productive employment for all, will yield
signicant social dividends while contributing to strengthening and rebalancing the global economy.

19. Which of the following statements is/are correct about IMPRINT?


1. It is a pan-IIT and IISc joint collaboration to develop a blueprint for improving the accreditation system of higher education.
2. It will ensure that higher education institutes are involved in research activities relevant to the society.
Codes:

(A) Only 1
(B) Only 2
(C) Both
(D) None
Selected answer: (Not selected..)
The correct answer is:(B)
Solution: The initiative, 'IMPRINT India', is a pan-IIT and IISc joint collaboration to develop a blueprint for research of immediate relevance to society requiring
innovation, direct scientic research into identied areas, ensure higher funding support for research into these areas and measure outcomes of the
research eorts with reference to the impact on the standard of living in rural/urban areas.
IMPRINT India will focus on 10 themes and each theme will be coordinated by one IIT/IISc. For instance, IIT-Kharagpur will work on the themes of health
care, computer science and information and communication technology, while IIT-Kanpur will work on advance material, water resources and river
systems.

20. Which of the following is/are not the objective of setting up of Atal Innovation Mission?
1. To create awareness and provide knowledge inputs in creating innovation challenges and funding mechanism to government.
2. To act as a Techno-Financial, Incubation and Facilitation Programme to support all aspects of start-up businesses.
3. To create an umbrella structure to oversee innovation eco-system of the country.
Codes:

(A) 1 and 2
(B) 1 and 3
(C) Only 2
(D) All
Selected answer: (Not selected..)
The correct answer is:(C)
Solution:ATAL Innovation Mission objectives:
To create an umbrella structure to oversee innovation eco-system of the country;
To provide platform and collaboration opportunities for dierent stakeholders; c.
To study and suggest best and novel practices to be adopted by dierent stakeholders in the innovation chain;
To provide policy inputs to NITI Aayog and various Government Departments and Organizations.
To create awareness and provide knowledge inputs in creating innovation challenges and funding mechanism to government; and,
To develop new programmes and policies for fostering innovation in dierent sectors of economy.

21. Which of the following are recommendations of Shanta Kumar committee on reforming PDS?
1. It has proposed for universal coverage under National Food Security Act.
2. It has recommended increment in monthly quantum of ration from 5kg to 7 kg.
3. It has recommended to link prices of grain sold to priority household to 50% of MSP.
Codes:

(A) Only 1
(B) 2 and 3
(C) All
(D) None
Selected answer: (Not selected..)
The correct answer is:(B)
Solution:The recommendations are
Reduce coverage from 67% to 40% of population
Raise monthly quantum of ration (5-------------> 7kg)
Link prices of grain sold through TPDS to 50% of MSP, deregulate fertiliser sector and give direct cash subsidy.

22. According to the TeamLease Employment Outlook Report 2016 which of the following factors are responsible for driving employment outlook in India?
1. High rate of Economic growth has been forecasted which may bring employment opportunities.
2. Consumer maturity has increased with Changing Consumer Attitude thus demand for new services has increased.
3. Engineering and Technology Based talent demand have increased by e-commerce and product companies.
4. Infrastructure projects are being accelerated.
Codes:

(A) 2, 3 and 4
(B) 1 and 4
(C) 1, 3 and 4

(D) All
Selected answer: (Not selected..)
The correct answer is:(D)
Solution:What factors drive employment?
1. Economic growth forecast at 8.1%:
The United Nations reports that the Indian economy is set to lead the way in Asia with a stupendous 8.1% growth rate in 2015 and follow it up with an
8.2% in 2016.
2. Consumer maturity moves to the next level:
Mobile-internet is proving to transform the consumer attitude and preference way more than organized retail did, and in way shorter a time frame
3. Tech talent lured by ecommerce and product companies:
Top Indian engineering campuses are abuzz with magnanimous oers from startups, multinational companies and homegrown technology giants.
4. Infrastructure projects pace likely to come of age:
The Government is pushing through and accelerating infrastructural projects, an action meant to be a booster dose to the 'Make in India' initiative.

=133378 4/6
3/27/2017 IASScore:PracticeTests

23. Which of the following initiative of the Ministry of Health & Family Welfare will focus on increasing adolescents' awareness about the adverse eects
and consequences of substance misuse?

(A) Kayakalp
(B) Kilkari
(C) Rashtriya Kishor Swasthya Karyakram
(D) Daksh
Selected answer: (Not selected..)
The correct answer is:(C)
Solution:.

24. ILO report 2015 states that falling participation rates are due to both cyclical and structural factors. Which of the following are related to structural
factors?
1. Population ageing.
2. Recession
3. Increasing years spent in education.
4. Slow recovery in the economic cycle
5. Slower growth in the working-age population.
Select the correct answer using the codes given below:
Codes:

(A) 1, 2, 3 and 5
(B) 1, 3 and 5
(C) 2, 3 and 5
(D) 2, 3 and 4
Selected answer: (Not selected..)
The correct answer is:(B)
Solution:As highlighted in the 2015 edition of this report, falling participation rates are due to both cyclical and structural factors.
When jobs are scarce due to recession or slow recovery in the economic cycle, some jobseekers become discouraged and drop out of the labour market.
In terms of structural factors, population ageing and increasing years spent in education in many countries result in shrinking or slower growth in the
working-age population.
These two eects need to be dierentiated to provide a clearer understanding of the future path of labour force participation and to design and
implement an eective set of policy interventions.

25. Which of the following statement correctly denes the term Optimum Population?

(A) Optimum population is dened as the size of population enabling maximum per capita output under given economic and
technological conditions.
(B) Optimum population is dened as the reduction in the decadal growth of population which enables maximum per capita output
under given economic and technological conditions.
(C) Optimum population is dened as the size of population enabling maximum per capita output due to rapid development of non-
agricultural sector.
(D) Optimum population is dened as the population growth that exceeds the existing resource base.
Selected answer: (Not selected..)
The correct answer is:(A)
Solution:Optimum population is dened as the size of population enabling maximum per capita output under given economic and technological conditions.

Information for the solved exam

Started in: March 27, 2017, 12:23 am

Finished in: March 27, 2017, 12:23 am

Marks Obtained: 0

Right Ques.: 0

Wrong Ques.: 0

Unattempted Ques.: 25

Try for a better result? (index.php?section=user&page=continue_exam&id=133378&renew_exam=yes)

Rank List (index.php?section=user&page=ranking&id=148)

=133378 5/6
3/27/2017 IASScore:PracticeTests

=133378 6/6
3/27/2017 IASScore:PracticeTests

Socio - Economic Development-15

1. Which of the following materials are used for production of Ethanol for the Ethanol Blending Programme in India apart from sugarcane molasses?
1. Saw mill and paper mill discards
2. Elephant grass
3. Corn stover
Codes:

(A) 1 and 2
(B) 2 and 3
(C) Only 3
(D) All
Selected answer: (Not selected..)
The correct answer is:(D)
Solution:Ethanol produced from other non-food feedstocks besides molasses, like cellulosic and ligno cellulosic materials such as Saw Mill and paper mill discards,
elephant grass, corn stover, etc. including petrochemical route, has also been allowed to be procured subject to meeting the relevant Bureau of Indian
Standards (BIS) specications.

2. Consider the following statements with respect to the budgetary provisions related to the education sector:
1. The share of education in the total Union Budget has increased whereas for primary education it has been decreased.
2. Entrepreneurship education and training will be provided in government colleges, ITI s.
3. There is substantial cut in teacher training and adult education programmes.
Which of the above statements is/are correct?

(A) 1 and 2
(B) Only 2
(C) 2 and 3
(D) Only 1
Selected answer: (Not selected..)
The correct answer is:(C)
Solution:The share of education in the total Union Budget has declined from 3.8% to 3.7%.

3. Which of the following is/are the objectives of country Strategic Plan signed by UN World Food Programme and Government of India?
1. Ensure access to safe, nutritious and sucient food for all people all year round.
2. Ensure ending malnutrition according to internationally agreed targets for children under 5.
3. Addressing nutritional needs of adolescent girls, pregnant and lactating women.
Codes:

(A) 1 and 2
(B) Only 2
(C) 2 and 3
(D) All
Selected answer: (Not selected..)
The correct answer is:(D)
Solution:The main focus of WFP is on supporting the Government of India to make signicant and measurable progress in order to contribute to the following two
objectives:- (i) ensuring access to safe, nutritious and sucient food for all people all year round. (ii) ensuring ending malnutrition according to
internationally agreed targets, with a focus on stunting and wasting for children under 5 years of age, and addressing the nutritional needs of adolescent
girls, pregnant and lactating women, and older persons.

4. Which of the following are part of "9 point action plan" to reinvent TPDS?
1. States to undertake campaign for review of BPL/ Antodaya list.
2. Strict action against guilty to ensure leakage free distribution.
3. BPL/Antodaya list to be displayed on all FPS.
4. Training of members of FPS level vigilance committee.
5. Computerization of TPDS.
Codes:

(A) 1, 3 and 5
(B) 2, 3 and 5
(C) 1, 3, 4 and 5
(D) All
Selected answer: (Not selected..)
The correct answer is:(D)
Solution:Nine point Action Plan are as under:
States should undertake a campaign for review of BPL/AAY list to eliminate bogus ration cards.
Strict action should be taken against the guilty to ensure leakage free distribution of foodgrains.
For sake of transparency, involvement of elected Panchayati Raj Institution (PRI) members in distribution of foodgrains; licenses be given to Self Help
Groups (SHGs), gram panchayats, cooperatives etc.
BPL and AAY lists should be displayed on all FPSs.
District-wise and FPS-wise allocations of foodgrains should be put up on websites and other prominent places for public scrutiny.
Wherever possible, door-step delivery of foodgrains should be ensured by States, instead of letting private transporter/ wholesalers to transport goods.
Timely availability of foodgrains at FPS level and xed dates of distribution to ration card holders should be ensured.
Training of members of FPS level Vigilance Committee should be ensured
Computerization of TPDS operations should be undertaken.

5. Consider the following statements about the ethical poverty line (EPL):
1. The ethical poverty line (EPL) is derived from globally standardised and ethically justiable well-being outcomes for suciently existing data.
2. The EPL overcomes the inherent problem of all income poverty lines as it provides a morally defensible basis for setting the line.
3. The EPL reveals that the price has to be paid for accepting a moral duty to remove widespread poverty by developed nations.
Which of the above statements are correct?

(A) 1 and 2
(B) 2 and 3
(C) 1 and 3
(D) All
Selected answer: (Not selected..)
The correct answer is:(C)
Solution:The ethical poverty line (EPL) proposed is derived from globally standardised and ethically justiable wellbeing outcomes for which sucient data already
exist in the World Development Indicators (WDI). The EPL does not overcome the inherent problem of all income poverty lines, namely that they
oversimplify and reduce the complexity of global poverty to a mono dimensional monetary measure. However, the EPL at least provides a morally
defensible basis for setting the line. The EPL, therefore, reveals that the price to be paid for accepting a moral duty to remove poverty today is one that
would cut deeply into the pockets of developed world populations.

6. Which of the following statements is/are correct about Rashtriya Arogya Nidhi?
1. It aims to provide nancial assistance to patients, living below poverty line who are suering from major life threatening diseases.
2. The patient is elligible to receive medical treatment at any of the super specialty hospitals/institutes or other government hospitals.
=133380 1/6
3/27/2017 IASScore:PracticeTests
2. The patient is elligible to receive medical treatment at any of the super specialty hospitals/institutes or other government hospitals.
3. Financial assistance is released in the form of "two times grant to the person after the treatment through Direct Benet Transfer.
Codes:

(A) Only 1
(B) 1 and 3
(C) 1 and 2
(D) All
Selected answer: (Not selected..)
The correct answer is:(C)
Solution:Financial assistance is released in the form of "one times grant" to the Medical Superintendent of the hospital in which the treatment is being received.

7. The Inequality Adjusted HDI, stands for which of the following?

(A) It takes the average of inequality within and among countries based on Gini-Coecient.
(B) It is an index to measure relative deprivation.
(C) It is actual level of human development accounting for inequality.
(D) It can be viewed as potential human development which can be achieved.
Selected answer: (Not selected..)
The correct answer is:(C)
Solution:The 2010 Human Development Report introduced an Inequality-adjusted Human Development Index (IHDI). While the simple HDI remains useful, it stated
that "the IHDI is the actual level of human development (accounting for inequality)," and "the HDI can be viewed as an index of 'potential' human
development (or the maximum IHDI that could be achieved if there were no inequality).

8. Consider the following statements about Women at Work Trends 2016 by International Labour Organization 2016:
1. Inequality between women and men persists in global labour markets, in respect of opportunities, treatment and outcomes.
2. The chances for women to participate in the labour market remain lower than those for men.
3. Between 1995 and 2015, the global female labour force participation rate has increased signicantly due to globalisation.
Which of the above statements are correct?

(A) 1 and 2
(B) 2 and 3
(C) 1 and 3
(D) All
Selected answer: (Not selected..)
The correct answer is:(A)
Solution:Between 1995 and 2015, the global female labour force participation rate decreased from 52.4 to 49.6 per cent. Worldwide, the chances for women to
participate in the labour market remain almost 27 percentage points lower than those for men.

9. Consider the following statements about Poverty and Well-being:


1. Economic growth is the best indicator of poverty reduction as it depicts the standard of living of an individual.
2. Poverty gap index measures incidence of poverty and depth of poverty.
3. The parameters like Electricity, ooring, drinking water, sanitation, cooking fuel and assets determine the level of multi-dimensional poverty.
Which of the above statements are correct?

(A) 2 and 3
(B) 1 and 3
(C) 1 and 2
(D) All
Selected answer: (Not selected..)
The correct answer is:(A)

Solution:Economic growth is not a good indicator of poverty reduction. Poverty means low levels of wellbeing, not just low income and it measures must relate
closely to people's lives. Poverty is failure of socially determined basic capabilities, but only when caused by lack of resources. A new international poverty
line is needed. It should reect the cost of basic human needs.

10. Which of the following is/are challenges to PDS?


1. Exclusion of APL has aected economic viability of FPS.
2. Large errors of exclusion and inclusion results in mis-targeting.
3. Regional disparities in the distribution of PDS benets.
4. Failure to translate into macro level food self suciency.
Codes:

(A) 1 and 2
(B) 1, 2 and 3
(C) 1, 2 and 4
(D) All
Selected answer: (Not selected..)
The correct answer is:(D)
Solution:.

11. In order to ensure that all eligible farmers are provided with hassle free and timely credit for their agricultural operation, Kisan Credit Card (KCC)
Scheme has been launched. Which of the following are the beneciaries to it?
1. Marginal farmers
2. Share Croppers
3. Oral lessee and tenant farmers
4. Farm labours
Codes:

(A) 1, 2 and 3
(B) 2, 3 and 4
(C) 1, 3 and 4
(D) All
Selected answer: (Not selected..)
The correct answer is:(A)
Solution:In order to ensure that all eligible farmers are provided with hassle free and timely credit for their agricultural operation, Kisan Credit Card (KCC) Scheme
was introduced in 1998-99. Marginal farmers, share croppers, oral lessee and tenant farmers are eligible to be covered under the scheme. The main
objectives of the scheme are to meet the short term credit requirements for cultivation of crops, post harvest expenses, produce marketing loan,
consumption requirements of farmer household, working capital for maintenance of farm assets and activities allied to agriculture like dairy animals,
inland shery, etc., investment credit requirement for agriculture and allied activities like pump sets, sprayers, dairy animals, etc.

12. According to the Rangarajan formula for estimation of poverty place the following states in the hierarchial form with respect to the percentage of
poverty.
1. Chattisgarh
2. Odisha
3. Kerala
4. Lakshadweep

=133380 2/6
3/27/2017 IASScore:PracticeTests
4. Lakshadweep
Codes:

(A) 1-2-3-4
(B) 2-1-3-4
(C) 2-1-4-3
(D) 1-2-4-3
Selected answer: (Not selected..)
The correct answer is:(A)
Solution:Chatisgarh - 47.9%
Orissa - 45.9
Keral -11.3%
Lakshadweep - 6.5%

13. Which of the following statements dene the benets of distribution of food coupons over cash transfers for maintenance of food security?
1. Food coupons insulate beneciaries from ination and price volatility as PDS shops supply foodgrains at subsidised rate.
2. Household is given the freedom to choose where it buys food.
3. Increases incentive for competitive prices and assured quality of foodgrain among PDS stores.
Codes:

(A) 1 and 2
(B) Only 1
(C) 2 and 3
(D) All
Selected answer: (Not selected..)
The correct answer is:(C)
Solution:Food coupons are not indexed for ination; may expose recipients to ination.

14. Which of the following statement correctly dences the term Shaala Darpan?

(A) It was launched to provide mobile access to parents of students of Government and Government aided schools.
(B) It was launched to provide direct internet access to Ministry of Human Resource Development of Government and Government
aided schools.
(C) It was launched to provide mobile access to Teachers of Government and Government aided schools about Monthly Progress of
Students.
(D) It was launched to provide ICT based platform to provide Fund Disbursal Mechanism of Government and Government aided
schools directly to Ministry of Human Resource Development.
Selected answer: (Not selected..)
The correct answer is:(A)
Solution:ShaalaDarpan - A focused approach to give the right impetus to all the States in deploying ICT
ShaalaDarpan - an ICT programme of Ministry of Human Resource Development was launched to provide mobile access to parents of students of
Government and Government aided schools.
Using ShaalaDarpan parents can view updates on their child's progress. They can view records of attendance, assignments and achievements of their
child.

15. Which of the following can be initiated to improve agriculture to reduce poverty in Rural areas?
1. Rationalisation of Subsidy.
2. Organic farming.
3. Implementation of Pradhan Mantri Fasal Bima Yojana.
4. Technology adoption.
Codes:

(A) 1 and 4
(B) 1, 2 and 3
(C) 2, 3 and 4
(D) All
Selected answer: (Not selected..)
The correct answer is:(C)
Solution:Organic farming can help in to Lower input and optimum output (yield), increases the income and reduces the dependence of farmers on money lenders.
Technology adoption: Mechanization of agriculture, promotion of modern technologies like Vermicomposting, Fertigaiton, plastic mulches, polyhouse
cultivation - These should be promoted through provision of subsidies, grants and credit facilities. Successful implementation of PM fasal bima yojana:
Protect the farmers from various risks.

16. Which of the following components constitute the vector of poverty in calculating Rural and Urban Poverty?
1. Power and politics.
2. Class-prejudice.
3. Organizational participation in dierent associations.
4. Religiosity.
Select the correct answer using the codes given below:

(A) 1, 2 and 3
(B) 2, 3 and 4
(C) 1, 2 and 4
(D) All
Selected answer: (Not selected..)
The correct answer is:(D)
Solution:The components that constitute the vector of poverty have been chosen in term of unsatisfaction and deprivation.
Contents of each of the components that constitute the vector of poverty
(i) Class-prejudice: To nd out the poor's attitude towards the non-poor in terms of education, behaviour, intelligence and other facilities.
(ii) Religiosity: To nd out the religiosity among the poor in terms of ritualistic, intellectual belief and good work dimensions.
(iii) Political aliation: to nd out the aliation of the poor with political parties and their mode of political participation.
(iv) Organizational participation: To nd out the poor's organizational participation in terms of labour union, and other associations, as well as their
interest in national political aairs.
Other components are food, shelter, health, education, assets, etc.

17. Which of the following government initiatives comes under the National Mission on Agricultural Extension and Technology?
1. Media support to farmers through radio, doordharshan, etc.
2. Agri-clinics.
3. Information dissemination through Agri-Fairs.
4. Paramparagat Krishi Vikas Yojana.
Codes:

(A) 1, 2 and 3
(B) Only 4
(C) 1 and 3
(D) All
Selected answer: (Not selected..)
The correct answer is:(A)
=133380 3/6
3/27/2017 IASScore:PracticeTests
The correct answer is:(A)
Solution:.

18. Match the following:


I. Stunting 1) It is Measure of Chronic Undernutrition.
II. Wasting 2) Composite of both acute and chronic Undernutrition.
III. Underweight 3) It is measure of acute Undernutrition.
Codes:

(A) I- 1, II- 2, III- 3


(B) I- 2, II- 3, III- 1
(C) I- 1, II- 3, III- 2
(D) I- 3, II- 1, III- 2
Selected answer: (Not selected..)
The correct answer is:(C)
Solution:.

19. Consider the following statements about Unnat Bharat Abhiyaan:


1. Under this IIT and NIT will adopt villages and develop appropriate rural technologies for sustainable development through peoples participation.
2. It aims at Skill Development and enhancing the economic prospects of those engaged in traditional crafts and arts as a means of livelihood.
3. It will act as a platform that will connect institutes of higher education with local communities through e-education.
Which of the above statements is/are correct?

(A) 1 and 3
(B) Only 1
(C) 2 and 3
(D) All
Selected answer: (Not selected..)
The correct answer is:(B)
Solution:Under this:
IIT, IISER, NIT to adopt villages and develop appropriate rural technologies for sustainable development through peoples' participation.
The Abhiyan will enable processes that connect institutes of higher education with local communities. Focus on water management, organic farming,
renewable energy, frugal technology, infrastructure, and livelihood.

20. Consider the following statements related to the 'Quetelet index':


1. The Quetelet index is a value derived from the mass (weight) and height of an individual.
2. It quantify the amount of tissue mass in an individual.
3. It is a measure of malnutrition in the adults only as in children it is calculated in terms of wasting and stunting.
Which of the above statements are correct?

(A) 1 and 2
(B) 2 and 3
(C) 1 and 3
(D) All
Selected answer: (Not selected..)
The correct answer is:(D)
Solution:The most useful measure of malnutrition in adults is the body mass index (BMI). The body mass index (BMI) or Quetelet index is a value derived from the
mass (weight) and height of an individual. The BMI is an attempt to quantify the amount of tissue mass (muscle, fat, and bone) in an individual, and then
categorize that person as underweight, normal weight, overweight, or obese based on that value.

21. Which of the following government initiatives will ensure food security in India?
1. National Food Security Mission.
2. Krishonnati Yojana.
3. Bringing Green Revolution to Eastern India.
4. Digitization of PDS.
Codes:

(A) 1 and 3
(B) 1 and 2
(C) 2, 3 and 4
(D) All
Selected answer: (Not selected..)
The correct answer is:(D)
Solution:.

22. Which of the following statement is correct with respect to Mosquirix or RTS,S vaccine?

(A) This is world's rst Malarial vaccine approved by European Medicines Agency.
(B) This is rst licensed human vaccine against parasitic disease, Zika in South America.
(C) This vaccine was developed and marketed by India and America for AIDS patient in Africa.
(D) This is the world largest selling vaccine for malaria & chikungunya.
Selected answer: (Not selected..)
The correct answer is:(A)
Solution:This is world's rst Malarial vaccine approved by European medicines Agency. This vaccine was licensed for use on babies aged between 6 weeks and 17
months in Africa, which is at the highest risk of the mosquito-borne disease. The vaccine developed by British drugmaker GlaxoSmithKline in partnership
with the PATH Malaria Vaccine Initiative. The vaccine was approved despite the dampened results released in 2011 and 2012. The results showed that it
only reduced episodes of malaria in babies aged 6-12 weeks by 27 percent, and by around 46 percent in children aged 5-17 months.

23. Which of the following indices are used for calculation of Human Poverty Index for industrialized nations?
1. Deprivation of longevity is measured as a percentage of the individuals with a life expectancy lower than 60 years.
2. Deprivation of knowledge is expressed as a percentage of illiterate adults.
3. Social exclusion is measured in terms of long-term unemployment rate.
Select the correct answer using the codes given below:
Codes:

(A) 1 and 2
(B) Only 2
(C) 1 and 3
(D) All
Selected answer: (Not selected..)
The correct answer is:(D)
Solution:The human poverty index for industrialised countries uses the same dimensions of the previous index, but the variables and reference values are
dierent:
Deprivation of longevity is measured by the percentage of individuals whose life expectancy is below 60 years of age (P1),
Deprivation of knowledge is based on the percentage of adults functionally illiterate according to the OECD denition (P2),

=133380 4/6
3/27/2017 IASScore:PracticeTests
Deprivation of knowledge is based on the percentage of adults functionally illiterate according to the OECD denition (P2),
Deprivation of decent life standards (P3) is the percentage of the population living below the poverty level, as dened according to the criteria of the
International Standard of Poverty Line, thus being equal to 50% of the per capita average national income.
HPI-2 also considers a fourth dimension, social exclusion, measured with the long-term unemployment rate

24. Which of the following is/are correct with respect to Crimean-Congo Haemorrhagic fever (CCHF)?
1. CCHF is mostly found among the hot and humid climate of Ganga plain and Southern States of India.
2. It is transmitted to people from tick bites or with close contact to livestock animals.
3. CCHF is endemic in countries south of the 50th parallel North.
Codes:

(A) Only 1
(B) Only 2
(C) 2 and 3
(D) All
Selected answer: (Not selected..)
The correct answer is:(C)
Solution:National Institute of Virology Pune- conrmed several outbreaks of CCHF in Rajsthan, Gujarat and Uttar Pradesh in 2014 and 2015.

25. Which of the following initiatives launched by GOI in eld of education are incorrectly matched?
1. SWAYAM: Under this scholarships will be provided to dierently-abled students to pursue technical education.
2. Ishan Vikas: Under this selected students from the school and college levels from the North-Eastern states will be proposed to the opportunities in the
IITs, NITs and IISERs during their vacation periods.
3. Saksham: Under this professors of centrally funded institutions will oer online courses to citizens free of cost.
Codes:

(A) Only 1
(B) 2 and 3
(C) 1 and 3
(D) Only 2
Selected answer: (Not selected..)
The correct answer is:(C)
Solution:SWAYAM: Under this professors of centrally funded institutions will oer online courses to citizens free of cost. Saksham: Under this scholarships will be
provided to dierently-abled students to pursue technical education.

Information for the solved exam

Started in: March 27, 2017, 12:24 am

Finished in: March 27, 2017, 12:24 am

Marks Obtained: 0

Right Ques.: 0

Wrong Ques.: 0

Unattempted Ques.: 25

Try for a better result? (index.php?section=user&page=continue_exam&id=133380&renew_exam=yes)

Rank List (index.php?section=user&page=ranking&id=147)

=133380 5/6
3/27/2017 IASScore:PracticeTests

=133380 6/6
3/27/2017 IASScore:PracticeTests

Socio - Economic Development-14

1. Which of the following statements are correct about Antyodaya Anna Yojana?
1. Only the poorest section of BPL are eligible to get foodgrains at highly subsidized rate.
2. People covered under Antyodaya Anna Yojana are not eligible to avail benets of PDS, meant for BPL.
3. Its a centrally sponsored programme, where center bears all the cost from foodgrain to distribution.
4. The Food Security Act has drastically reduced the quantity of grain allocated to Antodaya families.
Codes:

(A) 1 and 2
(B) 1 and 3
(C) 2, 3 and 4
(D) All
Selected answer: (Not selected..)
The correct answer is:(A)
Solution:In order to make TPDS more focused and targeted towards the poorest section of population, the "Antodaya Anna Yojana" was launched to benet
poorest of poor section within BPL families.
Highly subsidized food grain (Rs 2/ wheat, Rs 3/rice) is provided under this scheme.
Under this scheme state/UT bear the distribution cost including margins to dealers and retailers as well as the transportation cost.
NFSA has continued the Antodaya Anna Yojana.

2. Which of the following statement correctly explain the term "Genetic Superheroes"?

(A) People whose genomes contains only Y chromosomes that would have caused devastating disorders but they are not sick.
(B) These are people who remain healthy even though they were born with genetic mutations.
(C) This is Super Genome which can be use for producing any Human body part.
(D) A genome which can withstand any harsh nuclear or thermal condition without any mutation.
Selected answer: (Not selected..)
The correct answer is:(A)
Solution:These are people who remain healthy even though they were born with genetic mutations that would usually lead to devastating disorders.

3. Consider the following statement/s regarding National Food Security Mission:


1. It aims at additional production of rice, wheat, pulses, coarse cereals and commercial crops.
2. Krishi Karman award is given as a reward to the best performing state in foodgrains production category.
3. Under this cropping system based training is provided.
Which of the above statements is/are correct?

(A) 1 and 2
(B) Only 2
(C) 2 and 3
(D) All
Selected answer: (Not selected..)
The correct answer is:(D)
Solution:.

4. World Health Organisation has declared India free of Maternal and Neonatal Tetanus. Which of the following are correct with respect to Maternal and
Neonatal Tetanus?
1. It usually occurs through infection due to minor outer injury at childhood.
2. It is prevalent in both developed and developing countries.
3. Infants who have not acquired passive immunity because the mother has never been immunised are at more risk.
4. Now it has been reduced to less than one case per 1000 live births across the country.
Codes:

(A) 1 and 2
(B) 3 and 4
(C) 1, 3 and 4
(D) All
Selected answer: (Not selected..)
The correct answer is:(B)
Solution:It usually occurs through infection of the unhealed umbilical stump, particularly when the stump is cut with a non-sterile instrument. Neonatal tetanus
mostly occurs in developing countries, particularly those with the least developed health infrastructure. It is rare in developed countries.

5. Which of the following statements is/are correct with respect to the guidelines issued by Law Commission for Surrogacy in India?
1. The birth certicate of the surrogate child should contain the name of the commissioning parents and Surrogate mother.
2. During Surrogacy, cases of abortions should be governed by the Assisted Reproductive Technology Regulation Act 2010.
3. A surrogacy contract should necessarily take care of life insurance cover for surrogate mother.
Codes:

(A) 2 and 3
(B) Only 3
(C) Only 1

(D) All
Selected answer: (Not selected..)
The correct answer is:(B)
Solution:The following observations had been made by the Law Commission:
Surrogacy arrangement will continue to be governed by contract amongst parties, which will contain all the terms requiring consent of surrogate mother
to bear child, agreement of her husband and other family members for the same, medical procedures of articial insemination, reimbursement of all
reasonable expenses for carrying child to full term, willingness to hand over the child born to the commissioning parent(s), etc. But such an arrangement
should not be for commercial purposes.
A surrogacy arrangement should provide for nancial support for surrogate child in the event of death of the commissioning couple or individual before
delivery of the child, or divorce between the intended parents and subsequent willingness of none to take delivery of the child.
A surrogacy contract should necessarily take care of life insurance cover for surrogate mother.
One of the intended parents should be a donor as well, because the bond of love and aection with a child primarily emanates from biological
relationship. Also, the chances of various kinds of child-abuse, which have been noticed in cases of adoptions, will be reduced. In case the intended parent
is single, he or she should be a donor to be able to have a surrogate child. Otherwise, adoption is the way to have a child which is resorted to if biological
(natural) parents and adoptive parents are dierent.
Legislation itself should recognize a surrogate child to be the legitimate child of the commissioning parent(s) without there being any need for adoption
or even declaration of guardian.
The birth certicate of the surrogate child should contain the name(s) of the commissioning parent(s) only.
Right to privacy of donor as well as surrogate mother should be protected.
Sex-selective surrogacy should be prohibited.
Cases of abortions should be governed by the Medical Termination of Pregnancy Act 1971 only.

6. Which of the following is/are achievements under "end to end Computerization of PDS'?
=133384 1/6
3/27/2017 IASScore:PracticeTests
6. Which of the following is/are achievements under "end to end Computerization of PDS'?
1. Digitization of FPS and godown database.
2. Online allocation of foodgrains.
3. Toll free help-line for TPDS.
Codes:

(A) Only 1
(B) 1 and 2
(C) 2 and 3
(D) All
Selected answer: (Not selected..)
The correct answer is:(D)
Solution:The underlying objectives of the activities under the scheme are:
Digitization of beneciary and other database will enable correct identication of beneciaries and will lead to better targeting of subsidies.
It will provide for timely availability of food grain to intended beneciaries and checks leakage.
Setting up of redressal mechanism will create public accountability in implementation of TPDS.

7. Which of the following statements correctly denes the dierence in methodology adopted by Tendulkar Committee and Rangarajan Committee for
calculation of poverty?
1. The Tendulkar Committee adopted per capita monthly expenditure whereas Rangarajan Committee adopted calculation of monthly expenditure of
family of ve.
2. Rangarajan committee adopted only caloric value in expenditure for calculation of calorie intake whereas Tendulkar Committee adopted calorie,
protein and fat intake.
Codes:

(A) Only 1
(B) Only 2
(C) Both
(D) None
Selected answer: (Not selected..)
The correct answer is:(A)
Solution:Tendulkar committee adopted only caloric value in expenditure for calculation of calorie intake whereas Rangarajan Committee adopted calorie, protein
and fat intake.

8. Which of the following statements about Human Development Index (HDI) in the Human Development Report 2015 is/are correct?
1. It is a composite index measuring a long and healthy life, knowledge and a decent standard of living.
2. The HDI of Saudi Arabia, Kazakhstan, Libya and Tunisia are greater than that of India.
3. The range of very high developed nations in HDI lies from 0.7 to 1.
Codes:

(A) 1 and 2
(B) Only 2
(C) 1 and 3
(D) All
Selected answer: (Not selected..)
The correct answer is:(A)
Solution:These nations are in HIGH HUMAN DEVELOPMENT and VERY HIGH HUMAN DEVELOPMENT category. India falls in MEDIUM HUMAN DEVELOPMENT
category.
Human Development Index (HDI): A composite index measuring average achievement in three basic dimensions of human development-a long and
healthy life, knowledge and a decent standard of living. The range is from 0.8 to 1.

9. Consider the following statements about Padhe Bharat Badhe Bharat:


1. The programme was launched to improve access to Elementary Education.
2. The program focuses on language development to create interest in reading and writing in com-prehension.

3. The program also focuses on teaching Science during the early years of schooling.
Which of the above statements is/are correct?

(A) 1 and 2
(B) Only 2
(C) Only 1
(D) All
Selected answer: (Not selected..)
The correct answer is:(B)
Solution:The programme was launched to improve learning outcomes. The program focuses on language development to create interest in reading and writing in
comprehension &teaching mathematics in a way that develops liking and understanding during the early years of schooling, particularly in class I and II.

10. Which of the following statements about Skills Assessment Matrix for Vocational Advancement of Youth (SAMVAY) is correct?

(A) It allows vertical and lateral mobility within vocational education system and between the current education systems.
(B) It allows lateral mobility within current education system and between International accreditation education systems.
(C) It assists planning and delivery of training Identication of skill development needs and preparing a catalogue of skill types.
(D) Its main feature is Awareness building, popularisation and promotion of quality assurance mentoring Higher Education
Institutions.
Selected answer: (Not selected..)
The correct answer is:(A)
Solution:A credit framework - SAMVAY - allows vertical and lateral mobility within vocational education system and between the current education systems.
The strength of this framework is the seamless integration of pursuit of academic knowledge and practical vocational skills.

11. Which of the following measures have been undertaken by the Government for checking Malnutrition among Women?
1. Integrated Child Development Scheme.
2. Ujjawala.
3. Priyadarshini.
4. Mid day Meal Scheme.
5. MGNREGA.
Codes:

(A) 1, 2 and 3
(B) 1, 3 and 4
(C) 1, 3 and 5
(D) All
Selected answer: (Not selected..)
The correct answer is:(C)
Solution:The government is administering a pilot project titled Women's Empowerment and Livelihood Programme in the Mid Gangetic Plains "Priyadarshini" in 13
blocks spread over ve districts in Uttar Pradesh and two districts in Bihar. The project is assisted by IFAD. It aims at holistic empowerment (economic and
social) of vulnerable groups of women and adolescent girls in the project area through formation of women's Self Help Groups (SHGs) and promotion of
improved livelihood opportunities.
Mid Day meal scheme is for checking malnutrition in children.
Ujjawala scheme is for rehabilitation of tracked women.

=133384 2/6
3/27/2017 IASScore:PracticeTests
Ujjawala scheme is for rehabilitation of tracked women.

12. Which of the following is the objective of Family Planning Indeminty Scheme?

(A) Under this States/UTs would process and make payment of claims to beneciaries of sterilization in the event of
death/failure/complication.
(B) Under this States/UTs would provide sterilization facilities in the village with incentives.
(C) Under this the medical records and checklist for female/male sterilization in villages will be sterilised.
(D) Under this public sector and private/NGO facilities conducting sterilisation will be provided incentives.
Selected answer: (Not selected..)
The correct answer is:(A)
Solution:.

13. Which of the following has contributed to slow productivity growth in Agriculture?
1. Land degradation and soil fertility loss.
2. Shrinking farm size due to land fragmentation.
3. Exposure to international competition.
Codes:

(A) 1 and 2
(B) 2 and 3
(C) 1 and 3
(D) All
Selected answer: (Not selected..)
The correct answer is:(A)
Solution:International competition can have impact on production not productivity.

14. Consider the following statements about Kendriya Vidyalaya Sangathan (KVS):
1. All Kendriya Vidyalayas are co-educational.
2. KVs primarily cater to the educational needs of the wards of transferable Central & State Government ocers.
3. Tuition fee is not charged in case of girl students.
4. Bilingual medium of instructions, i.e. English and Hindi are followed in schools.
Which of the above statements are correct?

(A) 1 and 2
(B) 1, 3 and 4
(C) 1, 2 and 4
(D) All
Selected answer: (Not selected..)
The correct answer is:(B)
Solution:Salient Features of Kendriya Vidyalayas:
KVs primarily cater to the educational needs of the wards of transferable Central Government employees.
All Kendriya Vidyalayas are co-educational.
Common Text books, common curriculum and bilingual medium of instructions, i.e. English and Hindi are followed.
All KVs are aliated to the Central Board of Secondary Education.
In case of girl students, tuition fee is not charged at all.
Single girl child is exempted from the payment of all fees.

15. One of the key components of the National Rural Health Mission is Accredited Social Health Activists (ASHA). Which of the following statements is/are
correct regarding ASHA worker?
1. ASHA must primarily be a married/ widowed/divorced women, prefer-ably in the age group of 25 to 45 years from the group of villages coming under a
particular Tehsil.
2. She must be a literate with formal education of upto class Xth.
3. The training to the ASHA is provided by the particular Panchayat where she has been posted after selection.
Codes:

(A) Only 1
(B) 2 and 3
(C) All
(D) None
Selected answer: (Not selected..)
The correct answer is:(D)
Solution:ASHA must primarily be a woman resident of the village married/ widowed/ divorced, preferably in the age group of 25 to 45 years. She must be a literate
woman with formal education up to class eight. Selection and training of ASHA is by the concerned state.

16. Consider the following statements about the Expert Group (Rangarajan) for estimation of poverty:
1. The new poverty line estimated to monthly per capita consumption expenditure of Rs.972 in rural areas and Rs.1,407 in urban areas in 2011-12.
2. It uses the Modied Mixed Recall Period consumption expenditure data of the NSSO.
3. According to the report, the all-India poverty ratio is 21.5%.
Which of the above statements is/are correct?

(A) 1 and 3
(B) Only 2
(C) 1 and 2
(D) All
Selected answer: (Not selected..)
The correct answer is:(C)
Solution:The new poverty line thus work out to monthly per capita consumption expenditure of Rs.972 in rural areas and Rs.1,407 in urban areas in 2011-12.
The Expert Group (Rangarajan) uses the Modied Mixed Recall Period consumption expenditure data of the NSSO.
The all-India poverty ratio fell from 38.2% to 29.5%.

17. Which of following statements are correct regarding Stand Up India Scheme?
1. Stand Up India scheme aimed at encouraging entrepreneurship among Dalits, tribals and women by providing them bank loans upto one crore.
2. It provides for IPR protection to startups and ensures reduction in patent registration fee.
3. It provides for freedom from capital gains tax and freedom from tax in prots for 3 years.
4. It makes provision for setting innovation hub under Atal Innovation Mission.
Codes:

(A) 1, 2 and 4
(B) 2 and 3
(C) Only 1
(D) 1, 3 and 4
Selected answer: (Not selected..)
The correct answer is:(C)
Solution:Stand Up India Scheme" aimed to promote entrepreneurship among SC/ST and Women entrepreneurs. The Scheme is intended to facilitate at least two
=133384 3/6
3/27/2017 IASScore:PracticeTests
Solution:Stand Up India Scheme" aimed to promote entrepreneurship among SC/ST and Women entrepreneurs. The Scheme is intended to facilitate at least two
such projects per bank branch, on an average one for eachcategory of entrepreneur. It is expected to benet atleast 2.5 lakh borrowers.
The overall intent of the approval is to leverage the institutional credit structure to reach out to these under-served sectors of the population by
facilitating bank loans repayable up to 7 years and between Rs. 10 lakh to Rs. 100 lakh for greeneld enterprises in the non farm sector set up by such SC,
ST and Women borrowers.
The expected date of reaching the target of at least 2.5 lakh approvals is 36 months from the launch of the Scheme.
The Stand Up India Scheme provides for:
Renance window through Small Industries Development Bank of India (SIDBI) with an initial amount of Rs. 10,000 crore.
Creation of a credit guarantee mechanism through the National Credit Guarantee Trustee Company (NCGTC).
Handholding support for borrowers both at the pre loan stage and during operations. This would include increasing their familiarity with factoring
services, registration with online platforms and e-market places as well as sessions on best practices and problem solving.

18. Which of the following statements are correct with respect to National Family Health Survey?
1. National Family Health Survey is conducted by Ministry of Health and Family Welfare, in coordination with the International Institute for
Population Science, Mumbai.
2. Since 1992-93, in every three years Government publishes the National Family Health Survey.
3. It is funded by United States Agency for International Development with supplementary support from United Nations Children's Fund (UNICEF).
Codes:

(A) 1 and 2
(B) 2 and 3
(C) 1 and 3
(D) All
Selected answer: (Not selected..)
The correct answer is:(C)
Solution:The National Family Health Survey is a Large Scale, multi-round Survey conducted in a representative sample of household throughout India.Till now four
Family Health Survey has been conducted in NFHS-1 (1992-93), NFHS-2 (1998-99), NFHS-3 (2005-06) and NFHS-4 (2015-16). In NFHS-4, women aged 15-49
years and men aged 15-54 years are interviewed.

19. Capacity to be nourished (for the body to absorb food) depends on which of the following factors?
1. Health services
2. Basic education
3. Sanitary arrangements
4. Provision of clean water.
Codes:

(A) 1 and 2
(B) 2, 3 and 4
(C) All
(D) None
Selected answer: (Not selected..)
The correct answer is:(C)
Solution:Adequate availability and access to food does not necessarily mean that the food would be absorbed to ensure higher levels of nutrition. Food absorption
by the human body is a major problem particularly in rural areas and urban slums.According to researchers the "capacity to be nourished (for the body to
absorb food) depends crucially to other characteristics of a person that are inuenced by non-food factors such as medical attention, health services,
basic education, sanitary arrangements, provision of clean water, eradication of infectious epidemics and so on. This inability to absorb the food intake or
where the body is incapable of absorbing the nutrients from the food consumed can be termed absorption food insecurity.

20. Which of following statements are correct about Socio-Economic Caste Census (SECC-2011)?
1. SECC is rst paperless census in India conducted on hand-held devices.
2. SECC is rst ever post independance caste based Census since 1931 Census of India
3. It was conducted under Census of India Act, 1948.
4. It has better method of targetting beneciaries by 'automatic exclusion' based on certain parameters.
Codes:

(A) 1 and 2
(B) Only 3
(C) 1, 2 and 4
(D) None
Selected answer: (Not selected..)
The correct answer is:(C)
Solution:Socio Economic and Caste Census 2011 (SECC) was conducted in the 2011. The SECC 2011 was conducted in all states and union terriotories of India and
the rst ndings were revealed on 3 July 2015. SECC 2011 is also rst paperless census in India conducted on hand-held electronic devices by the
government.The rural development ministry has taken a decision to use the SECC data in all its programmes such as MGNREGA, National Food Security
Act, Deen Dayal Upadhyaya Grameen Kaushalya Yojana.
SECC 2011 was rst-ever post-Independent caste-based census since 1931 Census of India. SECC 2011 data will also be used to identify beneciary and
expand the direct benet transfer scheme SECC 2011 was not conducted under 1948 Census of India Act, which in turn made information disclosure
voluntary for citizens, and not a mandatory disclosure.
Socio Economic and Caste Census 2011 was the fourth exercise conducted by Government of India to identify households living below the poverty line in
India that would get various entitlements, after three censuses in 1992, 1997 and 2002.

21. Which of following statements regarding Gini coecient is/are correct?


1. Gini coecient states about income inequality
2. It is published by United Nation Development Programme.
3. '0' represents most unequal wealth distribution where as '1' represents most equal wealth distribution
Codes:

(A) 2 and 4
(B) Only 1
(C) Only 3
(D) All
Selected answer: (Not selected..)
The correct answer is:(B)
Solution:World Bank publishes Gini Coecient. Gini Coecient is an internationally accepted measure of income inequality. '0' represent equal wealth
distribution.'1'represent most unequal distribution of wealth.

22. India is a member of which of the following international agencies working in the eld of food related matters?
1. World Food Programme
2. International Grains Council
3. SAARC Food Bank
Codes:

(A) Only 3
(B) 1 and 3
(C) 2 and 3
(D) All
Selected answer: (Not selected..)
The correct answer is:(D)
Solution:India is associated with a number of international agencies working in the eld of food related matters. These include World Food Programme (WFP),
=133384 4/6
3/27/2017 IASScore:PracticeTests
Solution:India is associated with a number of international agencies working in the eld of food related matters. These include World Food Programme (WFP),
SAARC Food Bank, Food and Agriculture Organization (FAO), International Grains Council (IGC) etc.

23. Which of the following will result in enhancement of food availability?


1. Employment intensive pattern of growth wherein remunerative work is provided to the poor.
2. Increasing incomes and subsidizing food through social protection measures.
Codes:

(A) Only 1
(B) Only 2
(C) Both
(D) None
Selected answer: (Not selected..)
The correct answer is:(C)
Solution:Very often the poor cannot aord to obtain the available food due to the high level of market prices, therefore, various social protection programmes are
needed to ensure access. The sucient purchasing power in the hands of the poor can be made available through an employment intensive pattern of
growth wherein remunerative work is provided to the poor thereby enhancing their purchasing power. The other way of improving access is by increasing
incomes and subsidizing food through social protection measures such as employment generation programmes and the provision of aordable and
subsidized food through PDS.

24. Which of following statements regarding Demographic Transition are correct?


1. Demographic Transition (DT) refers to the transition from high birth and death rates to low birth and death rates as a country develops from a pre-
industrial to an industrialized economic system.
2. India is currently in second stage of demographic transition.
3. National Population Policy 2000 aims at bringing India to third stage of demographic transition.
Codes:

(A) 2 and 3
(B) 1 and 2
(C) 1, 2 and 3
(D) Only 1
Selected answer: (Not selected..)
The correct answer is:(D)
Solution:Demographic transition (DT) refers to the transition from high birth and death rates to low birth and death rates as a country develops from a pre-
industrial to an industrialized economic system. India is currently in third stage of demographic transition.

25. Which of the following factors are considered in FRP (Fair and Remunerative Price) of sugar?
1. Cost of production of sugar.
2. Return to growers from alternative crops.
3. Availability of sugar to consumers at fair price.
4. Recovery of sugar from sugarcane.
5. International price of sugar.
6. Reasonable margins for grower of sugarcane on account of risk and prot.
Codes:

(A) 1, 2 and 3
(B) 3, 4, 5 and 6
(C) 1, 2, 3, 4 and 6
(D) 1, 2, 4, 5 and 6
Selected answer: (Not selected..)
The correct answer is:(C)
Solution:Fixation of FRP of sugarcane is done according to following factors: (a) cost of production of sugarcane; (b) return to the growers from alternative crops
and the general trend of prices of agricultural commodities; (c) availability of sugar to consumers at a fair price; (d) price at which sugar produced from
sugarcane is sold by sugar producers; (e) recovery of sugar from sugarcane; (f) the realization made from sale of by-products viz. molasses, bagasse and
press mud or their imputed value; (g) reasonable margins for the growers of sugarcane on account of risk and prots.

Information for the solved exam

Started in: March 27, 2017, 12:25 am

Finished in: March 27, 2017, 12:25 am

Marks Obtained: 0

Right Ques.: 0

Wrong Ques.: 0

Unattempted Ques.: 25

Try for a better result? (index.php?section=user&page=continue_exam&id=133384&renew_exam=yes)

Rank List (index.php?section=user&page=ranking&id=146)

=133384 5/6
3/27/2017 IASScore:PracticeTests

=133384 6/6
3/27/2017 IASScore:PracticeTests

socio - Economic Development Test - 12

1. Which of the following initiatives launched by GOI in eld of education are incorrectly matched?
1. SWAYAM: Under this scholarships will be provided to dierently-abled students to pursue technical education.
2.Ishan Vikas: Under this selected students from the school and college levels from the North-Eastern states will be proposed to the opportunities in the
IITs, NITs and IISERs during their vacation periods.
3.Saksham: Under this professors of centrally funded institutions will oer online courses to citizens free of cost.
Codes:

(A) Only 1
(B) 2 and 3
(C) 1 and 3
(D) Only 2
Selected answer: (Not selected..)
The correct answer is:(C)
Solution:SWAYAM: Under this professors of centrally funded institutions will oer online courses to citizens free of cost. Saksham: Under this scholarships will be
provided to dierently-abled students to pursue technical education.

2. Pandit Madan Mohan Malviya National Mission for Teachers Training will focus on which of the following areas?
1.Teaching methodology development.
2.Research infrastructure in higher education.
3.Develop professional cadre of teachers.
4.Changes in Curriculum design.
5.Technology requirements for bringing inclusiveness.
Codes:

(A) 1, 3, 4 and 5
(B) 1, 2, 4 and 5
(C) 1, 2, 3 and 4
(D) 1, 3 and 4
Selected answer: (Not selected..)
The correct answer is:(D)
Solution:An umbrella scheme to create synergies among the various ongoing initiatives on Teachers and Teaching was launched.
The Scheme will address all issues related to teachers, teaching, teacher preparation, professional development, curriculum design.
Develop a strong professional cadre of teachers by setting performance standards and creating top class institutional facilities for innovative teaching.
The scheme will also address the need to induct qualied teachers, attracting talent into teaching profession and raising the quality of teaching in schools
and colleges.

3. Which of the following is the main reason behind decline in poverty rate to 12.4 per cent from the 2011-12 estimate of 21 per cent, according to new
data released by World Bank in 2015?

(A) Rural electrication and infrastructure development.


(B) Eective Implementation of MGNREGA.
(C) Employment Generation in Urban Area.
(D) Employment generation in Rural Area.
Selected answer: (Not selected..)
The correct answer is:(A)
Solution:India has reduced its poverty rate to 12.4 per cent from the 2011-12 estimate of 21 per cent, according to new data released by World Bank, which
identied rural electrication as an important driving factor for everything from greater rural spending to schooling for girls. The global poverty line is the
ability to live on $1.90 (Rs 123.5) per day, up from $1.25 (Rs 81). The change was made to reect dierences in cost of living across countries based on
2011 prices-in other words, to adjust for ination and other economic variables.
The World Bank said improved infrastructure, specically rural electrication, has had far-ranging eects, changed earnings, consumption and even
encouraged schooling for girls.

4. Which of the following statements is/are correct with respect to Rotavac, a Rotavirus medicine launched in India?
1.This vaccine will boost eorts to combat infant mortality due to diarrhea and this is the rst of its kind vaccine available globally against Rotavirus.
2.This was developed under the joint collaboration between India and the Israel in the area of medical research.
Codes:

(A) Only 1
(B) Only 2
(C) None
(D) Both
Selected answer: (Not selected..)
The correct answer is:(C)
Solution:This is the third such vaccine available globally against Rotavirus. This is joint collaboration of US and India. It was developed under the public-private
partnership (PPP) model that involved Union Ministry of Science and Technology, institutions of the US Government and NGOs in India supported by the
Bill and Melinda Gates Foundation.

5. Which of the following is the objective of 'Sehat' Initiative?

(A) This is telemedicine initiative by the central government in collaboration with Apollo Hospitals for urban slums in Delhi/NCR on
pilot basis.
(B) This is a initiative to promote sale of generic drugs through collaboration with Health Ministry by setting up 'Jan Aushadhi' Stores.
(C) This is telemedicine initiative by the central government in collaboration with Apollo Hospitals for rural areas.
(D) This is telemedicine initiative by the central government in collaboration with Medanta Hospitals for urban slums in Delhi/NCR
on pilot basis.
Selected answer: (Not selected..)
The correct answer is:(C)
Solution:Government of India launched a telemedicine initiative 'Sehat' for providing healthcare facilities in rural areas. It is collaboration of Apollo Hospitals and
GoI. The initiative was launched by Communication and IT Ministry and Health Ministry. Under this facility people can consult doctors through video link
and also order generic drugs. It will also help in providing quality and aordable healthcare in rural areas.

6. Consider the following statements about Udaan scheme launched by Ministry of Human Resource Development:
1.The Scheme is to promote the admission of girl students in primary education.
2.The aim is to address the teaching gap by providing free online supplementary lessons in form of videos, practice testa etc. science subjects.
3.It seeks to enhance the enrolment of girl students in prestigious technical education institutions through incentives & academic support.
Which of the above statements is/are correct?

(A) 1 and 2
(B) Only 1
(C) 2 and 3
(D) All
=133392 1/6
3/27/2017 IASScore:PracticeTests
(D) All
Selected answer: (Not selected..)
The correct answer is:(A)
Solution:The Scheme is dedicated to the development of girl child education, so as to promote the admission of girl students.
951 students have been selected by the CBSE.
The aim is to address the teaching gap between school education and engineering entrance examinations.
It seeks to enhance the enrolment of girl students in prestigious technical education institutions through incentives & academic support.
It oers free online supplementary lessons in physics, chemistry and maths specially prepared in the form of videos, text and practice tests on a tablet.

7. Which of the following statements correctly denes the term Maternal Mortality Rate (MMR) in India?

(A) It is the annual number of female deaths per 100,000 live births from any cause related to or aggravated by pregnancy or its
management.
(B) It is the annual number of female deaths per 1000 live births from any cause related to or aggravated by pregnancy or its
management.
(C) It is the annual number of female deaths per 100,000 live births after 42 days of termination of pregnancy or accidental or
incidental causes.
(D) It is the annual number of deaths of pregnant females per 100,000 live births.
Selected answer: (Not selected..)
The correct answer is:(A)
Solution:.

8. Which of the following statements is/are incorrect with respect to MPI (Multidimensional Poverty Index)?
1.The Index had been developed in 2010 by the World Bank.
2.The index uses the same three dimensions as the Human Development Index: health, education, and standard of living.
3.The dimensions have been divided in dierent indicators where each Indicator and dimension have been assigned dierent weightage according to their
importance in livelihood maintenance.
Codes:

(A) 1 and 3
(B) 2 and 3
(C) 1 and 2
(D) All
Selected answer: (Not selected..)
The correct answer is:(A)
Solution:This was developed in 2010 by the Oxford Poverty & Human Development Initiative (OPHI) and the United Nations Development Programme. Each
dimension and each indicator within a dimension is equally weighted.

9. What are the challenges faced by Mid Day Meal Programme?


1.Poor quality of food.
2.Exclusion of out of school children.
3.Inadequate arrangement for cooking and serving.
4.Issues related to additional work for school teachers.
Codes:

(A) Only 1
(B) 1 and 2
(C) 2, 3 and 4
(D) All
Selected answer: (Not selected..)
The correct answer is:(D)
Solution:.

10. According to the NSSO report which of the following statements regarding Rural & Urban food consumption pattern is/are correct?
1.Rural people consume more calories than their urban counterpart.
2.Urban people consume more protein than the rural counterpart.
3.An average rural person consumes more fat than the Urban counterpart.
Codes:

(A) Only 1
(B) 1 and 2
(C) Only 3
(D) All
Selected answer: (Not selected..)
The correct answer is:(A)
Solution:Year 2010
Calories (kcal)-Rural 2147
Calories (kcal)-Urban 2123
Protein-Rural 59.3
Protein- Urban 58.8
Fat (Rural 43.1
Fat -urban 53.0

11. Which of the following health interventions have not been proposed in the Union Budget 2016-17?

(A) LPG connection in the name of women members to BPL families over the next three years.
(B) Cess on cars to reduce pollution in urban areas.
(C) Increased allocation for Swatchch Bharat Abhiyan and National Rural Drinking Water Programme.
(D) The public expenditure on health has been increased to 1.3%.
Selected answer: (Not selected..)
The correct answer is:(D)
Solution:.

12. Consider the following statements related to TB:


1.Tuberculosis (TB) is an infectious disease caused by a Bacterium, Mycobacterium tuberculosis.
2.HIV- infected people are at increased risk of getting infected by TB.
3.MDRTB refers to strains of the bacterium which are proven in a laboratory to be resistant to the anti-TB drugs.
Which of the above statements are correct?

(A) 1 and 3
(B) 1 and 2
(C) 2 and 3
(D) All
Selected answer: (Not selected..)

=133392 2/6
3/27/2017 IASScore:PracticeTests
The correct answer is:(D)
Solution:.

13. According to WHO, which of the following parameters lead to food security in long term?
1.Availability of food
2.Accessibility of food
3.Absorption of food
4.Aordability of food
Codes:

(A) 1 and 2
(B) 1, 3 and 4
(C) 2 and 3
(D) All
Selected answer: (Not selected..)
The correct answer is:(D)
Solution:.

14. With the recommendation of the NITI AAYOG rationalization of Centrally Sponsored Scheme related to education have been done. Which of the
following schemes have been included under National Education Mission?
1.Rashtriya Uchchatar Shiksha Abhiyan.
2.Rashtriya Madhyamik Shiksha Abhiyan.
3.Navodaya Vidyalaya.
4.Support For Educational Development Including Teachers Training.
Codes:

(A) 1 and 2
(B) 1, 2 and 3
(C) 1, 2 and 4
(D) All
Selected answer: (Not selected..)
The correct answer is:(C)
Solution:Sarva Shiksha Abhiyan, Rashtriya Uchchatar Shiksha Abhiyan, Rashtriya Madhyamik Shiksha Abhiyan, support for educational development including
teachers training and Adult education have been placed under National Education Mission.

15. Consider the following statements regarding constituents of Indira Gandhi Matritva Sahyog Yojana (IGMSY):
1.It is a conditional cash transfer scheme for pregnant and lactating women.
2.It attempts to partly compensate for wage loss to pregnant and lactating women only prior to the delivery of child.
3.It covers both the organized and unorganized sector employees.
4.It encourages the women to follow (optimal) nutrition and feeding practices.
Which of the above statements are correct?

(A) 1, 2 and 4
(B) 1 and 4
(C) 3 and 4
(D) All
Selected answer: (Not selected..)
The correct answer is:(B)
Solution:IGMSY is a conditional cash transfer scheme for pregnant and lactating women launched. It attempts to partly compensate for wage loss to pregnant and
lactating women both prior and after the delivery of child. It covers only the unorganized sector employees. All organized sector employees are excluded
from the scheme as the entitled for paid maternity leave.

16. Which of the following reasons are responsible for the increment in the incidence of feminization of poverty in the developing nations?
1.The increasing prevalence of female-headed households due to rural-urban migration.
2.Gender biases often deprive women of opportunities to independently pursue education or careers.
3.Wage discrimination and deprivation of decision making power in women.
Codes:

(A) 2 and 3
(B) Only 3
(C) 1 and 3
(D) All
Selected answer: (Not selected..)
The correct answer is:(D)
Solution:.

17. What is the objective of the Swami Vivekananda Scholarship launched by UGC recently?

(A) To provide scholarship to Single Girl Child for research in Social Sciences.
(B) To provide scholarships to students from North East Region whose parental income is below Rs. 4.5 lakh per annum for studying
at under graduate level in Colleges/ Universities of the country.
(C) To provide scholarships to students from left-wing extremism region whose parental income is below Rs. 4.5 lakh per annum for
studying at under graduate level in Colleges/ Universities of the country.
(D) To provide scholarships to students from left-wing extremism region for preparing for medical and engineering entrance
examinations.
Selected answer: (Not selected..)
The correct answer is:(A)
Solution:.

18. Which of the following is/are true about the latest World bank poverty line calculation?
1.The poverty line has been increased to $1.90 from $ 1.25 using 2011 prices.
2.It uses Uniform Recall Period to estimate poverty.
3.The World Bank projects that the 9.6% of the global population is living in poverty.
Codes:

(A) Only 1
(B) 2 and 3
(C) 1 and 3
(D) All
Selected answer: (Not selected..)
The correct answer is:(C)
Solution:It has raised its ticket size to measure poverty to $1.90 from the older $1.25 since the price of goods and services across the world has risen.
Modied mixed reference period, or MMRP.(METHOD ADOPTED BY WB)
In this method, for some food items, instead of a 30-day recall, only a 7-day recall is collected. Also, for some low-frequency items, instead of a 30-day
recall, a 1-year recall is collected.
=133392 3/6
3/27/2017 IASScore:PracticeTests
recall, a 1-year recall is collected.
This is believed to provide a more accurate reection of consumption expenditures. When such data was collected, consumption expenditures for people
in both urban and rural areas went up by 10 per cent to 12 per cent.
This happened essentially because people could better recall their food expenditure over a shorter, 7-day period than what they might have done over
the longer 30-day period. The higher expenditures, combined with the high population density around the poverty line, essentially meant that the poverty
rate for India (for 2011-12) came down sharply.

19. According to the World Employment and Social Outlook: Trends 2016, which of the following short term measures will help in improving the quantity
and quality of jobs and tackling income inequality?
1.Changes in National macroeconomic policies.
2.Financing necessary infrastructure projects.
3.Strengthening labour market institutions.
4.Achieving the Sustainable Development Goals (SDGs).
5.Financial reforms with focus on Banking Sector.
Codes:

(A) 1 and 2
(B) 1, 3, 4 and 5
(C) 1, 2 and 5
(D) 1, 2, 3 and 4
Selected answer: (Not selected..)
The correct answer is:(A)
Solution:Source : World Employment and Social Outlook: Trends 2016
In the short term, there is room for manoeuvre in macroeconomic policies in many countries. In light of historically low interest rates, countries could
nance necessary infrastructure projects, which would have important multiplier eects without imposing a big burden on the public purse.
The need to address these long-term trends adds urgency to the calls by the ILO for a shift in economic and employment policies. It is particularly
important to strengthen labour market institutions and ensure that social protection systems are well designed, in order to prevent further increases in
long-term unemployment, underemployment and working poverty. A rebalancing in reform eorts is also needed. In particular, nancial reforms need to
ensure that banks perform their role of channelling resources into the real economy and into investment for sustainable enterprise expansion and job
creation.
In the medium to long term, achieving the sustainable development goals (SDGs), particularly decent and productive employment for all, will yield
signicant social dividends while contributing to strengthening and rebalancing the global economy.

20. Which of the following statements is/are correct about IMPRINT?


1.It is a pan-IIT and IISc joint collaboration to develop a blueprint for improving the accreditation system of higher education.
2.It will ensure that higher education institutes are involved in research activities relevant to the society.
Codes:

(A) Only 1
(B) Only 2
(C) Both

(D) None
Selected answer: (Not selected..)
The correct answer is:(B)
Solution:The initiative, 'IMPRINT India', is a pan-IIT and IISc joint collaboration to develop a blueprint for research of immediate relevance to society requiring
innovation, direct scientic research into identied areas, ensure higher funding support for research into these areas and measure outcomes of the
research eorts with reference to the impact on the standard of living in rural/urban areas.
IMPRINT India will focus on 10 themes and each theme will be coordinated by one IIT/IISc. For instance, IIT-Kharagpur will work on the themes of health
care, computer science and information and communication technology, while IIT-Kanpur will work on advance material, water resources and river
systems.

21. Which of the following is/are not the objective of setting up of Atal Innovation Mission?
1.To create awareness and provide knowledge inputs in creating innovation challenges and funding mechanism to government.
2.To act as a Techno-Financial, Incubation and Facilitation Programme to support all aspects of start-up businesses.
3.To create an umbrella structure to oversee innovation eco-system of the country.
Codes:

(A) 1 and 2
(B) 1 and 3
(C) Only 2
(D) All
Selected answer: (Not selected..)
The correct answer is:(C)
Solution:ATAL Innovation Mission objectives:
To create an umbrella structure to oversee innovation eco-systemof the country;
To provide platform and collaboration opportunities for dierent stakeholders; c.
To study and suggest best and novel practices to be adopted by dierent stakeholders in the innovation chain;
To provide policy inputs to NITI Aayog and various Government Departments and Organizations.
To create awareness and provide knowledge inputs in creating innovation challenges and funding mechanism to government; and,
To develop new programmes and policies for fostering innovation in dierent sectors of economy.

22. Which of the following are recommendations of Shanta Kumar committee on reforming PDS?
1.It has proposed for universal coverage under National Food Security Act.
2.It has recommended increment in monthly quantum of ration from 5kg to 7 kg.
3.It has recommended to link prices of grain sold to priority household to 50% of MSP.
Codes:

(A) Only 1
(B) 2 and 3
(C) All
(D) None
Selected answer: (Not selected..)
The correct answer is:(B)

Solution:The recommendations are


Reduce coverage from 67% to 40% of population
Raise monthly quantum of ration (5-------------> 7kg)
Link prices of grain sold through TPDS to 50% of MSP, deregulate fertiliser sector and give direct cash subsidy.

23. According to the TeamLease Employment Outlook Report 2016 which of the following factors are responsible for driving employment outlook in India?
1.High rate of Economic growth has been forecasted which may bring employment opportunities.
2.Consumer maturity has increased with Changing Consumer Attitude thus demand for new services has increased.
3.Engineering and Technology Based talent demand have increased by e-commerce and product companies.
4.Infrastructure projects are being accelerated.
Codes:

(A) 2, 3 and 4
(B)
=133392 4/6
3/27/2017 IASScore:PracticeTests
(B) 1 and 4
(C) 1, 3 and 4
(D) All
Selected answer: (Not selected..)
The correct answer is:(D)
Solution:What factors drive employment?
1.Economic growth forecast at 8.1%:
The United Nations reports that the Indian economy is set to lead the way in Asia with a stupendous 8.1% growth rate in 2015 and follow it up with an
8.2% in 2016.
2.Consumer maturity moves to the next level:
Mobile-internet is proving to transform the consumer attitude and preference way more than organized retail did, and in way shorter a time frame
3.Tech talent lured by ecommerce and product companies :
Top Indian engineering campuses are abuzz with magnanimous oers from startups, multinational companies and homegrown technology giants.
4. Infrastructure projects pace likely to come of age:
The Government is pushing through and accelerating infrastructural projects, an action meant to be a booster dose to the 'Make in India' initiative.

24. Which of the following initiative of the Ministry of Health & Family Welfare will focus on increasing adolescents' awareness about the adverse eects
and consequences of substance misuse?

(A) Kayakalp
(B) Kilkari
(C) Rashtriya Kishor Swasthya Karyakram
(D) Daksh
Selected answer: (Not selected..)
The correct answer is:(C)
Solution:.

25. ILO report 2015 states that falling participation rates are due to both cyclical and structural factors. Which of the following are related to structural
factors?
1.Population ageing.
2.Recession
3.Increasing years spent in education.
4.Slow recovery in the economic cycle
5.Slower growth in the working-age population.
Select the correct answer using the codes given below:
Codes:

(A) 1, 2, 3 and 5
(B) 1, 3 and 5
(C) 2, 3 and 5
(D) 2, 3 and 4
Selected answer: (Not selected..)
The correct answer is:(B)
Solution:As highlighted in the 2015 edition of this report, falling participation rates are due to both cyclical and structural factors.
When jobs are scarce due to recession or slow recovery in the economic cycle, some jobseekers become discouraged and drop out of the labour market.
In terms of structural factors, population ageing and increasing years spent in education in many countries result in shrinking or slower growth in the
working-age population.
These two eects need to be dierentiated to provide a clearer understanding of the future path of labour force participation and to design and
implement an eective set of policy interventions.

Information for the solved exam

Started in: March 27, 2017, 12:29 am

Finished in: March 27, 2017, 12:29 am

Marks Obtained: 0

Right Ques.: 0

Wrong Ques.: 0

Unattempted Ques.: 25

Try for a better result? (index.php?section=user&page=continue_exam&id=133392&renew_exam=yes)

Rank List (index.php?section=user&page=ranking&id=111)

=133392 5/6
3/27/2017 IASScore:PracticeTests

=133392 6/6
3/27/2017 IASScore:PracticeTests

socio - Economic Development Test - 11

1. Which of the following factors are considered in FRP (Fair and Remunerative Price) of sugar?
1.Cost of production of sugar.
2.Return to growers from alternative crops.
3.Availability of sugar to consumers at fair price.
4.Recovery of sugar from sugarcane.
5.International price of sugar.
6.Reasonable margins for grower of sugarcane on account of risk and prot.
Codes:

(A) 1, 2 and 3
(B) 3, 4, 5 and 6
(C) 1, 2, 3, 4 and 6
(D) 1, 2, 4, 5 and 6
Selected answer: (Not selected..)
The correct answer is:(C)
Solution:Fixation of FRP of sugarcane is done according to following factors: (a) cost of production of sugarcane; (b) return to the growers from alternative crops
and the general trend of prices of agricultural commodities; (c) availability of sugar to consumers at a fair price; (d) price at which sugar produced from
sugarcane is sold by sugar producers; (e) recovery of sugar from sugarcane; (f) the realization made from sale of by-products viz. molasses, bagasse and
press mud or their imputed value; (g) reasonable margins for the growers of sugarcane on account of risk and prots.

2. Which of the following materials are used for production of Ethanol for the Ethanol Blending Programme in India apart from sugarcane molasses?
1.Saw mill and paper mill discards
2.Elephant grass
3.Corn stover
Codes:

(A) 1 and 2
(B) 2 and 3
(C) Only 3
(D) All
Selected answer: (Not selected..)
The correct answer is:(D)
Solution:Ethanol produced from other non-food feedstocks besides molasses, like cellulosic and ligno cellulosic materials such as Saw Mill and paper mill discards,
elephant grass, corn stover, etc. including petrochemical route, has also been allowed to be procured subject to meeting the relevant Bureau of Indian
Standards (BIS) specications.

3. Consider the following statements with respect to the budgetary provisions related to the education sector:
1.The share of education in the total Union Budget has increased whereas for primary education it has been decreased.
2.Entrepreneurship education and training will be provided in government colleges, ITI s.
3.There is substantial cut in teacher training and adult education programmes.
Which of the above statements is/are correct?

(A) 1 and 2
(B) Only 2
(C) 2 and 3
(D) Only 1
Selected answer: (Not selected..)
The correct answer is:(C)
Solution:The share of education in the total Union Budget has declined from 3.8% to 3.7%.

4. Which of the following is/are the objectives of country Strategic Plan signed by UN World Food Programme and Government of India?
1.Ensure access to safe, nutritious and sucient food for all people all year round.
2.Ensure ending malnutrition according to internationally agreed targets for children under 5.
3.Addressing nutritional needs of adolescent girls, pregnant and lactating women.
Codes:

(A) 1 and 2
(B) Only 2
(C) 2 and 3
(D) All
Selected answer: (Not selected..)
The correct answer is:(D)
Solution:The main focus of WFP is on supporting the Government of India to make signicant and measurable progress in order to contribute to the following two
objectives:- (i) ensuring access to safe, nutritious and sucient food for all people all year round. (ii) ensuring ending malnutrition according to
internationally agreed targets, with a focus on stunting and wasting for children under 5 years of age, and addressing the nutritional needs of adolescent
girls, pregnant and lactating women, and older persons.

5. Which of the following are part of "9 point action plan "to reinvent TPDS?
1.States to undertake campaign for review of BPL/ Antodaya list.
2.Strict action against guilty to ensure leakage free distribution.
3.BPL/Antodaya list to be displayed on all FPS.
4.Training of members of FPS level vigilance committee.
5.Computerization of TPDS.
Codes:

(A) 1, 3 and 5
(B) 2, 3 and 5
(C) 1, 3, 4 and 5
(D) All
Selected answer: (Not selected..)
The correct answer is:(D)
Solution:Nine point Action Plan are as under:
States should undertake a campaign for review of BPL/AAY list to eliminate bogus ration cards.
Strict action should be taken against the guilty to ensure leakage free distribution of foodgrains.
For sake of transparency, involvement of elected Panchayati Raj Institution (PRI) members in distribution of foodgrains; licenses be given to Self Help
Groups (SHGs), gram panchayats, cooperatives etc.
BPL and AAY lists should be displayed on all FPSs.
District-wise and FPS-wise allocations of foodgrains should be put up on websites and other prominent places for public scrutiny.
Wherever possible, door-step delivery of foodgrains should be ensured by States, instead of letting private transporter/ wholesalers to transport goods.
Timely availability of foodgrains at FPS level and xed dates of distribution to ration card holders should be ensured.
Training of members of FPS level Vigilance Committee should be ensured
Computerization of TPDS operations should be undertaken.

6. Consider the following statements about the ethical poverty line (EPL):
=133395 1/6
3/27/2017 IASScore:PracticeTests
6. Consider the following statements about the ethical poverty line (EPL):
1.The ethical poverty line (EPL) is derived from globally standardised and ethically justiable well-being outcomes for suciently existing data.
2.The EPL overcomes the inherent problem of all income poverty lines as it provides a morally defensible basis for setting the line.
3.The EPL reveals that the price has to be paid for accepting a moral duty to remove widespread poverty by developed nations.
Which of the above statements are correct?

(A) 1 and 2
(B) 2 and 3
(C) 1 and 3
(D) All
Selected answer: (Not selected..)
The correct answer is:(C)
Solution:The ethical poverty line (EPL) proposed is derived from globally standardised and ethically justiable wellbeing outcomes for which sucient data already
exist in the World Development Indicators (WDI).
The EPL does not overcome the inherent problem of all income poverty lines, namely that they oversimplify and reduce the complexity of global poverty
to a mono dimensional monetary measure.
However, the EPL at least provides a morally defensible basis for setting the line.
The EPL, therefore, reveals that the price to be paid for accepting a moral duty to remove poverty today is one that would cut deeply into the pockets of
developed world populations.

7. Which of the following statements is/are correct about Rashtriya Arogya Nidhi?
1. It aims to provide nancial assistance to patients, living below poverty line who are suering from major life threatening diseases.
2.The patient is elligible to receive medical treatment at any of the super specialty hospitals/institutes or other government hospitals.
3. Financial assistance is released in the form of "two times grant to the person after the treatment through Direct Benet Transfer.
Codes:

(A) Only 1
(B) 1 and 3
(C) 1 and 2
(D) All
Selected answer: (Not selected..)
The correct answer is:(C)
Solution:Financial assistance is released in the form of "one times grant" to the Medical Superintendent of the hospital in which the treatment is being received.

8. The Inequality Adjusted HDI, stands for which of the following?

(A) It takes the average of inequality within and among countries based on Gini-Coecient.
(B) It is an index to measure relative deprivation.
(C) It is actual level of human development accounting for inequality.
(D) It can be viewed as potential human development which can be achieved.
Selected answer: (Not selected..)
The correct answer is:(C)
Solution:The 2010 Human Development Report introduced an Inequality-adjusted Human Development Index (IHDI). While the simple HDI remains useful, it stated
that "the IHDI is the actual level of human development (accounting for inequality)," and "the HDI can be viewed as an index of 'potential' human
development (or the maximum IHDI that could be achieved if there were no inequality).

9. Consider the following statements about Women at Work Trends 2016 by International Labour Organization 2016:
1.Inequality between women and men persists in global labour markets, in respect of opportunities, treatment and outcomes.
2.The chances for women to participate in the labour market remain lower than those for men.
3.Between 1995 and 2015, the global female labour force participation rate has increased signicantly due to globalisation.
Which of the above statements are correct?

(A) 1 and 2
(B) 2 and 3
(C) 1 and 3
(D) All
Selected answer: (Not selected..)
The correct answer is:(A)
Solution:Between 1995 and 2015, the global female labour force participation rate decreased from 52.4 to 49.6 per cent. Worldwide, the chances for women to
participate in the labour market remain almost 27 percentage points lower than those for men.

10. Consider the following statements about Poverty and Well-being:


1.Economic growth is the best indicator of poverty reduction as it depicts the standard of living of an individual.
2.Poverty gap index measures incidence of poverty and depth of poverty.
3.The parameters like Electricity, ooring, drinking water, sanitation, cooking fuel and assets determine the level of multi-dimensional poverty.
Which of the above statements are correct?

(A) 2 and 3
(B) 1 and 3
(C) 1 and 2
(D) All
Selected answer: (Not selected..)
The correct answer is:(A)
Solution:Economic growth is not a good indicator of poverty reduction.
Poverty means low levels of wellbeing, not just low income and it measures must relate closely to people's lives.
Poverty is failure of socially determined basic capabilities, but only when caused by lack of resources.A new international poverty line is needed. It should
reect the cost of basic human needs.

11. Which of the following is/are challenges to PDS?


1.Exclusion of APL has aected economic viability of FPS.
2.Large errors of exclusion and inclusion results in mis-targeting.
3.Regional disparities in the distribution of PDS benets.
4.Failure to translate into macro level food self suciency.
Codes:

(A) 1 and 2
(B) 1, 2 and 3
(C) 1, 2 and 4
(D) All
Selected answer: (Not selected..)
The correct answer is:(D)
Solution:.

12. In order to ensure that all eligible farmers are provided with hassle free and timely credit for their agricultural operation, Kisan Credit Card (KCC)
Scheme has been launched. Which of the following are the beneciaries to it?
1.Marginal farmers
=133395 2/6
3/27/2017 IASScore:PracticeTests
1.Marginal farmers
2.Share Croppers
3.Oral lessee and tenant farmers
4.Farm labours
Codes:

(A) 1, 2 and 3
(B) 2, 3 and 4
(C) 1, 3 and 4
(D) All
Selected answer: (Not selected..)
The correct answer is:(A)
Solution:In order to ensure that all eligible farmers are provided with hassle free and timely credit for their agricultural operation, Kisan Credit Card (KCC) Scheme
was introduced in 1998-99. Marginal farmers, share croppers, oral lessee and tenant farmers are eligible to be covered under the scheme. The main
objectives of the scheme are to meet the short term credit requirements for cultivation of crops, post harvest expenses, produce marketing loan,
consumption requirements of farmer household, working capital for maintenance of farm assets and activities allied to agriculture like dairy animals,
inland shery, etc., investment credit requirement for agriculture and allied activities like pump sets, sprayers, dairy animals, etc.

13. According to the Rangarajan formula for estimation of poverty place the following states in the hierarchial form with respect to the percentage of
poverty.
1.Chattisgarh
2.Odisha
3.Kerala
4.Lakshadweep
Codes:

(A) 1-2-3-4
(B) 2-1-3-4
(C) 2-1-4-3
(D) 1-2-4-3
Selected answer: (Not selected..)
The correct answer is:(A)

Solution:Chatisgarh - 47.9%
Orissa - 45.9
Keral -11.3%
Lakshadweep - 6.5%

14. Which of the following statements dene the benets of distribution of food coupons over cash transfers for maintenance of food security?
1.Food coupons insulate beneciaries from ination and price volatility as PDS shops supply foodgrains at subsidised rate.
2.Household is given the freedom to choose where it buys food.
3.Increases incentive for competitive prices and assured quality of foodgrain among PDS stores.
Codes:

(A) 1 and 2
(B) Only 1
(C) 2 and 3
(D) All
Selected answer: (Not selected..)
The correct answer is:(C)
Solution:Food coupons are not indexed for ination; may expose recipients to ination.

15. Which of the following statement correctly dences the term Shaala Darpan?

(A) It was launched to provide mobile access to parents of students of Government and Government aided schools.
(B) It was launched to provide direct internet access to Ministry of Human Resource Development of Gov-ernment and
Government aided schools.
(C) It was launched to provide mobile access to Teachers of Government and Government aided schools about Monthly Progress of
Students.
(D) It was launched to provide ICT based platform to provide Fund Disbursal Mechanism of Government and Government aided
schools directly to Ministry of Human Resource Development .
Selected answer: (Not selected..)
The correct answer is:(A)
Solution:ShaalaDarpan - A focused approach to give the right impetus to all the States in deploying ICT
ShaalaDarpan - an ICT programme of Ministry of Human Resource Development was launched to provide mobile access to parents of students of
Government and Government aided schools.
Using ShaalaDarpan parents can view updates on their child's progress. They can view records of attendance, assignments and achievements of their
child.

16. Which of the following can be initiated to improve agriculture to reduce poverty in Rural areas?
1.Rationalisation of Subsidy.
2.Organic farming.
3.Implementation of Pradhan Mantri Fasal Bima Yojana.
4.Technology adoption.
Codes:

(A) 1 and 4
(B) 1, 2 and 3
(C) 2, 3 and 4
(D) All
Selected answer: (Not selected..)
The correct answer is:(C)
Solution:Organic farming can help in to Lower input and optimum output (yield), increases the income and reduces the dependence of farmers on money lenders.
Technology adoption: Mechanization of agriculture, promotion of modern technologies like Vermicomposting, Fertigaiton, plastic mulches, polyhouse
cultivation - These should be promoted through provision of subsidies, grants and credit facilities. Successful implementation of PM fasal bima yojana:
Protect the farmers from various risks.

17. Which of the following components constitute the vector of poverty in calculating Rural and Urban Poverty?
1.Power and politics.
2.Class-prejudice.
3.Organizational participation in dierent associations.
4.Religiosity.
Select the correct answer using the codes given below:

(A) 1, 2 and 3
(B) 2, 3 and 4
(C) 1, 2 and 4
(D) All
=133395 3/6
3/27/2017 IASScore:PracticeTests
(D) All
Selected answer: (Not selected..)
The correct answer is:(D)
Solution:The components that constitute the vector of poverty have been chosen in term of unsatisfaction and deprivation.
Contents of each of the components that constitute the vector of poverty
(i)Class-prejudice: To nd out the poor's attitude towards the non-poor in terms of education, behaviour, intelligence and other facilities.
(ii)Religiosity: To nd out the religiosity among the poor in terms of ritualistic, intellectual belief and good work dimensions.
(iii) Political aliation : to nd out the aliation of the poor with political parties and their mode of political participation.
(iv) Organizational participation: To nd out the poor's organizational participation in terms of labour union, and other associations, as well as their
interest in national political aairs.
Other components are food, shelter, health, education, assets, etc.

18. Which of the following government initiatives comes under the National Mission on Agricultural Extension and Technology?
1.Media support to farmers through radio, doordharshan, etc.
2.Agri-clinics.
3.Information dissemination through Agri-Fairs.
4.Paramparagat Krishi Vikas Yojana.
Codes:

(A) 1, 2 and 3
(B) Only 4
(C) 1 and 3
(D) All
Selected answer: (Not selected..)
The correct answer is:(A)
Solution:.

19. Match the following:


I.Stunting 1) It is Measure of Chronic Undernutrition.
II.Wasting 2) Composite of both acute and chronic Undernutrition.
III.Underweight 3) It is measure of acute Undernutrition.
Codes:

(A) I- 1, II- 2, III- 3


(B) I- 2, II- 3, III- 1
(C) I- 1, II- 3, III- 2
(D) I- 3, II- 1, III- 2
Selected answer: (Not selected..)
The correct answer is:(C)
Solution:.

20. Consider the following statements about Unnat Bharat Abhiyaan:


1.Under this IIT and NIT will adopt villages and develop appropriate rural technologies for sustainable development through peoples participation.
2.It aims at Skill Development and enhancing the economic prospects of those engaged in traditional crafts and arts as a means of livelihood.
3.It will act as a platform that will connect institutes of higher education with local communities through e-education.
Which of the above statements is/are correct?

(A) 1 and 3
(B) Only 1
(C) 2 and 3
(D) All
Selected answer: (Not selected..)
The correct answer is:(B)
Solution:Under this:
IIT, IISER, NIT to adopt villages and develop appropriate rural technologies for sustainable development through peoples' participation.
The Abhiyan will enable processes that connect institutes of higher education with local communities. Focus on water management, organic farming,
renewable energy, frugal technology, infrastructure, and livelihood.

21. Consider the following statements related to the 'Quetelet index':


1.The Quetelet index is a value derived from the mass (weight) and height of an individual.
2.It quantify the amount of tissue mass in an individual.
3.It is a measure of malnutrition in the adults only as in children it is calculated in terms of wasting and stunting.
Which of the above statements are correct?

(A) 1 and 2
(B) 2 and 3
(C) 1 and 3
(D) All
Selected answer: (Not selected..)
The correct answer is:(D)
Solution:The most useful measure of malnutrition in adults is the body mass index (BMI). The body mass index (BMI) or Quetelet index is a value derived from the
mass (weight) and height of an individual. The BMI is an attempt to quantify the amount of tissue mass (muscle, fat, and bone) in an individual, and then
categorize that person as underweight, normal weight, overweight, or obese based on that value.

22. Which of the following government initiatives will ensure food security in India?
1.National Food Security Mission.
2.Krishonnati Yojana.
3.Bringing Green Revolution to Eastern India.
4.Digitization of PDS.
Codes:

(A) 1 and 3
(B) 1 and 2
(C) 2, 3 and 4
(D) All
Selected answer: (Not selected..)
The correct answer is:(D)
Solution:.

23. Which of the following statement is correct with respect to Mosquirix or RTS,S vaccine?

(A) This is world's rst Malarial vaccine approved by European Medicines Agency.
(B) This is rst licensed human vaccine against parasitic disease, Zika in South America.
(C) This vaccine was developed and marketed by India and America for AIDS patient in Africa.
(D) This is the world largest selling vaccine for malaria & chikungunya.
Selected answer: (Not selected..)
=133395 4/6
3/27/2017 IASScore:PracticeTests
Selected answer: (Not selected..)
The correct answer is:(A)
Solution:This is world's rst Malarial vaccine approved by European medicines Agency. This vaccine was licensed for use on babies aged between 6 weeks and 17
months in Africa, which is at the highest risk of the mosquito-borne disease. The vaccine developed by British drugmaker GlaxoSmithKline in partnership
with the PATH Malaria Vaccine Initiative. The vaccine was approved despite the dampened results released in 2011 and 2012. The results showed that it
only reduced episodes of malaria in babies aged 6-12 weeks by 27 percent, and by around 46 percent in children aged 5-17 months.

24. Which of the following indices are used for calculation of Human Poverty Index for industrialized nations?
1.Deprivation of longevity is measured as a percentage of the individuals with a life expectancy lower than 60 years.
2.Deprivation of knowledge is expressed as a percentage of illiterate adults.
3.Social exclusion is measured in terms of long-term unemployment rate.
Select the correct answer using the codes given below:
Codes:

(A) 1 and 2
(B) Only 2
(C) 1 and 3
(D) All
Selected answer: (Not selected..)
The correct answer is:(D)
Solution:The human poverty index for industrialised countries uses the same dimensions of the previous index, but the variables and reference values are
dierent:
Deprivation of longevity is measured by the percentage of individuals whose life expectancy is below 60 years of age (P1),
Deprivation of knowledge is based on the percentage of adults functionally illiterate according to the OECD denition (P2),
Deprivation of decent life standards (P3) is the percentage of the population living below the poverty level, as dened according to the criteria of the
International Standard of Poverty Line, thus being equal to 50% of the per capita average national income.
HPI-2 also considers a fourth dimension, social exclusion, measured with the long-term unemployment rate

25. Which of the following is/are correct with respect to Crimean-Congo Haemorrhagic fever (CCHF)?
1.CCHF is mostly found among the hot and humid climate of Ganga plain and Southern States of India.
2.It is transmitted to people from tick bites or with close contact to livestock animals.
3.CCHF is endemic in countries south of the 50th parallel North.
Codes:

(A) Only 1
(B) Only 2
(C) 2 and 3
(D) All
Selected answer: (Not selected..)
The correct answer is:(C)
Solution:National Institute of Virology Pune- conrmed several outbreaks of CCHF in Rajsthan, Gujarat and Uttar Pradesh in 2014 and 2015.

Information for the solved exam

Started in: March 27, 2017, 12:31 am

Finished in: March 27, 2017, 12:31 am

Marks Obtained: 0

Right Ques.: 0

Wrong Ques.: 0

Unattempted Ques.: 25

Try for a better result? (index.php?section=user&page=continue_exam&id=133395&renew_exam=yes)

Rank List (index.php?section=user&page=ranking&id=110)

=133395 5/6
3/27/2017 IASScore:PracticeTests

=133395 6/6
3/27/2017 IASScore:PracticeTests

socio - Economic Development Test - 10

1. Consider the given statements about antibiotic resistance:


1.It occurs when an antibiotic loose its ability to eectively control or kill bacterial growth.
2.Resistance occurs due to genetic mutation.
3.The current higher levels of antibiotic resistant bacteria are attributed to overuse and abuse of antibiotics.
4.A bacteria after becoming antibiotic resistant cannot lose its resistance traits i.e. the process of antibiotic resistance cannot be reversed.
Which of the above statements is/are correct?

(A) 1, 2 and 3
(B) 1, 3 and 4
(C) 1, 2 and 4
(D) All
Selected answer: (Not selected..)
The correct answer is:(A)
Solution:Bacteria can lose its traits and Antibiotic Resistance can be reversed. Resistant microorganisms (including bacteria, fungi, viruses and parasites) are able to
withstand attack by antimicrobial drugs such as antibacterial drugs (e.g. antibiotics), anti fungal, anti viral and anti malarial. During Antibiotic Resistance
standard treatments become ineective and infections increases the risk of spread to others.

2. Which of the following statements are correct about Antyodaya Anna Yojana?
1.Only the poorest section of BPL are eligible to get foodgrains at highly subsidized rate.
2.People covered under Antyodaya Anna Yojana are not eligible to avail benets of PDS, meant for BPL .
3.Its a centrally sponsored programme, where center bears all the cost from foodgrain to distribution.
4.The Food Security Act has drastically reduced the quantity of grain allocated to Antodaya families.
Codes:

(A) 1 and 2
(B) 1 and 3
(C) 2, 3 and 4
(D) All
Selected answer: (Not selected..)
The correct answer is:(A)
Solution:In order to make TPDS more focused and targeted towards the poorest section of population , the "Antodaya Anna Yojana" was launched to benet
poorest of poor section within BPL families.
Highly subsidized food grain (Rs 2/ wheat, Rs 3/rice) is provided under this scheme.
Under this scheme state/UT bear the distribution cost including margins to dealers and retailers as well as the transportation cost.
NFSA has continued the Antodaya Anna Yojana.

3. Which of the following statement correctly explain the term "Genetic Superheroes"?

(A) People whose genomes contains only Y chromosomes that would have caused devastating disorders but they are not sick.
(B) These are people who remain healthy even though they were born with genetic mutations.
(C) This is Super Genome which can be use for producing any Human body part.
(D) A genome which can withstand any harsh nuclear or thermal condition without any mutation.
Selected answer: (Not selected..)
The correct answer is:(A)
Solution:These are people who remain healthy even though they were born with genetic mutations that would usually lead to devastating disorders.

4. Consider the following statement/s regarding National Food Security Mission:


1.It aims at additional production of rice, wheat, pulses, coarse cereals and commercial crops.
2.Krishi Karman award is given as a reward to the best performing state in foodgrains production category.
3.Under this cropping system based training is provided.
Which of the above statements is/are correct?

(A) 1 and 2
(B) Only 2
(C) 2 and 3
(D) All
Selected answer: (Not selected..)
The correct answer is:(D)
Solution:.

5. World Health Organisation has declared India free of Maternal and Neonatal Tetanus. Which of the following are correct with respect to Maternal and
Neonatal Tetanus?
1.It usually occurs through infection due to minor outer injury at childhood.
2.It is prevalent in both developed and developing countries.

3.Infants who have not acquired passive immunity because the mother has never been immunised are at more risk.
4.Now it has been reduced to less than one case per 1000 live births across the country.
Codes:

(A) 1 and 2
(B) 3 and 4
(C) 1, 3 and 4
(D) All
Selected answer: (Not selected..)
The correct answer is:(B)
Solution:It usually occurs through infection of the unhealed umbilical stump, particularly when the stump is cut with a non-sterile instrument. Neonatal tetanus
mostly occurs in developing countries, particularly those with the least developed health infrastructure. It is rare in developed countries.

6. Which of the following statements is/are correct with respect to the guidelines issued by Law Commission for Surrogacy in India?
1.The birth certicate of the surrogate child should contain the name of the commissioning parents and Surrogate mother.
2.During Surrogacy, cases of abortions should be governed by the Assisted Reproductive Technology Regulation Act 2010.
3.A surrogacy contract should necessarily take care of life insurance cover for surrogate mother.
Codes:

(A) 2 and 3
(B) Only 3
=133397 1/6
3/27/2017 IASScore:PracticeTests
(B) Only 3
(C) Only 1
(D) All
Selected answer: (Not selected..)
The correct answer is:(B)
Solution:The following observations had been made by the Law Commission:
Surrogacy arrangement will continue to be governed by contract amongst parties, which will contain all the terms requiring consent of surrogate mother
to bear child, agreement of her husband and other family members for the same, medical procedures of articial insemination, reimbursement of all
reasonable expenses for carrying child to full term, willingness to hand over the child born to the commissioning parent(s), etc. But such an arrangement
should not be for commercial purposes.
A surrogacy arrangement should provide for nancial support for surrogate child in the event of death of the commissioning couple or individual before
delivery of the child, or divorce between the intended parents and subsequent willingness of none to take delivery of the child.
A surrogacy contract should necessarily take care of life insurance cover for surrogate mother.
One of the intended parents should be a donor as well, because the bond of love and aection with a child primarily emanates from biological
relationship. Also, the chances of various kinds of child-abuse, which have been noticed in cases of adoptions, will be reduced. In case the intended parent
is single, he or she should be a donor to be able to have a surrogate child. Otherwise, adoption is the way to have a child which is resorted to if biological
(natural) parents and adoptive parents are dierent.
Legislation itself should recognize a surrogate child to be the legitimate child of the commissioning parent(s) without there being any need for adoption
or even declaration of guardian.
The birth certicate of the surrogate child should contain the name(s) of the commissioning parent(s) only.
Right to privacy of donor as well as surrogate mother should be protected.
Sex-selective surrogacy should be prohibited.
Cases of abortions should be governed by the Medical Termination of Pregnancy Act 1971 only.

7. Which of the following is/are achievements under "end to end Computerization of PDS'?
1.Digitization of FPS and godown database.
2.Online allocation of foodgrains.
3.Toll free help-line for TPDS.
Codes:

(A) Only 1
(B) 1 and 2
(C) 2 and 3
(D) All
Selected answer: (Not selected..)
The correct answer is:(D)
Solution:The underlying objectives of the activities under the scheme are:
Digitization of beneciary and other database will enable correct identication of beneciaries and will lead to better targeting of subsidies.
It will provide for timely availability of food grain to intended beneciaries and checks leakage.
Setting up of redressal mechanism will create public accountability in implementation of TPDS.

8. Which of the following statements correctly denes the dierence in methodology adopted by Tendulkar Committee and Rangarajan Committee for
calculation of poverty?
1.The Tendulkar Committee adopted per capita monthly expenditure whereas Rangarajan Committee adopted calculation of monthly expenditure of
family of ve.
2.Rangarajan committee adopted only caloric value in expenditure for calculation of calorie intake whereas Tendulkar Committee adopted calorie,
protein and fat intake.
Codes:

(A) Only 1
(B) Only 2
(C) Both
(D) None

Selected answer: (Not selected..)


The correct answer is:(A)
Solution:Tendulkar committee adopted only caloric value in expenditure for calculation of calorie intake whereas Rangarajan Committee adopted calorie, protein
and fat intake.

9. Which of the following statements about Human Development Index (HDI) in the Human Development Report 2015 is/are correct?
1.It is a composite index measuring a long and healthy life, knowledge and a decent standard of living.
2.The HDI of Saudi Arabia, Kazakhstan, Libya and Tunisia are greater than that of India.
3.The range of very high developed nations in HDI lies from 0.7 to 1.
Codes:

(A) 1 and 2
(B) Only 2
(C) 1 and 3
(D) All
Selected answer: (Not selected..)
The correct answer is:(A)
Solution:These nations are in HIGH HUMAN DEVELOPMENT and VERY HIGH HUMAN DEVELOPMENT category. India falls in MEDIUM HUMAN DEVELOPMENT
category.
Human Development Index (HDI): A composite index measuring average achievement in three basic dimensions of human development-a long and
healthy life, knowledge and a decent standard of living.
The range is from 0.8 to 1.

10. Consider the following statements about Padhe Bharat Badhe Bharat:
1.The programme was launched to improve access to Elementary Education.
2.The program focuses on language development to create interest in reading and writing in com-prehension.
3.The program also focuses on teaching Science during the early years of schooling.
Which of the above statements is/are correct?

(A) 1 and 2
(B) Only 2
(C) Only 1
(D) All
Selected answer: (Not selected..)
The correct answer is:(B)
Solution:The programme was launched to improve learning outcomes.
The program focuses on language development to create interest in reading and writing in comprehension &teaching mathematics in a way that develops
liking and understanding during the early years of schooling, particularly in class I and II.

11. Which of the following statements about Skills Assessment Matrix for Vocational Advancement of Youth (SAMVAY) is correct?

=133397 2/6
3/27/2017 IASScore:PracticeTests

(A) It allows vertical and lateral mobility within vocational education system and between the current education systems.
(B) It allows lateral mobility within current education system and between International accreditation education systems.
(C) It assists planning and delivery of training Identication of skill development needs and preparing a catalogue of skill types.
(D) Its main feature is Awareness building, popularisation and promotion of quality assurance mentoring Higher Education
Selected answer: (Not selected..)
The correct answer is:(A)
Solution:A credit framework - SAMVAY - allows vertical and lateral mobility within vocational education system and between the current education systems.
The strength of this framework is the seamless integration of pursuit of academic knowledge and practical vocational skills.

12. Which of the following measures have been undertaken by the Government for checking Malnutrition among Women?
1.Integrated Child Development Scheme.
2.Ujjawala.
3.Priyadarshini.
4.Mid day Meal Scheme.
5.MGNREGA.
Codes:

(A) 1, 2 and 3
(B) 1, 3 and 4
(C) 1, 3 and 5
(D) All
Selected answer: (Not selected..)
The correct answer is:(C)
Solution:The government is administering a pilot project titled Women's Empowerment and Livelihood Programme in the Mid Gangetic Plains "Priyadarshini" in 13
blocks spread over ve districts in Uttar Pradesh and two districts in Bihar. The project is assisted by IFAD. It aims at holistic empowerment (economic and
social) of vulnerable groups of women and adolescent girls in the project area through formation of women's Self Help Groups (SHGs) and promotion of
improved livelihood opportunities.
Mid Day meal scheme is for checking malnutrition in children.
Ujjawala scheme is for rehabilitation of tracked women.

13. Which of the following is the objective of Family Planning Indeminty Scheme?

(A) Under this States/UTs would process and make payment of claims to beneciaries of sterilization in the event of
death/failure/complication.
(B) Under this States/UTs would provide sterilization facilities in the village with incentives.
(C) Under this the medical records and checklist for female/male sterilization in villages will be sterilised.
(D) Under this public sector and private/NGO facilities conducting sterilisation will be provided incentives.
Selected answer: (Not selected..)
The correct answer is:(A)
Solution:.

14. Which of the following has contributed to slow productivity growth in Agriculture?
1.Land degradation and soil fertility loss.
2.Shrinking farm size due to land fragmentation.
3.Exposure to international competition.
Codes:

(A) 1 and 2
(B) 2 and 3
(C) 1 and 3
(D) All
Selected answer: (Not selected..)
The correct answer is:(A)
Solution:International competition can have impact on production not productivity.

15. Consider the following statements about Kendriya Vidyalaya Sangathan (KVS):
1.All Kendriya Vidyalayas are co-educational.
2.KVs primarily cater to the educational needs of the wards of transferable Central & State Government ocers.
3.Tuition fee is not charged in case of girl students.
4.Bilingual medium of instructions, i.e. English and Hindi are followed in schools.
Which of the above statements are correct?

(A) 1 and 2
(B) 1, 3 and 4
(C) 1, 2 and 4
(D) All
Selected answer: (Not selected..)
The correct answer is:(B)
Solution:Salient Features of Kendriya Vidyalayas:
KVs primarily cater to the educational needs of the wards of transferable Central Government employees.
All Kendriya Vidyalayas are co-educational.
Common Text books, common curriculum and bilingual medium of instructions, i.e. English and Hindi are followed.
All KVs are aliated to the Central Board of Secondary Education.
In case of girl students, tuition fee is not charged at all.
Single girl child is exempted from the payment of all fees.

16. One of the key components of the National Rural Health Mission is Accredited Social Health Activists (ASHA). Which of the following statements is/are
correct regarding ASHA worker?
1.ASHA must primarily be a married/ widowed/divorced women, prefer-ably in the age group of 25 to 45 years from the group of villages coming under a
particular Tehsil.
2.She must be a literate with formal education of upto class Xth.
3.The training to the ASHA is provided by the particular Panchayat where she has been posted after selection.
Codes:

(A) Only 1
(B) 2 and 3
(C) All
(D) None
Selected answer: (Not selected..)
=133397 3/6
3/27/2017 IASScore:PracticeTests
Selected answer: (Not selected..)
The correct answer is:(D)
Solution:ASHA must primarily be a woman resident of the village married/ widowed/ divorced, preferably in the age group of 25 to 45 years. She must be a literate
woman with formal education up to class eight. Selection and training of ASHA is by the concerned state.

17. Consider the following statements about the Expert Group (Rangarajan) for estimation of poverty:
1.The new poverty line estimated to monthly per capita consumption expenditure of Rs.972 in rural areas and Rs.1,407 in urban areas in 2011-12.
2.It uses the Modied Mixed Recall Period consumption expenditure data of the NSSO .
3.According to the report, the all-India poverty ratio is 21.5%.
Which of the above statements is/are correct?

(A) 1 and 3
(B) Only 2
(C) 1 and 2
(D) All
Selected answer: (Not selected..)
The correct answer is:(C)
Solution:The new poverty line thus work out to monthly per capita consumption expenditure of Rs.972 in rural areas and Rs.1,407 in urban areas in 2011-12.
The Expert Group (Rangarajan) uses the Modied Mixed Recall Period consumption expenditure data of the NSSO
The all-India poverty ratio fell from 38.2% to 29.5%.

18. Which of following statements are correct regarding Stand Up India Scheme?
1.Stand Up India scheme aimed at encouraging entrepreneurship among Dalits, tribals and women by providing them bank loans upto one crore.
2.It provides for IPR protection to startups and ensures reduction in patent registration fee.
3.It provides for freedom from capital gains tax and freedom from tax in prots for 3 years.
4.It makes provision for setting innovation hub under Atal Innovation Mission.
Codes:

(A) 1, 2 and 4
(B) 2 and 3
(C) Only 1
(D) 1, 3 and 4
Selected answer: (Not selected..)
The correct answer is:(C)
Solution:Stand Up India Scheme" aimed to promote entrepreneurship among SC/ST and Women entrepreneurs. The Scheme is intended to facilitate at least two
such projects per bank branch, on an average one for eachcategory of entrepreneur. It is expected to benet atleast 2.5 lakh borrowers.
The overall intent of the approval is to leverage the institutional credit structure to reach out to these under-served sectors of the population by
facilitating bank loans repayable up to 7 years and between Rs. 10 lakh to Rs. 100 lakh for greeneld enterprises in the non farm sector set up by such SC,
ST and Women borrowers.
The expected date of reaching the target of at least 2.5 lakh approvals is 36 months from the launch of the Scheme.
The Stand Up India Scheme provides for:
Renance window through Small Industries Development Bank of India (SIDBI) with an initial amount of Rs. 10,000 crore.
Creation of a credit guarantee mechanism through the National Credit Guarantee Trustee Company (NCGTC).
Handholding support for borrowers both at the pre loan stage and during operations. This would include increasing their familiarity with factoring
services, registration with online platforms and e-market places as well as sessions on best practices and problem solving.

19. Which of the following statements are correct with respect to National Family Health Survey?
1.National Family Health Survey is conducted by Ministry of Health and Family Welfare, in coordination with the International Institute for Population
Science, Mumbai.
2.Since 1992-93, in every three years Government publishes the National Family Health Survey.
3.It is funded by United States Agency for International Development with supplementary support from United Nations Children's Fund (UNICEF).
Codes:

(A) 1 and 2
(B) 2 and 3
(C) 1 and 3
(D) All
Selected answer: (Not selected..)
The correct answer is:(C)
Solution:The National Family Health Survey is a Large Scale, multi-round Survey conducted in a representative sample of household throughout India.Till now four
Family Health Survey has been conducted in NFHS-1 (1992-93), NFHS-2 (1998-99), NFHS-3 (2005-06) and NFHS-4 (2015-16). In NFHS-4, women aged 15-49
years and men aged 15-54 years are interviewed.

20. Capacity to be nourished (for the body to absorb food) depends on which of the following factors?
1.Health services
2.Basic education
3.Sanitary arrangements
4.Provision of clean water.
Codes:

(A) 1 and 2
(B) 2, 3 and 4
(C) All
(D) None
Selected answer: (Not selected..)
The correct answer is:(C)
Solution:Adequate availability and access to food does not necessarily mean that the food would be absorbed to ensure higher levels of nutrition. Food absorption
by the human body is a major problem particularly in rural areas and urban slums.According to researchers the "capacity to be nourished (for the body to
absorb food) depends crucially to other characteristics of a person that are inuenced by non-food factors such as medical attention, health services,
basic education, sanitary arrangements, provision of clean water, eradication of infectious epidemics and so on. This inability to absorb the food intake or
where the body is incapable of absorbing the nutrients from the food consumed can be termed absorption food insecurity.

21. Which of following statements are correct about Socio-Economic Caste Census (SECC-2011)?
1.SECC is rst paperless census in India conducted on hand-held devices.
2.SECC is rst ever post independance caste based Census since 1931 Census of India
3.It was conducted under Census of India Act, 1948.
4.It has better method of targetting beneciaries by 'automatic exclusion' based on certain parameters.
Codes:

(A) 1 and 2
(B) Only 3
(C) 1, 2 and 4
(D) None
Selected answer: (Not selected..)
The correct answer is:(C)
=133397 4/6
3/27/2017 IASScore:PracticeTests
The correct answer is:(C)
Solution:Socio Economic and Caste Census 2011 (SECC) was conducted in the 2011. The SECC 2011 was conducted in all states and union terriotories of India and
the rst ndings were revealed on 3 July 2015. SECC 2011 is also rst paperless census in India conducted on hand-held electronic devices by the
government.The rural development ministry has taken a decision to use the SECC data in all its programmes such as MGNREGA, National Food Security
Act, Deen Dayal Upadhyaya Grameen Kaushalya Yojana.
SECC 2011 was rst-ever post-Independent caste-based census since 1931 Census of India. SECC 2011 data will also be used to identify beneciary and
expand the direct benet transfer scheme
SECC 2011 was not conducted under 1948 Census of India Act, which in turn made information disclosure voluntary for citizens, and not a mandatory
disclosure.
Socio Economic and Caste Census 2011 was the fourth exercise conducted by Government of India to identify households living below the poverty line in
India that would get various entitlements, after three censuses in 1992, 1997 and 2002.

22. Which of following statements regarding Gini coecient is/are correct?


1.Gini coecient states about income inequality
2.It is published by United Nation Development Programme.
3.'0' represents most unequal wealth distribution where as '1' represents most equal wealth distribution
Codes:

(A) 2 and 4
(B) Only 1
(C) Only 3
(D) All
Selected answer: (Not selected..)
The correct answer is:(B)
Solution:World Bank publishes Gini Coecient. Gini Coecient is an internationally accepted measure of income inequality. '0' represent equal wealth
distribution.'1'represent most unequal distribution of wealth.

23. India is a member of which of the following international agencies working in the eld of food related matters?
1.World Food Programme
2.International Grains Council
3.SAARC Food Bank
Codes:

(A) Only 3
(B) 1 and 3
(C) 2 and 3
(D) All
Selected answer: (Not selected..)
The correct answer is:(D)
Solution:India is associated with a number of international agencies working in the eld of food related matters. These include World Food Programme (WFP),
SAARC Food Bank, Food and Agriculture Organization (FAO), International Grains Council (IGC) etc.

24. Which of the following will result in enhancement of food availability?


1.Employment intensive pattern of growth wherein remunerative work is provided to the poor.
2.Increasing incomes and subsidizing food through social protection measures.
Codes:

(A) Only 1
(B) Only 2
(C) Both
(D) None
Selected answer: (Not selected..)
The correct answer is:(C)
Solution:Very often the poor cannot aord to obtain the available food due to the high level of market prices, therefore, various social protection programmes are
needed to ensure access. The sucient purchasing power in the hands of the poor can be made available through an employment intensive pattern of
growth wherein remunerative work is provided to the poor thereby enhancing their purchasing power. The other way of improving access is by increasing
incomes and subsidizing food through social protection measures such as employment generation programmes and the provision of aordable and
subsidized food through PDS.

25. Which of following statements regarding Demographic Transition are correct?


1.Demographic Transition (DT) refers to the transition from high birth and death rates to low birth and death rates as a country develops from a pre-
industrial to an industrialized economic system.
2.India is currently in second stage of demographic transition.
3. National Population Policy 2000 aims at bringing India to third stage of demographic transition.
Codes:

(A) 2 and 3
(B) 1 and 2
(C) 1, 2 and 3
(D) Only 1
Selected answer: (Not selected..)
The correct answer is:(D)
Solution:Demographic transition (DT) refers to the transition from high birth and death rates to low birth and death rates as a country develops from a pre-
industrial to an industrialized economic system.
India is currently in third stage of demographic transition.

Information for the solved exam

Started in: March 27, 2017, 12:32 am

Finished in: March 27, 2017, 12:32 am

Marks Obtained: 0

Right Ques.: 0

Wrong Ques.: 0

Unattempted Ques.: 25

Try for a better result? (index.php?section=user&page=continue_exam&id=133397&renew_exam=yes)


=133397 5/6
3/27/2017 IASScore:PracticeTests
Try for a better result? (index.php?section=user&page=continue_exam&id=133397&renew_exam=yes)

Rank List (index.php?section=user&page=ranking&id=108)

=133397 6/6
3/27/2017 IASScore:PracticeTests

Socio - Economic Development-13

1. Which of the following steps have been taken by the government for 'ensuring availability of quality medicines at aordable prices to all'?
1. Price control of Scheduled Drugs through the National Pharmaceutical Pricing Authority.
2. Government has xed a uniform and low rate of 4% VAT on medicines in the country.
3. Selling generic medicines through the dedicated stores under Jan Aushadhi initiative.
Codes:

(A) 1 and 2
(B) Only 3
(C) 2 and 3
(D) All
Selected answer: (Not selected..)
The correct answer is:(D)
Solution:Accordingly, 'ensuring availability of quality medicines at aordable prices to all', has been a key objective of the Government. Some of the important
steps taken to enable this are:
Price control of Scheduled Drugs through the National Pharmaceutical pricing authority (NPPA): Under the Drug Price Control Order, 1995 NPPA has
been given the mandate to control and x the maximum retail prices of a number of scheduled/listed bulk drugs and their formulations, in accordance
with well dened criteria and methods of accounting, relating to costs of production and marketing
Price regulation of Non-Scheduled Drugs: Apart from the scheduled medicines under DPCO, 1995, the NPPA monitors the prices of other medicines not
listed in the DPCO schedule, such that they do not have a price variation of more than 10% per annum.
Uniform VAT of 4% on medicines: Government has xed a uniform and low rate of 4% VAT on medicines in the country. This policy has been adopted, in
almost all the States in the country, and has reduced the incidence of sales tax on medicines and thereby assisted in keeping their prices low.
The government has decided to launch a country wide Jan Aushadhi Campaign.

2. Union Human Resource Development Ministry has released 'India Rankings 2016' for the survey of educational institutions commissioned by the
government. Which of the following parameters have been used under the survey?
1. Teaching, Learning & Resources available.
2. Research, Professional Practice & Collaborative Performance of the institute.
3. Outreach & Inclusivity.
4. Industrial and vocational training.
5. Perception about the college.
Codes:

(A) 1, 2 and 3
(B) 1, 2, 3 and 4
(C) 1, 2, 3 and 5
(D) All
Selected answer: (Not selected..)
The correct answer is:(C)
Solution:The criteria for ranking included teaching/learning resources, research, graduation outcomes, outreach/inclusivity and perception. The data for the rst
four parameters, which account for 90% of the weightage, was submitted by the institutions and veried by National Institutional Ranking Framework
(NIRF), a body constituted by the HRD ministry last year to conduct annual surveys. For the perception criterion, various stakeholders including parents,
teachers, and alumni were engaged to give their feedback.

3. Which of the following statements are correct with respect to the achievement of Goal 1 of Millenium Development Goals by India?
1. India has successfully achieved Goal 1, to eradicate Extreme Poverty and Hunger.
2. India has achieved the target of reducing poverty by half, with 21.9% of its 1.2 billion people living below the poverty line.
3. Estimation of Poverty for this Goal was based on Global Poverty Line of less than $1.0 a day.
Select the correct answer using the codes given below:
Codes:

(A) 1 and 3
(B) 2 and 3
(C) Only 1
(D) Only 2
Selected answer: (Not selected..)
The correct answer is:(D)
Solution:India has partially achieved Goal 1 of Eradicate Extreme Poverty and Hunger.
India has been moderately successful in reducing poverty. In 1990, the all India Poverty Head Count Ratio (PHCR) was estimated to be 47.8%. In order to
meet the 2015 target, the PHCR level has to be 23.9%. In 2011-12, the PHCR was 21.9%. This indicates that, India has achieved the poverty reduction
target, however, progress is uneven. Faster reduction in poverty since the mid-2000s helped India halve the incidence of poverty.
However in case of eradicating hunger remains a key challenge. In 1990, when the MDGs were formulated, 53.5 percent of all Indian children were
malnourished. Since then, progress has been slow. In India, the proportion of underweight children below three years has declined marginally between
1998-99 and 2005-06 to 46 percent. In 2015, malnourishment declined to 40 percent. This is still below the target of reducing malnourishment to 26
percent.
Global Poverty Line was $1.25 a day for MDGs but recently World Bank changed it to $1.9 a day.

4. What is the concept of "Tithi bhojan" proposed by the Government?

(A) Government provides food to all village poor on a special occasion.


(B) Involving local community and charitable institutions to improve the quality of food in Mid Day Meal programme.
(C) Providing culturally acceptable food in schools.
(D) It is a scheme for improving nutritional security in tribal areas.

Selected answer: (Not selected..)


The correct answer is:(B)
Solution:Under this initiative, food is voluntarily served among school children by villagers in several forms like sweet and namkeen with the regular midday meal
and supplementary nutritive items like sprouted beans. The objective is to inculcate the feeling of equity and brotherhood among the children of all
communities and reduction of the gap between the school administration and the community.

5. The excessive food stock in the central pool has been optimized through various policy measures. Which of the following measure can be used to
optimize the central food stock?
1. Generating competitiveness in the domestic grain market.
2. Open market sale.
3. Lowering rate of imposition of levy for rice in certain states.
4. Liquidation of excess stock through export.
Codes:

(A) 1 and 2
(B) 2, 3 and 4
(C) 1, 3 and 4
(D) All
Selected answer: (Not selected..)
The correct answer is:(D)
Solution:Due to an open ended procurement policy of food grains providing price support to every farmer, who wants to sell his produce to the Government at
MSP, usually Government agencies end up in procuring more food grains. This has led to accumulation of excess stock under central pool as against the
required buer norms. As excess stock involve carrying cost which adds to the burden of food subsidy on the Government, it is necessary that they are

=133388 1/6
3/27/2017 IASScore:PracticeTests
required buer norms. As excess stock involve carrying cost which adds to the burden of food subsidy on the Government, it is necessary that they are
liquidated from time to time either through sale in the open domestic market or through exports. Liquidation of excess stocks under Open Market Sale
Scheme-Domestic (OMSS-D) also helps in keeping food ination under check and controlling prices of food grains during o season, especially in the
decit areas.

6. Trans fats are a type of unsaturated fats that have high health side-eects then also why is it used in food processing industry?
1. It give products a longer shelf life.
2. It helps in improving the texture and avour of the food.
3. It help in preserving the milk products.
Codes:

(A) Only 1
(B) 1 and 2
(C) 2 and 3
(D) All
Selected answer: (Not selected..)
The correct answer is:(B)
Solution:It is used mainly for frying to give longer shelf life & enhancing texture & avour.

7. Recently 62 new Navodaya Vidyalayas have been proposed in Union Budget to promote quality education. Consider the following statements related to
it:
1. Its objective is to educate children of the Central Government Employees who are often posted to remote locations to provide uniform education.
2. These are fully residential and co-educational schools aliated to Central Board of Secondary Education (CBSE).
3. Navodaya Vidyalayas exist all over India except Jammu and Kashmir.
Which of the above statements are incorrect?

(A) 1 and 2
(B) Only 3
(C) 1 and 3
(D) Only 1
Selected answer: (Not selected..)
The correct answer is:(C)
Solution:The objective of Kendriya Vidyalaya is to educate children of the Central Government Employees who are often posted to remote locations to provide
uniform education. JNVs exist all over India, with the exception of Tamil Nadu.

8. Which of the following modications have been brought in the 2010 HDI report over the 1990 report?
1. Income measure was changed from Gross National Income to Gross Domestic Product.
2. The 2010 report considers mean years of schooling and expected years of schooling for calculation of knowledge parameter.
Codes:

(A) Only 1
(B) Only 2
(C) Both
(D) None
Selected answer: (Not selected..)
The correct answer is:(B)
Solution:It measures living standards by Gross National Income Per Capita.

9. Which of the following committees have been formed for the Poverty Estimation In India?
1. Alagh Committee
2. Saxena Committee
3. Hashim Committee
4. Naresh Chandra Committee
Select the correct answer using the codes given below:
Codes:

(A) 1, 2 and 3
(B) 2 and 3
(C) 1, 3 and 4
(D) All
Selected answer: (Not selected..)
The correct answer is:(A)
Solution:Naresh Chandra Committee was formed in 2012 to reform Defence Sector.
Following are the committees formed for determination of poverty:
Alagh Committee (1977)
Lakdawala Committee (1989)
Tendulkar Committee (2005)
Saxena committee (2009)
Hashim Committee (2012)
C. Rangrajan Committee (2012)

10. Consider the following statements about "Global Monitoring Report":


1. It examines the impact of demographic change on achieving World Development Goals.
2. The report in 2015 was jointly produced by World Bank and IMF.
3.With the revised poverty line, the recent report stated that the number of people below poverty line has almost doubled in comparison to 2012 gure.
Which of the above statements is/are correct?

(A) Only 1
(B) 1 and 2
(C) All
(D) 2 and 3
Selected answer: (Not selected..)
The correct answer is:(B)
Solution:Global Monitoring Report 2015 , produced jointly by the World Bank and International Monetary Fund, details the progress the world has made towards
global development goals and examines the impact of demographic change on achieving these goals.
The report details the decline of those living in global poverty, which is reclassied as living on $1.90 or less a day, to a forecast 9.6 percent of the world's
population in 2015 -- a projected 200 million fewer people living in extreme poverty than in 2012. It also revises world economic growth projections for
2015 down to 3.3 percent on the basis of lower growth prospects in emerging markets.

11. Which of the following is/are correct about SABLA scheme?


1. It is implemented by Ministry of Human Resource Development.
2. The scheme targets at adolescent girls for improvement of their nutrition and health status.
3. It aims at improving various vocational skills by integrating with National Skill Development Program.
4. It has provisions for subsidized bank loans for adolescent girls to start self employment business.
Codes:

(A) 1, 3 and 4
(B) 1, 2 and 4
=133388 2/6
3/27/2017 IASScore:PracticeTests
1, 2 and 4
(C) 1, 2 and 3
(D) All
Selected answer: (Not selected..)
The correct answer is:(C)
Solution:The objectives of the program are:
Enable the Adolescent girls for self-development and empowerment
Improve their nutrition and health status.
Promote awareness about health, hygiene, nutrition, adolescent reproductive and sexual health (ARSH) and family and child care.
Upgrade home-based skills, life skills and integrate with the National Skill Development Program (NSDP) for vocational skills.
Mainstream out of school adolescent girls into formal/non formal education.
Provide information/guidance about existing public services such as PHC, CHC, Post Oce, Bank, Police Station, etc.

12. Which of the following Indices are presented by Human Development Report?
1. Gender Development Index.
2. Multi Dimensional Poverty Index.
3. Gender Inequality Index.
4. Inequality Adjusted HDI.
Codes:

(A) 1 and 4
(B) 1, 2 and 4
(C) 2 and 4
(D) All
Selected answer: (Not selected..)
The correct answer is:(D)
Solution:To measure human development more comprehensively, the Human Development Report also presents four other composite indices such as:
The Inequality-adjusted HDI discounts the HDI according to the extent of inequality.
The Gender Development Index compares female and male HDI values.
The Gender Inequality Index highlights women's empowerment.
The Multidimensional Poverty Index measures non income dimensions of poverty.

13. Consider the following statements with respect to National Family Health Survey- 4 (NFHS-4):
1. There is a decline in number of Under 5 wasting.
2. Number of people suering from hypertension is more in urban areas than in rural areas.
3. Alcohol and tobacco consumption has fallen since NFHS-3.
4. There is a fall in number of fully immunized children in rich states like Haryana and Maharashtra.
Which of the above statements is/are incorrect?

(A) Only 1
(B) Only 2
(C) 1 and 3
(D) None
Selected answer: (Not selected..)
The correct answer is:(B)
Solution:Under 5: The survey found that there were reduced levels of children under 5 who were wasted., decreased under 5 mortality, increased institutional
deliveries, rise in fully immunized children, increase in children who were breast feeded., increased ante-natal care.
However, the issues which are a cause of concern are slow reduction in child stunting, anaemia, rapid increase in male obesity.
Non communicable disease: Consistent with the burden of non-communicable diseases in India, over-nutrition or obesity among adults has emerged as
a major concern. At least three in 10 women are overweight or obese in the Andaman and Nicobar Islands, Andhra Pradesh, Goa.
Hypertension: While obesity levels have shot up in the country since the last NFHS survey in 2005-06, the number of people suering from hypertension
in rural India is, in many cases, higher than in urban parts.
Among women, obesity Levels shot up from 13.92 per cent in 2005-06 to 19.56 per cent in 2015-16. For men, the rise from the last decade has been
from 10.35 per cent to 18.04 per cent.
Alcohol: Men and women across India smoke and drink less than what they used a decade ago.
Tobacco use fallen from 50% to 47%. India has a part of global campaign against tobacco use has introduced a series of measures to control and
discourage the use- increase social awareness, ban on smoking in public places and working areas., establishing smoking zones in hotels, airports and
restaurants. (India is a party to WHO Framework Convention on Tobacco control (FCTC)).

14. The World Development Report (World Bank) highlighted that "To get the most out of the digital revolution, countries need to work on the "analog
complements". Which of the following comes under the analog complements?
1. Strengthening regulations that ensure competition among businesses.
2. Adapting workers' skills to the demands of the new economy.
3. Ensuring that institutions are accountable.
Codes:

(A) Only 1
(B) 1 and 2
(C) All
(D) None
Selected answer: (Not selected..)
The correct answer is:(C)
Solution:Digital technologies have spread rapidly in much of the world. Digital dividends-the broader development benets from using these technologies-have
lagged behind. In many instances digital technologies have boosted growth, expanded opportunities, and improved service delivery. Yet their aggregate
impact has fallen short and is unevenly distributed. For digital technologies to benet everyone everywhere requires closing the remaining digital divide,
especially in internet access. But greater digital adoption will not be enough. To get the most out of the digital revolution, countries also need to work on
the "analog complements"-by strengthening regulations that ensure competition among businesses, by adapting workers'skills to the demands of the
new economy, and by ensuring that institutions are accountable.

15. What is "Digital dividend" as emphasized by World Bank?

(A) It is the high rate of return investors get on their investment in long term digital infrastructures.
(B) It is the income generated by Government through improved service delivery using digital technology.
(C) It is the improvement in growth rate, jobs and services in return to investment in digital technology.
(D) It is the extra income that internet service providers accrue due to higher internet use.
Selected answer: (Not selected..)
The correct answer is:(C)
Solution:What are the digital dividends?
Growth, jobs, and services are the most important returns to digital investments. With digital technologies help businesses become more productive;
people nd jobs and greater opportunities; and governments deliver better public services to all.
By reducing information costs, digital technologies greatly lower the cost of economic and social transactions for rms, individuals, and the public sector.
They promote innovation when transaction costs fall to essentially zero. They boost eciency as existing activities and services become cheaper, quicker,
or more convenient. And they increase inclusion as people get access to services that previously were out of reach.

16. Consider the following statements related to Human Development Report, 2015 with respect to India:
1. India's HDI has improved 5 notches to 130.
2. The improvement is mainly because of improvement in Life expectancy and Per Capita Income.
3. India still falls in "Low Human Development category".
Which of the above statements is/are correct?

(A)
=133388 3/6
3/27/2017 IASScore:PracticeTests
(A) 1 and 2
(B) 2 and 3
(C) 1 and 3
(D) All
Selected answer: (Not selected..)
The correct answer is:(A)

Solution:India continued to rank low in the Human Development Index (HDI), but climbed ve notches to the 130th rank in the latest UNDP report on account of
rise in life expectancy and per capita income.
India ranked 130 among 188 countries in 2014 in Human Development Report 2015 by the United Nations Development Programme (UNDP). The
country's rank was 135 according to the 2014 report.
Between 1980 and 2014, India's HDI value increased from 0.362 to 0.609, an increase of 68.1 per cent or an average annual increase of about 1.54 per
cent. India is in medium human development category.

17. What are the main objectives of "policy on food grain stocking"?
1. To meet the prescribed minimum food grain stocking norms for food security.
2. To release food grains for supply through TPDS monthly.
3. To meet emergency situation arising out of unexpected crop failure and natural disasters.
4. For market intervention to augment supply.
Codes:

(A) Only 1
(B) 2 and 3
(C) 1, 2 and 3
(D) All
Selected answer: (Not selected..)
The correct answer is:(D)
Solution:Buer stock of food grains in the Central Pool is maintained by the Government of India (GOI)/Central Government for:
Meeting the prescribed minimum buer stock norms for food security,
Monthly release of food grains for supply through Targeted Public Distribution System (TPDS) and Other Welfare Schemes (OWS),
Meeting emergency situations arising out of unexpected crop failure, natural disasters, etc., and
Price stabilisation or market intervention to augment supply so as to help moderate the open market prices.

18. Which of the following statements are correct about Wheat Based Nutrition programme?
1. The scheme is implemented by Ministry of Food and Civil Supplies.
2. The foodgrains allotted under this scheme are utilized by States under ICDS.
Codes:

(A) Only 1
(B) Only 2
(C) Both
(D) None
Selected answer: (Not selected..)
The correct answer is:(B)
Solution:This Scheme is implemented by the Ministry of Women & Child Development. The foodgrains allotted under this Scheme are utilized by the States/UTs
under the Integrated Child Development Scheme (ICDS) for providing nutritious/ energy food to children below 6 years of age and expectant /lactating
women from disadvantaged sections.

19. Which of the following is/are part of 5 fold principles of tribal development in India, put forward by Verrier Elwin?
1. People should develop along the line of their own genius and nothing should be imposed on them from outside.
2. Tribal rights to land and forest should be respected.
3. Tribals should be gradually assimilated in mainland development process.
4. Tribal lands should not be over administered.
Codes:

(A) 1 and 2
(B) 2, 3 and 4
(C) 1, 2 and 4
(D) All
Selected answer: (Not selected..)
The correct answer is:(C)
Solution:The principles are:
People should develop along the line of their own genius and we should avoid imposing anything on them. We should try to encourage in every way
their own traditional arts and culture.
Tribal rights to land and forest should be respected.
We should try to train and build up a team of their own people to do the work of administration and development. Some technical personnel from
outside will no doubt, be needed, especially in the beginning. But we should avoid introducing too many outsiders into tribal territory.
We should not over administer these areas or overwhelm them with a multiplicity of schemes. We should rather works through, and not in rivalry to,
their own social and cultural institutions.
We should judge results, not by statistics or the amount of money spent, but by the quality of human character that is evolved.

20. According to the United Nations, World Water Development Report 2016 which of the following are the likely impacts of water scarcity?
1. Unsustainable management of water may lead to reversing many poverty reduction, job creation and hard-won development gains.
2. Water scarcity will aect geopolitical security due to prompting of migration at various scales.
3. Reduced water availability will impact the women empowerment initiatives.
Codes:

(A) 1 and 3
(B) Only 2
(C) 1 and 2
(D) 1, 2 and 3
Selected answer: (Not selected..)
The correct answer is:(D)

Solution:Water scarcity is likely to limit opportunities for economic growth and the creation of decent jobs in the upcoming years and decades. Unless there is
sucient infrastructure to manage and store the water, as is the case in many developed countries, water availability might vary signicantly, leaving
(parts of) countries 'water scarce' for extended periods. Water availability is also highly dependent on water quality. Poor quality water may not be t for
several uses and the cost of the required treatment may be a prohibiting factor, thus contributing to the burden of economic water scarcity.
Reduced water availability will further intensify competition for water among users, including agriculture, maintenance of ecosystems, human
settlements, industry and energy production. This will aect regional water, energy and food security, and potentially geopolitical security, prompting
migration at various scales. The potential impacts on economic activity and the job market are real and possibly severe. Many developing economies are
located in hotspots of water-related stress, particularly in Africa, Asia, Latin America and the Middle East.

21. Consider the following statements regarding the comparison of India's human development indices with that of other Asian nations:
1. India's Healthy life expectancy at birth is signicantly lower than China, Vietnam and Bangladesh.
2. Indonesia has much higher purchasing power parity (PPP) adjusted per capita national income than India.
3. India's under-ve mortality level is markedly higher than Pakistan.
Which of the above statements is/are correct?
=133388 4/6
3/27/2017 IASScore:PracticeTests
3. India's under-ve mortality level is markedly higher than Pakistan.
Which of the above statements is/are correct?

(A) Only 1
(B) Only 2
(C) 2 and 3
(D) 1 and 2
Selected answer: (Not selected..)
The correct answer is:(D)
Solution:India's under-ve mortality level is markedly higher than in all these countries except Pakistan; four times higher than China's and over double Vietnam's.

22. Which of following statements is/are correct regarding Biwako Millennium Framework?
1. It is action by Governments in the African region to achieve an inclusive, barrier-free and rights-based society for Persons with Disabilities in the
new decade, 2015-2030.
2. This regional framework for action focusses on rights based society for Persons with Disability.
Codes:

(A) Only 1
(B) Only 2
(C) Both
(D) None
Selected answer: (Not selected..)
The correct answer is:(B)
Solution:Biwako framework is for promoting an inclusive, barrier-free and rights-based society for people with disabilities in the Asian and Pacic region in the
twenty-rst century, by which it proclaimed the extension of the Asian and Pacic Decade of Disabled Persons, 1993-2002, for another decade, 2003-2012.

23. Which of following statements is/are correct regarding 'Gift Basket Diplomacy'?
1. It is an approach to multilateral negotiation aimed at pushing forward progress on a particular issue without the requirement of consensus.
2. It is extension of Gujaral doctrine where Developed Nation gives more to the developing nation.
3. It is collective action agreed by smaller groups of participants in multilateral fora without support of major nations.
Codes:

(A) 1 and 3
(B) Only 1
(C) Only 3
(D) 2 and 3
Selected answer: (Not selected..)
The correct answer is:(A)
Solution:Gift basket diplomacy is an approach to multilateral negotiation aimed at pushing forward progress on a particular issue without the requirement of
consensus. The policy is most often seen in United Nations style diplomatic meetings where a particular group of countries wishes to take action or make
a joint statement but is unable to do so without the consensus of all parties involved. Gift basket diplomacy fundamentally is collective action agreed by
smaller groups of participants that goes beyond the lowest common denominator consensus that larger groups often reach in large multilateral fora. The
United States rst introduced Gift basket diplomacy in 2011 during the Nuclear Security Summit preparation process and more than 30 countries
participated in Gift basket diplomacy statements at the 2012 Nuclear Security Summit in Seoul.

24. Which of the following institutes are engaged in food and public distribution?
1. Food Corporation of India.
2. Central Warehousing Corporation.
3. Indian Grain Storage Management & Research Institute.
4. Rice Research Institute.
Codes:

(A) 1 and 2
(B) 1 and 3
(C) 1, 2 and 3
(D) All
Selected answer: (Not selected..)
The correct answer is:(C)
Solution:.

25. Consider the given statements about antibiotic resistance:


1. It occurs when an antibiotic loose its ability to eectively control or kill bacterial growth.
2. Resistance occurs due to genetic mutation.
3. The current higher levels of antibiotic resistant bacteria are attributed to overuse and abuse of antibiotics.
4. A bacteria after becoming antibiotic resistant cannot lose its resistance traits i.e. the process of antibiotic resistance cannot be reversed.
Which of the above statements is/are correct?

(A) 1, 2 and 3
(B) 1, 3 and 4
(C) 1, 2 and 4
(D) All
Selected answer: (Not selected..)
The correct answer is:(A)
Solution:Bacteria can lose its traits and Antibiotic Resistance can be reversed. Resistant microorganisms (including bacteria, fungi, viruses and parasites) are able to
withstand attack by antimicrobial drugs such as antibacterial drugs (e.g. antibiotics), anti fungal, anti viral and anti malarial. During Antibiotic Resistance
standard treatments become ineective and infections increases the risk of spread to others.

Information for the solved exam

Started in: March 27, 2017, 12:27 am

Finished in: March 27, 2017, 12:27 am

Marks Obtained: 0

Right Ques.: 0

Wrong Ques.: 0

Unattempted Ques.: 25

Try for a better result? (index.php?section=user&page=continue_exam&id=133388&renew_exam=yes)

=133388 5/6
3/27/2017 IASScore:PracticeTests

Rank List (index.php?section=user&page=ranking&id=145)

=133388 6/6

Você também pode gostar